Download as pdf or txt
Download as pdf or txt
You are on page 1of 213

Problem 8.

1 [Difficulty: 1]

Given: Air entering duct

Find: Flow rate for turbulence; Entrance length

Solution:
V⋅ D π 2
The basic equations are Re = Recrit = 2300 Q= ⋅D ⋅V
ν 4

2
−5 m
The gi ven data is D = 125⋅ mm From Table A.10 ν = 2.29 × 10 ⋅
s

Llaminar = 0.06⋅ Recrit⋅ D or, fo r turbulent, L turb = 25D to 40D

Q
⋅D
π 2
⋅D Recrit⋅ π⋅ ν ⋅ D 3
4 −3m
Hence Recrit = or Q = Q = 5.171 × 10
ν 4 s

For laminar flow Llaminar = 0.06⋅ Recrit⋅ D Llaminar = 17.3 m

For turbulent flow Lmin = 25⋅ D Lmin = 3.13 m Lmax = 40⋅ D Lmax = 5.00 m
Problem 8.2

Given: Reynolds number in a section where the tube diameter is 8mm

To find: Reynolds number for the flow rate in a section where the tube diameter is 5mm

Solution:

Assumptions:

1) Steady
2) Incompressible flow

Calculations:-

DV  D2
Basic equations: Re   , Q  A V , m   AV , and A 
 4

Then,

DV D Q D Q4 4Q
Re       
  A   D  D
2

4Q
Re 
 DV
Also
DV D VA D m4 4m
Re      
  A   D  D
2

4m
Re 
 D

From equation (1)

 DV Re
Q
4

Then for same flow rate in section with different channel diameter
D1 Re1  D 2 Re 2
D1 8mm
Re 2  Re1   2000
D2 5mm
Re 2  3200
Problem 8.3 [Difficulty: 3]

Given: Air entering pi pe system

Find: Flow rate for turbulence in each section; Which become fully developed

Solution:
2
−5 m
From Table A.10 ν = 1.69 × 10 ⋅
s

The gi ven data is L = 2⋅ m D1 = 25⋅ mm D2 = 15⋅ mm D3 = 10⋅ mm

The critical Reynolds number is Recrit = 2300

Writing the Reynolds nu mber as a function of flow rate

V⋅ D Q D Re⋅ π⋅ ν⋅ D
Re = = ⋅ or Q=
ν π 2 ν 4
⋅D
4
Then the flow rates for turbulence to begin in each section of pipe are

Recrit⋅ π⋅ ν⋅ D1 3
Q1 = −4m
4 Q1 = 7.63 × 10
s

Recrit⋅ π⋅ ν⋅ D2 3
Q2 = −4m
4 Q2 = 4.58 × 10
s

Recrit⋅ π⋅ ν⋅ D3 3
Q3 = −4m
4 Q3 = 3.05 × 10
s

Hence, smallest pipe becomes turbulent first, then second, then the largest.

For the smallest pipe transitioning to turbulence (Q3)

For pipe 3 Re3 = 2300 Llaminar = 0.06⋅ Re3 ⋅ D3 Llaminar = 1.38 m Llaminar < L: Fully developed

or, fo r turbulent, Lmin = 25⋅ D3 Lmin = 0.25 m Lmax = 40⋅ D3 Lmax = 0.4 m Lmax/min < L: Fully developed

⎛ 4⋅ Q3 ⎞
For pipes 1 and 2 Llaminar = 0.06⋅ ⎜ ⋅ D1 Llaminar = 1.38 m Llaminar < L: Fully developed
π⋅ ν⋅ D1
⎝ ⎠
⎛ 4⋅ Q3 ⎞
Llaminar = 0.06⋅ ⎜ ⋅ D2 Llaminar = 1.38 m Llaminar < L: Fully developed
π⋅ ν⋅ D2
⎝ ⎠

For the middle pipe transitioning to turbulence (Q2)

For pipe 2 Re2 = 2300 Llaminar = 0.06⋅ Re2 ⋅ D2 Llaminar = 2.07 m Llaminar > L: Not fully developed

or, fo r turbulent, Lmin = 25⋅ D2 Lmin = 1.23⋅ ft Lmax = 40⋅ D2 Lmax = 0.6 m

Lmax/min < L: Fully developed

⎛ 4 ⋅ Q2 ⎞
For pipes 1 and 3 L1 = 0.06⋅ ⎜ ⋅ D1 L1 = 2.07⋅ m Llaminar > L: Not fully developed
π⋅ ν⋅ D1
⎝ ⎠
L3min = 25⋅ D3 L3min = 0.25⋅ m L3max = 40⋅ D3 L3max = 0.4 m

Lmax/min < L: Fully developed

For the large pipe transitioning to turbulence (Q1)

For pipe 1 Re1 = 2300 Llaminar = 0.06⋅ Re1 ⋅ D1 Llaminar = 3.45 m Llaminar > L: Not fully developed

or, fo r turbulent, Lmin = 25⋅ D1 Lmin = 2.05⋅ ft Lmax = 40⋅ D1 Lmax = 1.00 m

Lmax/min < L: Fully developed

For pipes 2 and 3 L2min = 25⋅ D2 L2min = 1.23⋅ ft L2max = 40⋅ D2 L2max = 0.6 m

Lmax/min < L: Fully developed

L3min = 25⋅ D3 L3min = 0.82⋅ ft L3max = 40⋅ D3 L3max = 0.4 m

Lmax/min < L: Fully developed


Problem 8.4 [Difficulty: 4] Part 1/2
[Difficulty: 4] Part 2/2
Problem 8.5 [Difficulty: 2]
Problem 8.6 [Difficulty: 2]
Problem 8.7 [Difficulty: 3]

8.7 A fluid flows steadily between two parallel plates. The flow
is fully developed and laminar. The distance between the plates
is h. y
x
(a) Derive an equation for the shear stress as a function of y. h 5 1.3 mm
Sketch this function.
2
(b) For µ = 1.15 N⋅s/m , ∂p/∂x = −58 Pa/m, and h = 1.3 mm,
calculate the maximum shear stress, in Pa.

h 2 ∂p   2 y  
2
Solution: From Eq. 8.7, with a = h, u = − 1 −   
8µ ∂x   h  
By symmetry, the origin for y must be located at the channel centerline. Apply Newton’s law of viscosity.

du
τ yx = µ
dy

Assumption: Newtonian fluid

d  h2 ∂p   2 y    τ yx
2
∂p
Then τ y x = µ − 1 −     = y ←
dy  8µ ∂x   
h   ∂x
 

For u > 0, ∂p/∂x < 0. Thus τyx < 0 for y > 0 and τyx > 0 for y < 0.

On the upper plate (a minus y surface), τyx < 0, so shear stress acts to the right.

On the lower plate (a plus y surface), τyx > 0, so shear stress acts to the right.

The maximum stress occurs when y = ± h/z. Thus

 h h ∂p 1 m  Pa  τmax
τ max = τ y x   = = × 1.3 mm × ×  −58  = −0.038 ← 
 2  2 ∂x 2 1000 mm  m

 h
or τ max = τ y x  −  = 0.038 Pa
 2
Problem 8.8 [Difficulty: 3]

8.8 Oil is confined in a 100 mm diameter cylinder by a piston


having a radial clearance of 0.025 mm and a length of 50 mm.
A steady force of 20,000 N is applied to the piston. Assume the
properties of SAE 30 oil at 49°C. Estimate the rate at which oil
leaks past the piston.

Given: Piston cylinder assembly.

Find: Rate of oil leak.

Solution:
Basic equation Q a 3 ⋅ ∆p π ⋅ D ⋅ a 3 ⋅ ∆p (from Eq. 8.6c; we assume laminar flow
= Q= and verify this is correct after solving)
1 12 ⋅ µ ⋅ L 12 ⋅ µ ⋅ L

F 4⋅F
For the sytem ∆p = p1 − patm = =
A π ⋅ D2

2
4  1 1000 mm 
∆p = × 20, 000 N ×  ×  ∆p = 2546.5 kPa
π  100 mm 1m 

N ⋅s
At 49°C from Fig. A2 µ = 0.06
m2

3
π  1m  m2 1 m3 mm3
Q= × 100 mm ×  0.025 mm ×  × 2546.5 kPa × × Q = 3.47 × 10 −7 Q = 347
12  1000 mm  0.06 N ⋅ s 50 mm s s

2
Q Q 1 m3 1 1  1000 mm  m
Check Re: V= = V= × 3.47 × 10−7 × × ×  V = 0.044
A a⋅π⋅D π s 0.025 mm 100 mm  1 m  s

V ⋅a m2
Re = ν = 6 × 10 −5 (at 49°C, from Fig. A.3)
ν s

m 1m 5
Re = 0.044 × 0.025 mm × × Re = 0.0183 so flow is very much laminar
s 1000 mm 6 × 10−5 m 2

The speed of the piston is approximately

2
Q 4 m3  1 1000 mm  m
Vp = Vp = × 3.47 ×10−7 × ×  Vp = 4.42 × 10 −5
 π⋅D 2  π s  100 mm 1m  s
 
 4 

The piston motion is negligible so our assumption of flow between parallel plates is reasonable.
Problem 8.9 [Difficulty: 2]
Problem 8.10

Given: Laminar flow between flat plates

To find: Shear stress on upper plate; Volume flow rate per width.

Solution:

Basic Equation:

du
Tyx  
dy
h 2 dP   y  
2

u  y    
   
1
2  dx   h  
Then,
h 2 dP  2 y 
Tyx     
2 dx  h 2 
dP
 y
dx
At the upper surface; y  h
1m
Tyx  0.2 mm  1.50 103 N/m 2  m
1000 mm
 3.0 Pa

Hence, the shear stress on upper plate is 3.0 Pa .

The volume flow rate is


h
Q   udA   u  bdy
h

h 2  b dP h   y  
2

2 dx  h   h  
  1     dy

2h3b dP
 
3 dx
Q
Substitute corresponding values in the expression of
b
3
2   m2 
    0.7 N  s 
Q 1
   2.0 mm  1.50  10 3
N/m 2
 m
b 3 1000 mm   
 4.02 106 m 2 /s

Hence, the volume rate per width is 4.02 106 m2 /s .


Problem 8.11 [Difficulty: 3]

8.11 A large mass is supported by a piston of diameter D = 100


mm and length L = 100 mm. The piston sits in a cylinder closed at
the bottom, and the gap a = 0.025 mm between the cylinder wall
and piston is filled with SAE 10 oil at 20°C. The piston slowly
sinks due to the mass, and oil is forced out at a rate of 6 × 10−
6

3
m /sec. What is the mass (kg)?

Given: Piston-cylinder assembly.

Find: Mass supported by piston.

Solution:

Q a 3 ⋅ ∆p
Basic equation = This is the equation for pressure-driven flow between parallel plates;
1 12 ⋅ µ ⋅ L
for a small gap a, the flow between the piston and cylinder can be modeled this way, with l = πD.

m3
Available data L = 100 mm D = 100 mm a = 0.025 mm Q = 6 × 10−6
s

N ⋅s
From Fig. A.2, SAE 10 oil at 20°C is µ = 0.1⋅
m2

12 ⋅ µ ⋅ L ⋅ Q
Hence, solving for ∆p ∆p = ∆p = 146.677 MPa
π ⋅ D ⋅ a3

π 2
A force balance for the piston involves the net pressure force F = ∆p ⋅ A = ∆p ⋅ ⋅D and the weight W = M ⋅g
4

π ⋅ D 2 ∆p
Hence M= ⋅ M = 117.43 × 103 kg
4 g

Q m m2
Note the following Vave = Vave = 0.764 ν = 10 −4 ⋅
a ⋅π⋅D s s

a ⋅ Vave
Hence an estimate of the Reynolds number in the gap is Re = Re = 0.191
ν

This is a highly viscous flow; it can be shown that the force on the piston due to this motion is much less than that due to ∆p!

4⋅Q m
Note also that the piston speed is Vpiston = 2
Vpiston = 0.000764
π⋅D s

Vpiston
= 0.1 ⋅ % so the approximation of stationary walls is valid.
Vave
Problem 8.12 [Difficulty: 3]
Problem 8.13 [Difficulty: 3]

8.13 A hydrostatic bearing is to support a load of 50,000 N/m of


length perpendicular to the diagram. The bearing is supplied
with SAE 10W-30 oil at 35°C and 700 kPa through the central
slit. Since the oil is viscous and the gap is small, the flow may
be considered fully developed. Calculate (a) the required width
of the bearing pad, (b) the resulting pressure gradient, dp/dx,
and (c) the gap height, if the flow rate is Q = 1 mL/min/m.

Given: Hydrostatic bearing.

Find: Required pad width; Pressure gradient; Gap height.

Solution:

Q h3  dp 
Basic equation = ⋅ 
1 12 ⋅ µ  dx 

mL
Available data F = 50, 000 N l = 0.3 m (F is the load on width l) pi = 700 kPa Q =1 per meter
min

N ⋅s
At 100°C from Fig. A.2, for SAE 10-30 µ = 0.01⋅
m2

For a laminar flow (we will verify this assumption later), the pressure gradient is constant.

 2.x 
p(x) = pi ⋅ 1 −  where pi = 700 kPa is the inlet pressure (gage), and x = 0 to W/2
 W

Hence the total force in the y direction due to pressure is ∫


F = 1⋅ pdx where b is the pad width into the paper.

W
 2⋅ x 1
F = 2 ⋅ 1⋅ ∫0
2
pi ⋅  1 −
  dx
W
F=
2
⋅ pi ⋅ 1 ⋅ W

2 F
This must be equal to the applied load F. Hence W= ⋅ W = 0.143 m
pi 1

dp ∆p 2 ⋅ ∆p 700 ×103 N 1 MPa


The pressure gradient is then =− w =− = −2 × 2
× = −9.79
dx 2
W m 0.143 m m
1
 Q 3
Q h 3  dp   12 ⋅µ ⋅ 1 
From the basic equation =− ⋅ we can solve for h = −  h = 1.452 × 10 −5 m
1 12 ⋅µ  dx   dp 
 dx 
V⋅D D Q h Q
Check Re: Re = = ⋅ = ⋅
ν ν A ν 1⋅ h
m2 Q
From Fig. A3 ν = 1.2 × 10 −5 ⋅ Re = Re = 0.72
s ν ⋅1
so flow is very laminar.
Problem 8.14 [Difficulty: 3]
Problem 8.15 [Difficulty: 2]

Given: Navier-Stokes Equations

Find: Derivation of Eq. 8.5

Solution:

The Navier-Stokes equations are


4 3
∂u ∂v ∂w
+ + =0 (5.1c)
∂x ∂y ∂z
1 4 5 3 6 4 3
⎛ ∂u ∂u ∂u ∂u ⎞ ∂p ⎛∂ u ∂ u ∂ u⎞2 2 2
ρ ⎜⎜ +u +v + w ⎟⎟ = ρg x − + µ ⎜⎜ 2 + 2 + 2 ⎟⎟ (5.27a)
⎝ ∂t ∂x ∂y ∂z ⎠ ∂x ⎝ ∂x ∂y ∂z ⎠
1 4 5 3 4 5 3
⎛ ∂v ∂v ∂v ∂v ⎞ ∂p ⎛ ∂ 2v ∂ 2v ∂ 2v ⎞
ρ ⎜⎜ + u + v + w ⎟⎟ = ρg y − + µ ⎜⎜ 2 + 2 + 2 ⎟⎟ (5.27b)
⎝ ∂t ∂x ∂y ∂z ⎠ ∂y ⎝ ∂x ∂y ∂z ⎠
1 3 3 3 3 3 3 3 3
⎛ ∂w ∂w ∂w ∂w ⎞ ∂p ⎛∂ w ∂ w ∂ w⎞ 2 2 2
ρ ⎜⎜ +u +v +w ⎟⎟ = ρg z − + µ ⎜⎜ 2 + 2 + 2 ⎟⎟ (5.27c)
⎝ ∂t ∂x ∂y ∂z ⎠ ∂z ⎝ ∂x ∂y ∂z ⎠

The following assumptions have been applied:

(1) Steady flow (given).


(2) Incompressible flow; ρ = constant.
(3) No flow or variation of properties in the z direction; w= 0 and ∂/∂z = 0.
(4) Fully developed flow, so no properties except pressure p vary in the x direction; ∂/∂x = 0.
(5) See analysis below.
(6) No body force in the x direction; gx = 0

Assumption (1) eliminates time variations in any fluid property. Assumption (2) eliminates space variations in density. Assumption
(3) states that there is no z component of velocity and no property variations in the z direction. All terms in the z component of the
Navier–Stokes equation cancel. After assumption (4) is applied, the continuity equation reduces to ∂v/∂y = 0. Assumptions (3) and (4)
also indicate that ∂v/∂z = 0 and ∂v/∂x = 0. Therefore v must be constant. Since v is zero at the solid surface, then v must be zero
everywhere. The fact that v = 0 reduces the Navier–Stokes equations further, as indicated by (5). Hence for the y direction

∂p
= ρg
∂y

which indicates a hydrostatic variation of pressure. In the x direction, after assumption (6) we obtain

∂ 2u ∂p
µ 2 − =0
∂y ∂x
Integrating twice

1 ∂p 2 c1
u= y + y + c2
2 µ ∂x µ
To evaluate the constants, c1 and c2, we must apply the boundary conditions. At y = 0, u = 0. Consequently, c2 = 0. At y = a, u = 0.
Hence

1 ∂p 2 c1
0= a + a
2 µ ∂x µ
which gives
1 ∂p
c1 = − a
2 µ ∂x
and finally
a 2 ∂p ⎡⎛ y ⎞ ⎛ y ⎞⎤
2

u= ⎢⎜ ⎟ − ⎜ ⎟ ⎥
2 µ ∂x ⎢⎣⎝ a ⎠ ⎝ a ⎠⎥⎦
Problem 8.16 [Difficulty: 5]
Problem 8.17

Solution:

“Unfold” bearing since gap is small, and consider as flow between parallel plates. Apply
Newton’s law of viscosity.

Basic equation:

du
 yx  
dy

Assumption: Linear velocity profile

Then
U  ri
 yx   
r r
and
2 ri 3 L
T  ri  2 ri L yx   2 ri L yx
2

r
solving
rT

2 ri 3 L
1 min 1 1 rev 60s
  0.002m  0.3Nm     
2 3000 rev  0.027  m 0.11m 2 rad min
3 3

Ns
  0.17
m2

Bearing is sealed, so oil temperature will increase as energy is dissipated by friction. For liquids,
 decreases as  increases. Thus torque will decrease. Since it is proportional to  .
Problem 8.18 [Difficulty: 2]

8.18 Consider fully developed laminar flow between infinite


parallel plates separated by gap width d = 10 mm. The upper
plate moves to the right with speed U2 = 0.5 m/s; the lower plate
moves to the left with speed U1 = 0.25 m/s. The pressure
gradient in the direction of flow is zero. Develop an expression
for the velocity distribution in the gap. Find the volume flow
3
rate per unit depth (m /sec/m) passing a given cross section.

Given: Laminar flow between moving plates.

Find: Expression for velocity, Volume flow rate per depth.

Solution:

m m
Given data d = 10 mm U1 = 0.5 U 2 = 0.25
s s

Using the analysis of Section 8.2, the sum of forces in the x direction is

 ∂ dy  ∂ dy    ∂ dx ∂ dx 
τ + τ ⋅ −  τ − τ ⋅   ⋅ b ⋅ dx +  p − p⋅ −p+ p ⋅  ⋅ b ⋅ dy = 0
 ∂y 2  ∂y 2    ∂x 2 ∂x 2 

dτ dp d2 u
Simplifying = =0 or µ⋅ =0
dy dx dy 2

Integrating twice u = c1 ⋅ y + c2

m U1 + U 2
Boundary conditions: u(0) = − U1 c2 = −Ul c2 = −0.5 u(y = d) = U 2 c1 = c1 = 75 s −1
s d

y
Hence u(y) = (U1 + U 2 ) ⋅ − U1 u(y) = 75 ⋅ y − 0.5 u in m/s, y in m
d

d y   U1 + U 2 d 2 
The volume flow rate is ∫
Q = u dA = b ⋅ u dy ∫ Q = b⋅ ∫
0
(U
 1

+ U 2 ) ⋅
d
− U1

dx = b ⋅ 
 d
⋅ 2 − U1 ⋅ d 

(U 2 − U1 ) Q (U − U1 )
Hence Q = b⋅d ⋅ = d⋅ 2
2 b 2

m3
Q 1m 1 m Q
= 10 mm × × × (0.5 − 0.25) × = 0.00125 s
b 1000 mm 2 s b m
Problem 8.19 [Difficulty: 2]
Problem 8.20 [Difficulty: 3]

Given: Properties of two fluids flowing between parallel plates; applied pressure gradient

Find: Velocity at the interface; maximum velocity; plot velocity distribution

Solution:

dp kPa N⋅ s N⋅ s
Given data =k k = −50⋅ h = 5 ⋅ mm μ1 = 0.1⋅ μ2 = 4 ⋅ μ1 μ2 = 0.4⋅
dx m 2 2
m m
(Lower fluid is fluid 1; upper is fluid 2)

Following the analysis of Section 8.2, analyse the forces on a differential CV of either fluid

The net force is zero for steady flow, so

⎡τ + dτ ⋅ dy − ⎛ τ − dτ ⋅ dy ⎞⎤ ⋅ dx⋅ dz + ⎡p − dp ⋅ dx − ⎛ p + dp ⋅ dx ⎞⎤ ⋅ dy⋅ dz = 0
⎢ dy 2 ⎜ ⎥ ⎢ dx 2 ⎜ ⎥
⎣ ⎝ dy 2 ⎠⎦ ⎣ ⎝ dx 2 ⎠⎦

dτ dp 2
Simplifying so for each fluid d
= =k μ⋅ u =k
dy dx 2
dy

Applying this to fluid 1 (lower fluid) and fluid 2 (upper fluid), integrating twice yields

k 2 k 2
u1 = ⋅ y + c1 ⋅ y + c2 u2 = ⋅ y + c3 ⋅ y + c4
2 ⋅ μ1 2 ⋅ μ2

For convenience the origin of coordinates is placed at the centerline

We need four BCs. Three are obvious y = −h u1 = 0 (1) y=0 u 1 = u 2 (2) y=h u2 = 0 (3)

The fourth BC comes from the fact that the stress at the interface generated by each fluid is the same

du1 du2
y=0 μ1 ⋅ = μ2 ⋅ (4)
dy dy

k 2 k 2
Using these four BCs 0= ⋅ h − c1 ⋅ h + c2 c2 = c4 0= ⋅ h + c3 ⋅ h + c4 μ1 ⋅ c1 = μ2 ⋅ c3
2 ⋅ μ1 2 ⋅ μ2

Hence, a fter some al gebra

k⋅ h (μ2 − μ1) k⋅ h
2
k⋅ h (μ2 − μ1)
c1 = ⋅ c4 = − c2 = c4 c3 = ⋅
2 ⋅ μ1 (μ2 + μ1) μ2 + μ1 2 ⋅ μ2 (μ2 + μ1)
1 m 1 m
c1 = −750 c2 = 2.5 c3 = −187.5 c4 = 2.5
s s s s

The velocity distributions are then

k ⎡ 2 (μ2 − μ1)⎤ k⋅ h
2
k ⎡ 2 (μ2 − μ1)⎤ k⋅ h
2
u1( y) = ⋅ ⎢y + y ⋅ h ⋅ ⎥− u2( y) = ⋅ ⎢y + y ⋅ h ⋅ ⎥−
2 ⋅ μ1
⎣ (μ2 + μ1)⎦ μ2 + μ1 2 ⋅ μ2
⎣ (μ2 + μ1)⎦ μ2 + μ1

Evaluating either velocity at y = 0, gives the velocity at the interface

2
k⋅ h m
u interface = − u interface = 2.5
μ2 + μ1 s

The plots of these velocity distributions can be done in Excel. Typical curves are shown below

2.5
y (mm)

0 0.5 1 1.5 2 2.5 3 3.5

− 2.5

−5

u (m/s)

Clearly, u 1 has the maximum velocity, when


du1
or
(μ2 − μ1)
=0 2 ⋅ y max + h ⋅ =0
dy (μ2 + μ1)
h μ2 − μ1 ( ) m
y max = − ⋅ y max = −1.5 mm (
u max = u 1 y max) u max = 3.06
2 μ2 + μ1 ( ) s

(We could also have used Excel's Solver for this.)


Problem 8.21 [Difficulty: 3]

8.21 Two immiscible fluids are contained between infinite


parallel plates. The plates are separated by distance 2h, and the
two fluid layers are of equal thickness h: the dynamic viscosity
of the upper fluid is three times that of the lower fluid. If the
lower plate is stationary and the upper plate moves at constant
speed U = 6.1 m/s, what is the velocity at the interface? Assume
laminar flows, and that the pressure gradient in the direction of
flow is zero.

Given: Laminar flow of two fluids between plates.

Find: Velocity at the interface.

Solution:
Using the analysis of Section 8.2, the sum of forces in the x direction is

 ∂ dy  ∂ dy    ∂ dx ∂ dx 
τ + τ⋅ −τ − τ ⋅   ⋅ b ⋅ dx +  p − p⋅ −p+ p ⋅  ⋅ b ⋅ dy = 0
 ∂ y 2  ∂y 2    ∂ x 2 ∂ x 2

dτ dp d2 u
Simplifying = =0 or µ⋅ =0
dy dx dy 2

Applying this to fluid 1 (lower fluid) and fluid 2 (upper fluid), integrating twice yields u1 = c1 ⋅ y + c2 u 2 = c3 ⋅ y + c 4

We need four BCs. Three are obvious y = 0 u1 = 0 y = h u1 = u2 y = 2 ⋅ h u2 = U

The fourth BC comes from the fact that the stress at the interface generated by each fluid is the same

du1 du
y=h µ1 . = µ2 ⋅ 2
dy dy

Using these four BCs 0 = c2 c1 ⋅ h + c2 = c 3 ⋅ h + c4 U = c 3 ⋅ 2 ⋅ h + c4 µ1 ⋅ c1 = µ2 ⋅ c3

Hence c2 = 0

From the 2nd and 3rd equations c1 ⋅ h − U = −c3 ⋅ h and µ1 ⋅ c1 = µ2 ⋅ c3

µ1 U
Hence c1 ⋅ h − U = − c3 ⋅ h = − ⋅ h ⋅ c1 c1 =
µ2  µ 
h ⋅ 1 + 1 
 µ 2

U
Hence for fluid 1 (we do not need to complete the analysis for fluid 2) u1 = ⋅y
 µ 
h ⋅ 1 + 1 
 µ 2

m
6.1
s m
Evaluating this at y = h, where u1 = uinterface u interface = u interface = 4.6
 1 s
1 + 
3
Problem 8.22

Given: Compute disk drive

To find: Flow Reynolds number; Shear Stress; Power required

Solution:

For a distance R from the centre of a disk spinning at speed 

V  R
1m 2 rad 1min
V  30mm   9000rpm  
1000mm rev 60s
m
V  28.27
s

The gap Reynolds number is

 Va Va
Re  
 v
m2
v  28.27 105 from table A.10 at 15C
s
m s
Re = 28.27  0.35 106 m 
s 1.45×10-5 m 2
Re  0.682

The flow is definitely laminar

The shear stress is then,


du V
  
dy a
N s
  1.79 105 2 from table A.10 at 15°C
m
N s m 1
  1.79 105 2  28.27 
m s 0.35 106
  1.446 kPa

The power required is

P = T  where torque T is given by


T =  AR with A =  7mm   4.9 10 5 m 2
2

P =  AR
N 1m 2 rad 1min
 1446 2  4.9 105  30mm   9000rpm  
m 1000m rev 60s
P  2.0 W
Problem 8.23 [Difficulty: 2]

Given: Velocity profile between parallel plates

Find: Pressure gradients for zero stress at upper/lower plates; plot

Solution:

⎛ ∂ ⎞ ⎡⎢⎛ y ⎞ 2 y⎤
2
U⋅ y a
From Eq. 8.8, the velocity distribution is u= + ⋅⎜p ⋅ ⎜ − ⎥
a 2 ⋅ μ ⎝ ∂x ⎠ ⎣⎝ a ⎠ a⎦

2
⎛∂ ⎞
⋅ ⎜ p ⋅ ⎛ 2⋅ − ⎞
du U y 1 a
The shear stress is τyx = μ⋅ = μ⋅ +
dy a 2 ⎝ ∂x ⎠ ⎜ 2 a
a ⎝ ⎠

U a ∂ ∂ 2 ⋅ U⋅ μ
(a) For τyx = 0 at y = a 0 = μ⋅ + ⋅ p p =−
a 2 ∂x ∂x 2
a

U⋅ y
2
2 ⋅ U⋅ μ ⎡ y ⎞2 y⎤
2
⋅ ⎢⎛⎜ ⎛y⎞
a u y
The velocity distribution is then u= − ⋅ − ⎥ = 2⋅ − ⎜a
a 2⋅ μ
a
2 ⎣⎝ a ⎠ a⎦ U a ⎝ ⎠

U a ∂ ∂ 2 ⋅ U⋅ μ
(b) For τyx = 0 at y = 0 0 = μ⋅ − ⋅ p p =
a 2 ∂x ∂x 2
a

U⋅ y
2
2 ⋅ U⋅ μ ⎡ y ⎞2 y⎤
2
⋅ ⎢⎛⎜ ⎛y⎞
a u
The velocity distribution is then u= + ⋅ − ⎥ = ⎜a
a 2⋅ μ
a
2 ⎣⎝ a ⎠ a⎦ U ⎝ ⎠

The velocity distributions can be plotted in Excel.


y /a (a) u /U (b) u /U
0.0 0.000 0.000
0.1 0.190 0.010
0.2 0.360 0.040
0.3 0.510 0.090
0.4 0.640 0.160
0.5 0.750 0.250
0.6 0.840 0.360
0.7 0.910 0.490
0.8 0.960 0.640
0.9 0.990 0.810
1.0 1.00 1.000

Zero-Stress Velocity Distributions


1.00 Zero Stress Upper Plate

Zero Stress Lower Plate


0.75
y /a

0.50

0.25

0.00
0.00 0.25 0.50 0.75 1.00
u /U
Problem 8.24 [Difficulty: 2]
Problem 8.25 [Difficulty: 2]
Problem 8.26

8.26 A capillary tube is 40 mm long and 2 mm bore. The head required


to produce a flow rate of 10 mm3/s is 40 mm. The fluid density is 10
600 kg/m3. Calculate the dynamic and kinematic viscosity of the oil.

Solution:

Rearrange the Poiseuille’s equation and write expression for  :


h  gD 2
 f ...... (1)
32 Lum

Write expression of A
d2
A
4
  2
2


4
 3.14 mm 2

Write expression for um


Q
um 
A
10 mm3 /s

3.14mm 2
 3.185 mm/s
Substitute corresponding values in equation (1)

 0.04  600  9.81 0.002 


2


 32  0.03  3.185 103 
 0.3080 Ns/m

Hence, the dynamic viscosity is 0.3080 Ns/m .

Also, the Kinematic viscosity is determined by the following below expression:



v

Substitute corresponding values in the above expression:
0.3080
v
600
 51.33 105 m 2 /s

Hence, the Kinematic Viscosity is 51.33 105 m2 /s .


Problem 8.27 [Difficulty: 2]

Given: Velocity distribution on incline

Find: Expression for shear stress; Maximum shear; volume flow rate/mm width; Reynolds number

Solution:
ρ⋅ g ⋅ sin( θ) ⎛ y
2⎞
From Example 5.9 u(y) = ⋅ ⎜ h⋅ y −
μ ⎝ 2 ⎠
du
For the shear stress τ = μ⋅ = ρ⋅ g ⋅ sin( θ) ⋅ ( h − y )
dy

τ is a maximum at y = 0 τmax = ρ⋅ g ⋅ sin( θ) ⋅ h = SG ⋅ ρH2O⋅ g ⋅ sin( θ) ⋅ h

2
kg m N⋅ s
τmax = 1.2 × 1000 × 9.81⋅ × sin( 15⋅ deg) × 0.007 ⋅ m × τmax = 21.3 Pa
3 2 kg⋅ m
m s
This stress is in the x direction on the wall
h

The flow rate is ⌠ ⌠
h ⎮ ρ⋅ g ⋅ sin( θ) ⎛ y
2⎞
ρ⋅ g ⋅ sin( θ) ⋅ w⋅ h
3
Q = ⎮ u dA = w⋅ ⎮ u ( y ) dy = w⋅ ⎮ ⋅ ⎜ h⋅ y − dy Q=
⌡ ⌡
0 ⎮

μ ⎝ 2 ⎠ 3⋅ μ
0

3 3
m mm
2 2
Q 1 kg m 3 m N⋅ s −4 s Q s
= × 1.2 × 1000 × 9.81⋅ × sin( 15⋅ deg) × ( 0.007 ⋅ m) × ⋅ = 2.18 × 10 = 217
w 3 3 2 1.60⋅ N⋅ s kg⋅ m m w mm
m s
3
mm
Q Q s 1 mm
The average velocity is V= = V = 217 ⋅ × V = 31.0⋅
A w⋅ h mm 7 ⋅ mm s

ρ⋅ V⋅ h
The gap Reynolds number is Re =
μ
2 2
kg mm m ⎛ 1⋅ m ⎞
Re = 1.2 × 1000 × 31⋅ × 7 ⋅ mm × × ⎜ 1000⋅ mm Re = 0.163
m
3 s 1.60⋅ N⋅ s ⎝ ⎠
The flow is definitely laminar
Problem 8.28 [Difficulty: 3]

8.28 Consider fully developed flow between parallel plates with


the upper plate moving at U = 1.5 m/s; the spacing between the
plates is a = 2.5 mm. Determine the flow rate per unit depth for
the case of zero pressure gradient. If the fluid is air, evaluate the
shear stress on the lower plate and plot the shear stress
distribution across the channel for the zero pressure gradient
case. Will the flow rate increase or decrease if the pressure
gradient is adverse? Determine the pressure gradient that will
give zero shear stress at y = 0.25a. Plot the shear stress
distribution across the channel for the latter case.

Given: Flow between parallel plates.

Find: Shear stress on lower plate; Plot shear stress; Flow rate for pressure gradient; Pressure gradient for zero shear; Plot
the shear stress distribution across the channel for the latter case.

Solution:
U ⋅ y a 2 dp  y  y
2
From Section 8-2 u(y) = + ⋅ ⋅   − 
a 2 ⋅ µ dx  a  a 

m3
y Q a a y U⋅a 1 m 2.5
For dp/dx = 0 u = U.
a 1
= ∫
0
u(y)dy = w ⋅ ∫0
U ⋅ dy =
a 2
Q=
2
× 1.5 ×
s 1000
m Q = 1.88 ×10−3
m
s

du µ ⋅ U N ⋅s
For the shear stress τ = µ⋅ = when dp/dx = 0 µ = 1.81 × 10 −5 (Table A.10)
dy a m2
The shear stress is constant - no need to plot!
2
Ns m 1000  1 m 
τ = 1.81× 10−5 2
×1.5 × ×  τ = 1.1 × 10 −8 N/mm 2
m s 2.5 mm  1000 mm 

Q will decrease if dp/dx > 0; it will increase if dp/dx < 0.

du µ ⋅ U dp  y 1 
For non-zero dp/dx: τ = µ⋅ = + a⋅ ⋅ − 
dy a dx  a 2 

U dp  1 1  U a dp
At y = 0.25a, we get τ(y = 0.25 ⋅ a) = µ ⋅ + a ⋅ ⋅ −  = µ ⋅ − ⋅
a dx  4 2  a 4 dx
2
dp 4 ⋅ µ ⋅ U N m  1000  N Pa
Hence this stress is zero when = = 4 × 1.81 × 10 −5 2 × 1.5 ×   = 17.4 2 = 17.4
dx a 2
m s  2.5 m  m m
m
Problem 8.29 [Difficulty: 3]

8.33 Glycerin at 15°C flows between parallel plates with gap


width b = 2.5 mm. The upper plate moves with speed U = 0.6 m
in the positive x direction. The pressure gradient is ∂p/∂x = −1150
kPa/m. Locate the point of maximum velocity and determine its
magnitude (let y = 0 at the bottom plate). Determine the volume
2
of flow (m ) that passes a given cross section (x = constant) in 10
s. Plot the velocity and shear stress distributions.

Given: Flow between parallel plates.

Find: Location and magnitude of maximum velocity; Volume flow in 10 s; Plot velocity and shear stress.

Solution:

U ⋅ y b2 dp  y  y
2
From Section 8.2 u(y) = + ⋅ ⋅   − 
b 2 ⋅ µ dx  b  b 

du U b2 dp  2 ⋅ y 1  U 1 dp
For umax set du/dx = 0 =0= + ⋅ ⋅ − = + ⋅ ⋅ (2 ⋅ y − b)
dy b 2 ⋅ µ dx  b2 a  b 2 ⋅ µ dx

b µ⋅U
Hence u = u max at y= −
2 b ⋅ dp
dx
N ⋅s
From Fig. A.2 at 15°C µ = 4⋅
m2
2.5
m
N ⋅s m 1000 m
y = 1000 + 4 2 × 0.6 × × y = 2.08 mm
2 m s 2.5 m N
1150 × 103
m2

U ⋅ y b 2 dp  y  y
2
Hence u max = + ⋅ ⋅   −  with y = 2.08 mm
b 2 ⋅ µ dx  b  b 

 2.08 
m
m  1000  1  2.5  1150 × 103 Pa  2.08  2.08 
2 2
m2
u max = 0.6 ×   + ×  m  × × − ×    − 
s  2.5 m  2  1000  4 N ⋅ s m 
  2.5  2.5 
 
 1000 
m
u max = 0.625
s

Q b bU⋅y b2 dp  y 
2
y  U⋅b b3 dp
w
= ∫0
u(y)dy = w ⋅ ∫
0


b
+ ⋅ ⋅   −
2 ⋅ µ dx  b 
  dy =
b  

2
− ⋅
12 ⋅ µ dx

3
Q 1 m 2.5 1  2.5  m2  1150000 Pa 
= × 0.6 × m− × m × ×− 
w 2 s 1000 12  1000  4 N⋅s  m

m3
Q
= 0.0011 s
w m
Q m2
Flow = ⋅ ∆t = 0.0011 × 10 s Flow = 0.011 m 2
w s

dp  y  y
2
u y b2 du U b dp   y  
The velocity profile is = + ⋅ ⋅   −  For the shear stress τ = µ⋅ = µ ⋅ + ⋅ ⋅ 2 ⋅   − 1
U b 2 ⋅ µ ⋅ U dx  b  b  dy b 2 dx   b  

The graphs below can be plotted in Excel


Problem 8.30 [Difficulty: 3]

Given: Flow between parallel plates

Find: Pressure gradient for no flow; plot velocity and stress distributions; also plot for u = U at y = a/2

Solution:
U⋅ y a
2 ⎡ y ⎞2
dp y⎤ Q U⋅ a a
3
dp U dp y 1⎞
Basic equations u(y) = + ⋅ −⋅ ⎢⎛⎜ ⎥ (1) = − ⋅ (2) τ = μ⋅ + a⋅ ⋅ ⎛⎜ − (3)
a 2 ⋅ μ dx ⎣⎝ a ⎠ a⎦ l 2 12⋅ μ dx a dx ⎝a 2⎠

m N⋅ s
Available data U = 1.5⋅ a = 5 ⋅ mm From Fig. A.2 for castor oil a t 20oC μ = 1⋅
s 2
m

dp 6 ⋅ μ⋅ U N⋅ s m 1 dp kPa
From Eq 2 for Q = 0 = = 6 × 1⋅ × 1.5⋅ × = 360 ⋅
dx 2 2 s 2 dx m
a m ( 0.005 ⋅ m)

The graphs be low, u sing E qs. 1 and 3, c an be plotted in Excel

0.75
y/a

0.5

0.25

− 0.5 0 0.5 1 1.5

u (m/s)

0.75
y/a

0.5

0.25

−1 − 0.5 0 0.5 1 1.5

Shear Stress (kPa)

The pressure gradient is adverse, to counteract the flow generated by the upper plate motion
U⋅ y a
2 ⎡ y ⎞2
dp y⎤
For u = U at y = a/2 we need to adjust the pressure gradient. From Eq. 1 u(y) = + ⋅ −⋅ ⎢⎛⎜ ⎥
a 2 ⋅ μ dx ⎣⎝ a ⎠ a⎦

a ⎡⎢⎛ a ⎞ 2 a ⎤⎥
U⋅
a dp ⎢⎜ 2
2
2 2⎥ dp 4 ⋅ U⋅ μ N⋅ s m 1
Hence U= + ⋅ ⋅ ⎢⎜ − or =− = −4 × 1 ⋅ × 1.5⋅ ×
a 2 ⋅ μ dx ⎣⎝ a ⎠ a ⎥⎦ dx 2 2 s 2
a m ( 0.005 ⋅ m)

dp kPa
= −240 ⋅
dx m

0.75
y/a

0.5

0.25

0 0.5 1 1.5 2

u (m/s)

0.75
y/a

0.5

0.25

−1 − 0.5 0 0.5 1 1.5

Shear Stress (kPa)

The pressure gradient is positive to provide the "bulge" needed to satisfy the velocity requirement
Problem 8.31 [Difficulty: 3]

8.31 The velocity profile for fully developed flow of carbon


tetrachloride at 15°C between parallel plates (gap a = 1.25 mm),
with the upper plate moving, is given by Eq. 8.8. Assuming a
volume flow rate per unit depth is 3.15 × 10− m /sec/m for zero
4 3

pressure gradient, find U. Evaluate the shear stress on the lower


plate. Would the volume flow rate increase or decrease with a
mild adverse pressure gradient? Calculate the pressure gradient
that will give zero shear stress at y/a = 0.25. Plot the velocity
distribution and the shear stress distribution for this case.

Given: Flow between parallel plates.

Find: Shear stress on lower plate; pressure gradient for zero shear stress at y/a = 0.25; plot velocity and shear stress.

Solution:

U ⋅ y a 2 dp   y  y
2
Q U.a a 3 dp U dp  y 1 
Basic equations u(y) = + ⋅ ⋅    −  (1) = − ⋅ (2) τ = µ⋅ + a ⋅ ⋅  −  (3)
a 2 ⋅ µ dx   a  a  1 2 12 ⋅ µ dx a dx  a 2 

Available area q = 3.15 × 10 −4 m3 / sec /m a = 1.25 mm at 20°C

N ⋅s
From Fig. A.2, Carbon tetrachloride at 20°C µ = 0.001 ⋅
m2
2⋅Q 2⋅q m
From Eq. 2, for zero pressure gradient U= or U= U = 0.504
a ⋅1 a s
m µ⋅U
From Eq. 3, when y = 0, with U = 0 ⋅ 504 τ yx = τ yx = 0.4032 Pa
s a
A mild adverse pressure gradient would reduce the flow rate.

U dp  1 1  dp 4 ⋅ µ ⋅ U dp kPa
For zero shear stress at y/a = 0.25, from Eq. 3 0 = µ⋅ + a⋅ ⋅ −  or = = 1.29
a dx  4 2  dx a2 dx m

1
1

0.75
0.75
y/a
y/a

0.5 0.5

0.25 0.25

−0.15 0 0.15 0.3 0.45 0.6 −0.7 0 0.7 1.4


u (m/s) Shear Stress (Pa)
Note that the location of zero shear is also where u is maximum!
Problem 8.32 [Difficulty: 3]
Problem 8.33 [Difficulty: 5]
Problem 8.34 [Difficulty: 2]

Given: Expression for efficiency

Find: Plot; find flow rate for maximum efficiency; explain curve

Solution:

q η
0.00 0.0% Efficiency of a Viscous Pump
0.05 7.30%
0.10 14.1% 35%
0.15 20.3% 30%
0.20 25.7%
0.25 30.0% 25%
η

0.30 32.7% 20%


0.35 33.2%
0.40 30.0% 15%
0.45 20.8% 10%
0.50 0.0%
5%
0%
0.00 0.10 0.20 0.30 0.40 0.50
q

For the maximum efficiency point we can use Solver (or alternatively differentiate)

q η The efficiency is zero at zero flow rate because there is no output at all
0.333 33.3% The efficiency is zero at maximum flow rate ∆p = 0 so there is no output
The efficiency must therefore peak somewhere between these extremes
Problem 8.35 [Difficulty: 5]

Problem 2.66
Problem 8.36 [Difficulty: 3]

Given: Data on a journal bearing

Find: Time for the bearing to slow to 100 rpm; visocity of new fluid

Solution:
2
The given data is D = 35⋅ mm L = 50⋅ mm δ = 1 ⋅ mm I = 0.125 ⋅ kg⋅ m

N⋅ s
ωi = 500 ⋅ rpm ωf = 100 ⋅ rpm μ = 0.1⋅
2
m

D
The equation of motion for the slowing bearing is I⋅ α = Torque = −τ⋅ A⋅
2

where α is the angular acceleration and τ is the viscous stress, and A = π⋅ D⋅ L is the surface area of the bearing

U μ⋅ D⋅ ω
As in Example 8.2 the stress is given by τ = μ⋅ =
δ 2⋅ δ

where U and ω are the instantaneous linear and angular velocities.


3
dω μ⋅ D⋅ ω D μ⋅ π⋅ D ⋅ L
Hence I⋅ α = I⋅ =− ⋅ π⋅ D⋅ L⋅ =− ⋅ω
dt 2⋅ δ 2 4⋅ δ
3
dω μ⋅ π⋅ D ⋅ L
Separating variables =− ⋅ dt
ω 4 ⋅ δ⋅ I

3
μ⋅ π⋅ D ⋅ L
− ⋅t
4⋅ δ⋅ I
Integrating and using IC ω = ω0 ω( t) = ωi ⋅ e
3
μ⋅ π⋅ D ⋅ L
− ⋅t
4⋅ δ⋅ I
The time to slow down to ω f = 10 rpm is obtained from solving ωf = ωi ⋅ e

4 ⋅ δ⋅ I ⎛ ωf ⎞ 3
so t = − ⋅ ln⎜ Hence t = 1.19 × 10 s t = 19.9⋅ min
3
μ⋅ π⋅ D ⋅ L ⎝ ωi ⎠

For the new fluid, the time to slow down is t = 10⋅ min

4 ⋅ δ⋅ I ⎛ ωf ⎞
Rearranging the equation μ = − ⋅ ln⎜ kg
3 μ = 0.199 It is more viscous as it slows
π ⋅ D ⋅ L⋅ t ⎝ ωi ⎠ m⋅ s down the rotation in a
shorter time
Problem 8.37 [Difficulty: 2]

Given: Navier-Stokes Equations

Find: Derivation of Example 8.3 result

Solution:

The Navier-Stokes equations are (using the coordinates of Example 8.3, so that x is vertical, y is horizontal)
4 3
∂u ∂v ∂w
+ + =0 (5.1c)
∂x ∂y ∂z
1 4 5 3 4 3
⎛ ∂u ∂u ∂u ∂u ⎞ ∂p ⎛ ∂ 2u ∂ 2u ∂ 2u ⎞
ρ ⎜⎜ + u + v + w ⎟⎟ = ρg x − + µ ⎜⎜ 2 + 2 + 2 ⎟⎟ (5.27a)
⎝ ∂t ∂x ∂y ∂z ⎠ ∂x ⎝ ∂x ∂y ∂z ⎠
1 4 5 3 6 4 5 3
⎛ ∂v ∂v ∂v ∂v ⎞ ∂p ⎛∂ v ∂ v ∂ v⎞ 2 2 2
ρ ⎜⎜ + u + v + w ⎟⎟ = ρg y − + µ ⎜⎜ 2 + 2 + 2 ⎟⎟ (5.27b)
⎝ ∂t ∂x ∂y ∂z ⎠ ∂y ⎝ ∂x ∂y ∂z ⎠
1 3 3 3 3 3 3 3 3
⎛ ∂w ∂w ∂w ∂w ⎞ ∂p ⎛∂ w ∂ w ∂ w⎞ 2 2 2
ρ ⎜⎜ +u +v +w ⎟⎟ = ρg z − + µ ⎜⎜ 2 + 2 + 2 ⎟⎟ (5.27c)
⎝ ∂t ∂x ∂y ∂z ⎠ ∂z ⎝ ∂x ∂y ∂z ⎠

The following assumptions have been applied:

(1) Steady flow (given).


(2) Incompressible flow; ρ = constant.
(3) No flow or variation of properties in the z direction; w= 0 and ∂/∂z = 0.
(4) Fully developed flow, so no properties except possibly pressure p vary in the x direction; ∂/∂x = 0.
(5) See analysis below.
(6) No body force in the y direction; gy = 0

Assumption (1) eliminates time variations in any fluid property. Assumption (2) eliminates space variations in density. Assumption
(3) states that there is no z component of velocity and no property variations in the z direction. All terms in the z component of the
Navier–Stokes equation cancel. After assumption (4) is applied, the continuity equation reduces to ∂v/∂y = 0. Assumptions (3) and (4)
also indicate that ∂v/∂z = 0 and ∂v/∂x = 0. Therefore v must be constant. Since v is zero at the solid surface, then v must be zero
everywhere. The fact that v = 0 reduces the Navier–Stokes equations further, as indicated by (5). Hence for the y direction

∂p
=0
∂y

which indicates the pressure is a constant across the layer. However, at the free surface p = patm = constant. Hence we conclude that p
= constant throughout the fluid, and so
∂p
=0
∂x
In the x direction, we obtain

∂ 2u
µ + ρg = 0
∂y 2
Integrating twice

1 c
u=− ρgy 2 + 1 y + c2
2µ µ
To evaluate the constants, c1 and c2, we must apply the boundary conditions. At y = 0, u = 0. Consequently, c2 = 0. At y = a, du/dy =
0 (we assume air friction is negligible). Hence

1
τ (y = δ ) = µ
du c1
=− ρ gδ + =0
dy y =δ
µ µ
which gives
c1 = ρgδ
and finally
ρg 2 ⎡ ⎛ y ⎞ 1 ⎛ y ⎞ ⎤
2
1 ρg
u=− ρgy +
2
y= δ ⎢⎜ ⎟ − ⎜ ⎟ ⎥
2µ µ µ ⎢⎣⎝ δ ⎠ 2 ⎝ δ ⎠ ⎥⎦
Problem 8.38 [Difficulty: 3]

Given: Paint flow (Bingham fluid)

Find: Maximum thickness of paint film before flow occur

Solution:
du
Basic equations: Bingham fluid: τyx = τy + μp ⋅
dy
dτyx
Use the analysis of Example 8.3, where we obtain a force balance between gravity and shear stresses: = −ρ⋅ g
dy
kg
The gi ven data is τy = 40⋅ Pa ρ = 1000⋅
3
m

From the force balance equation, itegrating τyx = −ρ⋅ g ⋅ y + c and we ha ve bo undary condition τyx( y = δ) = 0

Hence τyx = −ρ⋅ g ⋅ ( δ − y ) and this is a maximum at the wall τmax = ρ⋅ g ⋅ δ

Motion occurs when τmax ≥ τy or ρ⋅ g ⋅ δ ≥ τy


τy −3
Hence the maximum thickness is δ = δ = 4.08 × 10 m δ = 4.08 mm
ρ⋅ g
Problem 8.39 [Difficulty: 2]
Problem 8.40 [Difficulty: 2]
Problem 8.41 [Difficulty: 2]

8.41 A hypodermic needle, with inside diameter d = 0.127 mm


and length L = 25 mm is used to inject saline solution with
viscosity five times that of water. The plunger diameter is D = 10
mm; the maximum force that can be exerted by a thumb on the
plunger is F = 33.4 N. Estimate the volume flow rate of saline
that can be produced.

Given: Hyperdermic needle.

Find: Volume flow rate of saline.

Solution:

π ⋅ ∆p ⋅ d 4
Basic equation Q= (Eq. 8.13c; we assume laminar flow and verify this is correct after solving.)
128 ⋅ µ ⋅ L

F 4⋅F
For the system ∆p = p1 − patm = =
A π ⋅ D2

2
4  1 1000 mm 
∆p = × 33.4 N ×  ×  ∆p = 425.3 kPa
π  10 mm m

N ⋅s N ⋅s
At 20°C from Table A.8 µ H2O = 1.01×10 −3 µ = 5 ⋅µ H2O µ = 5.05 × 10 −3
m 2
m2

4
π N  1m  m2 1 1000 mm
Q= × 425.3 × 103 2 ×  0.127 mm ×  × −3
× ×
128 m  1000 mm  5.05 × 10 N ⋅ s 25 mm 1m

m3 mm3 mm3
Q = 2.15 × 10 −8 Q = 21.5 Q = 1290
s s min

2 2
Q Q 4 m3  1   1000 mm 
Check Re: V= = V = × 2.15 ×10−8 ×  × 
A π ⋅ d2 π s  0.127 mm   1 m 
4

m
V = 1.7
s

ρ⋅ V ⋅ d kg
Re = ρ = 103 (assuming saline is close to water)
µ m3

kg m m m2 Re = 42.8
Re = 103 ×1.7 × 0.127 mm × × Flow is laminar
m3 s 1000 mm 5.05 × 10 −3 N ⋅ s
Problem 8.42 [Difficulty: 3]

Given: Data on a tube

Find: "Resistance" of tube; maximum flow rate and pressure difference for which electrical analogy holds for
(a) kerosine and (b) castor oil
Solution:
The given data is L = 250 ⋅ mm D = 7.5⋅ mm

From Fig. A.2 and Table A.2


− 3 N⋅ s kg kg
Kerosene: μ = 1.1 × 10 ⋅ ρ = 0.82 × 990 ⋅ = 812 ⋅
2 3 3
m m m
N⋅ s kg kg
Castor oil: μ = 0.25⋅ ρ = 2.11 × 990 ⋅ = 2090⋅
2 3 3
m m m

For an electrical resistor V = R⋅ I (1)

The governing equation for the flow rate for laminar flow in a tube is Eq. 8.13c
4
π⋅ ∆p⋅ D
Q=
128 ⋅ μ⋅ L

128 ⋅ μ⋅ L
or ∆p = ⋅Q (2)
4
π⋅ D
By analogy, current I is represented by flow rate Q, and voltage V by pressure drop ∆p.
Comparing Eqs. (1) and (2), the "resistance" of the tube is

128 ⋅ μ⋅ L
R=
4
π⋅ D
The "resistance" of a tube is directly proportional to fluid viscosity and pipe length, and strongly dependent on the inverse
of diameter
ρ⋅ V⋅ D
The analogy is only valid for Re < 2300 or < 2300
μ
Q
ρ⋅ ⋅D
π 2
⋅D
4 2300⋅ μ⋅ π⋅ D
Writing this constraint in terms of flow rate < 2300 or Qmax =
μ 4⋅ ρ
The corresponding maximum pressure gradient is then obtained from Eq. (2)

2
128 ⋅ μ⋅ L 32⋅ 2300⋅ μ ⋅ L
∆pmax = ⋅ Qmax =
4 3
π⋅ D ρ⋅ D

Substituting values
3
−5m l
(a) For kerosine Qmax = 1.84 × 10 Qmax = 1.10⋅ ∆pmax = 65.0⋅ Pa
s min

3
−3m l
(b) For castor oil Qmax = 1.62 × 10 Qmax = 97.3⋅ ∆pmax = 1.30⋅ MPa
s min

The analogy fails when Re > 2300 because the flow becomes turbulent, and "resistance" to flow is then no longer linear with flow
rate
Problem 8.43 [Difficulty: 3]
Problem 8.44 [Difficulty: 3]
Problem 8.45 [Difficulty: 2]

Given: Definition of hydraulic resistance

Find: Hydraulic resistance in a diffuser

Solution:

∆p
Basic equation: Rhyd =
Q

∆p 8µ z2 1 8µ z2 1
Rhyd =
Q
= ∫
π 1 r
z 4
dz = ∫
π 1 (ri + αz ) 4
z
dz

8µ z 1
=
π ∫
0 (ri + αz ) 4
d (ri + αz )

8µ 1 8µ −3
=− (ri + αz ) −3 0z = − [(ri + αz ) −3 − ri ]
πα 3 3πα

8µ ⎡ 1 1⎤
Rhyd = − ⎢ − ⎥
3πα ⎣ (ri + αz ) 3 ri 3 ⎦
Problem 8.47

Given: fully developed flow in a pipe, slip boundary condition on the wall

To find: velocity profile and flow rate

Solution:

Similar to the example described in section 8.3, one obtained

r 2 p
u  c2 1
4 x

The constant C2 will be determined by the slip velocity boundary condition at r = R

u
u l  2
r

And one obtains

R 2 p  l 
c2   2  1  3
4 x  R 

Substituting c2 into equation (1), one obtains

1 p 2 2
u
4 x
 R  r  2lR   4
 R 4 p  l
 5
R
Q   u  2 r  dr   1 4 
0 8 x  R

N s p
Substituting R  12 m,   2.24 105 , mean free path l  74nm, and   1.5 106 Pa m
m 2
x
into equation (5)
 12 10 
6 4
m4 Pa   74×10-9 m  
Q   1.5 106   1  4  
8 2.24 105 Pa×s m  -6
 12×10 m  
m3
Q  3.036 1010
s
Problem 8.48 [Difficulty: 3]

Given: Fully developed flow, velocity profile, and expression to calculate the flow rate

Find: Velocity and flow rate

Solution:
⎛ ∂ 2 u ∂ 2 u ⎞ ∂p
For the fully developed flow, the N-S equations can be simplified to µ ⎜⎜ 2 + 2 ⎟⎟ = = constant (1)
⎝ ∂y ∂z ⎠ ∂x
⎛ 1 1 ⎞ ∂p
Substituting the trial solution in equation (1), one obtains − 2µu 0 ⎜ 2 + 2 ⎟ = (2)
⎝a b ⎠ ∂x
a 2b 2 ∂p
Rearrange it and one obtains u0 = − (3)
2 µ (a + b ) ∂x
2 2

2π 1
The flow rate is Q = ∫ u ( y, z )dydz = ab ∫ ∫ ρu ( ρ , φ )dρdφ (4)
0 0

2π 1 1
Substituting u ( ρ , φ ) = u 0 (1 − ρ ) into Eq. (4) and integrating twice:
2
Q = ab ∫ ∫ ρu (1 − ρ
0
2
)dρdφ = πabu0 (5)
0 0 2

1 πa 3b 3 ∂p
Substituting u0 into (5), one obtains Q = πabu0 = − (6)
2 4 µ (a + b ) ∂x
2 2

For a pipe radius R, a = b = R, from equation (6),


1 ⎛ πR 4 ∂p ⎞
Q pipe = ⎜⎜ − ⎟
8 ⎝ µ ∂x ⎟⎠
which is the same as equation (8.13b) in the book.

For a channel with an elliptic cross-section with a = R and b = 1.5R, from equation (6), one has

29 ⎛ πR 4 ∂p ⎞
Q pipe = ⎜− ⎟.
104 ⎜⎝ µ ∂x ⎟⎠
Problem 8.49 [Difficulty: 2]

8.49 A horizontal pipe carries fluid in fully developed turbulent


flow. The static pressure difference measured between two
sections is 35 kPa. The distance between the sections is 10 m,
and the pipe diameter is 150 mm. Calculate the shear stress, τw,
that acts on the walls.

Given: Turbulent pipe flow.

Find: Wall shear stress.

Solution:


∫ ∫
 
Basic equation Fx = FS x + FBx = u ρ d ∀+ u ρV ⋅dA (Eq. 4.18a)
∂t CV CS

Assumptions: 1) Horizontal pipe 2) Steady flow 3) Fully developed flow

With these assumptions the x momentum equation becomes

π ⋅ D2 π ⋅ D2 (p2 − p1 ) ⋅ D ∆p ⋅ D
p1 ⋅ + τ w ⋅ π ⋅ D ⋅ L − p2 ⋅ =0 or τw = =−
4 4 4⋅L 4⋅L

1 N 1m 1
τw = − × 35 × 103 2 × 150 mm × × τ w = −131 Pa
4 m 1000 mm 10 m

Since τw is negative it acts to the left on the fluid, to the right on the pipe wall.
Problem 8.50 [Difficulty: 2]

8.50 One end of a horizontal pipe is attached using glue to a


pressurized tank containing liquid, and the other has a cap
attached. The inside diameter of the pipe is 2.5 cm, and the tank
pressure is 250 kPa (gage). Find the force the glue must
withstand with the cap on, and the force it must withstand when
the cap is off and the liquid is discharging to atmosphere.

Given: Pipe glued to tank.

Find: Force glue must hold when cap is on and off.

Solution:


∫ ∫
 
Basic equation Fx = FS x + FBx = uρd∀+ u ρ V ⋅d A (Eq. 4.18a)
∂t CV CS

First solve when the cap is on. In this static case

π ⋅ D2
Fglue = ⋅ p1 where p1 is the tank pressure
4

Second, solve for when flow is occuring:

Assumptions: 1) Horizontal pipe 2) Steady flow 3) Fully developed flow

With these assumptions, the x momentum equation becomes

π ⋅ D2 π ⋅ D2
p1 . + τ w ⋅ π ⋅ D ⋅ L − p2 ⋅ =0
4 4

Here p1 is again the tank pressure and p2 is the pressure at the pipe exit; the pipe exit pressure is patm = 0 kPa gage. Hence

π ⋅ D2
Fpipe = Fglue = −τ w ⋅ π ⋅ D ⋅ L = ⋅ p1
4

We conclude that in each case the force on the glue is the same! When the cap is on, the glue has to withstand the tank pressure;
when the cap is off, the glue has to hold the pipe in place against the friction of the fluid on the pipe, which is equal in magnitude to
the pressure drop.

2
π  1m  N
Fglue = ×  2.5 cm ×  × 250 × 103 2 Fglue = 123 N
4  100 cm  m
Problem 8.51

Given: Data on pressure drops in flow in a tube

To find: Which pressure drop is laminar flow, which is turbulent

Solution:

Given data:


p1   3.5 kPa m
x

p2   12 kPa m
x
D  40 mm

From section 8-4, a force balance on a section of fluid leads to

R  D 
w    p  p
2 x 4 x

Here for the two cases

D 
w   p1  w  35 Pa
1
4 x 1

D 
w  p2  w  120 Pa
2
4 x 1

Because both flows are at the same nominal flow rate, the higher pressure drop must correspond
to the turbulent flow, because as indicated in section 8-4, turbulent flows experience additional
stresses. Also indicated in section 8-4 is that for both flows the shear stress varies from zero at
the centreline to the maximum computed above at the walls.

The stress distributions are linear in both cases: Maximum at the walls and zero at the centreline.
Problem 8.52 [Difficulty: 2]

8.52 The pressure drop between two taps separated in the


streamwise direction by 9 m in a horizontal, fully developed
channel flow of water is 6.9 kPa. The cross section of the
channel is a 25 mm × 240 mm rectangle. Calculate the average
wall shear stress.

Given: Flow through channel.

Find: Average wall stress.

Solution:


∫ ∫
 
Basic equation Fx = FS x + FBx = uρd∀+ u ρ V ⋅d A (Eq. 4.18a)
∂t CV CS

Assumptions: 1) Horizontal pipe 2) Steady flow 3) Fully developed flow

With these assumptions, the x momentum equation becomes

H
W⋅H L
p1 ⋅ W ⋅ H + τ w ⋅ 2 ⋅ L ⋅ (W + H) − p2 ⋅ W ⋅ H = 0 or τ w = (p 2 − p1 ) ⋅ τ w = −∆p ⋅
2 ⋅ (W + H) ⋅ L  H
2 ⋅ 1 + 
 W

1m
25 mm ×
1 N 1000 mm 1
τw = − × 6.9 × 103 2 × × τ w = −9.33 Pa
2 m 9m 1m
240 mm ×
1000 mm
1+
9m

Since τw < 0, it acts to the left on the fluid, to the right on the channel wall.
Problem 8.53

8.53 An oil with a viscosity of µ = 0.50 N·s/m2 and density ρ = 800 kg/m3
flows in pipe of diameter D = 0.040 m.
(a) What pressure drop, p1 – p2 is needed to produce a flow rate of Q =
3.0 x 10-5 m3/s, if the pipe is horizontal with x1 = 0 and x2 = 20 m?
(b) How steep a hill θ, must the pipe be on if the oil is to flow through the
pipe at the same rate as in part (a), but with p1 – p2?

Solution:

(a)

If the Reynolds number is less than 2100 the flow is laminar and the equations derived in this
section are valid. Since the average velocity is
Q
V
A

Substitute corresponding values in the above expression


3.0 105 m3 /s
V
  0.030  m 2
2

4
 0.133 m/s
The Reynolds number is
VD
Re 

Substitute corresponding values in the above equation

Re 
 800 kg/m3   0.133 m/s  0.040 m 
0.50 N  s/m 2
 8.512  2100

Hence, the flow is laminar and from the equation


 D 4 p
Q and
128ml
l  x2  x1  20 m , the pressure drop is
p  p1  p2
128 lQ

 D4

Substitute corresponding values in the above expression


128  0.50 N  s/m 2   20.0 m   3.0 105 m3 /s 
p 
  0.040 m 
4

 4.78 kPa

Hence, the required pressure drop is 4.78 kPa .


(b)

If the pipe is on a hill of angle  such that


p  p1  p2
0
Expression for Q is
  p   l sin    D 4
Q
128 l
128Q
sin   ....... (1)
 gD 4

Substitute corresponding values in equation (1)

128  0.50 N  s/m 2  3.0 105 m3 /s 


sin   
  800 kg/m3  9.81 m/s 2   0.040 m 
4

 0.03042
  1.743

Hence, the angle  is 1.743 .


Problem 8.54 [Difficulty: 1]

Given: Data from a funnel viscometer filled with pitch.

Find: Viscosity of pitch.

Solution:

V πD 4 ρg ⎛ h ⎞
Basic equation: Q = = ⎜1 + ⎟ (Volume flow rate)
t 128µ ⎝ L ⎠
where Q is the volumetric flow rate, V flow volume, t is the time of flow, D is the diameter of the funnel stem, ρ is the density of the
pitch, µ is the viscosity of the pitch, h is the depth of the pitch in the funnel body, and L is the length of the funnel stem.

Assumption: Viscous effects above the stem are negligible and the stem has a uniform diameter.

The given or available data is: V = 4.7 ×10 −5 m 3 t = 17,708days D = 9.4mm

kg
h = 75mm L = 29mm ρ = 1.1×103
m3

V 4.7 × 10 −5 m 3 m3
Calculate the flow rate: Q= = = 3.702 × 10 −14
t 17708day × 24hour × 3600s s
day hour

Solve the governing equation for viscosity:

πD 4 ρg ⎛h⎞
µ= ⎜1 + ⎟
128Q ⎝ L ⎠

4
⎛ m ⎞ 3 kg m
π × (9.4mm )4 × ⎜ ⎟ ×1.1× 10 3 × 9.81 2
⎝ 1000mm ⎠ m s ⎛ 75mm ⎞ N × s 2
µ= ⎜1 + ⎟×
⎝ 29mm ⎠ kg × m
3
−14 m
128 × 3.702 ×10
s
N ⋅s
µ = 2.41×108
m2
Compare this to the viscosity of water, which is 10-3 N·s/m2!
Relate this equation to 8.13c for flow driven by a pressure gradient:

π∆pD 4 πD 4 ∆p
Q= = × .
128µL 128µ L

For this problem, the pressure (∆p) is replaced by the hydrostatic force of the pitch.
Consider the pressure variation in a static fluid.

dp ∆p ∆p
= − ρg = − ρg = = .
dz ∆z L + h
Replacing the term in 8.13c
πD 4 ∆p πD 4 ρg × (L + h ) πD 4 ⎛ h⎞
Q= × = × = × ρg × ⎜1 + ⎟
128µ L 128µ L 128µ ⎝ L⎠

V πD 4 ρg ⎛ h ⎞
Hence Q= = ⎜1 + ⎟
t 128µ ⎝ L ⎠

which is the same as the given equation.


Problem 8.55 [Difficulty: 3]
Problem 8.56 [Difficulty: 3] Part 1/2
[Difficulty: 3] Part 2/2
Problem 8.57 [Difficulty: 3]

8.57 Consider fully developed laminar flow of water between


stationary parallel plates. The maximum flow speed, plate
spacing, and width are 6.1 m/s, 1.9 mm, 38 mm respectively.
Find the kinetic energy coefficient, α.

Given: Laminar flow between parallel plates.

Find: Kinetic energy coefficient, α.

Solution:

Basic Equation: The kinetic energy coefficient, α is given by

α=
∫ A
ρ V 3 dA
(8.26b)
m V 2

From Section 8-2, for flow between parallel plates

  y 2  3   y  2 
u = umax 1 −    = V 1 −   
  a2   2   a2  
   

3
since umax = V .
2

Substituting

a a

∫ ρV 3 dA ∫ ρ u 3 dA 1 u
3
1
2
u
3 2
2 u
3
α= A
 2
mV
= A
ρVAV 2
=
A ∫   dA = wa
A V  ∫ ∫
 V  wdy = a  V  dy
a  0
 

2

Then

1 3 3 3 1
2 a  u   umax   y   3 
α=  ∫  d   =
a 2  umax   V   a 2   2 
0 0
(1 − η 2 )3 dη ∫
y
where η = a
2

Evaluating,

(1 − η2 )3 = 1 − 3η 2 + 3η4 − η6

The integral is then

3 1 3 1
 3  3  3 1  27 16
α= 
 2 ∫ (1 − 3η 2 + 3η 4 − η6 ) d η =   η − η3 + η5 − η7  =
 2  5 7 0 8 35
= 1.54
0
Problem 8.58 [Difficulty: 3]
Problem 8.59 [Difficulty: 2]

Given: Data on flow through elbow

Find: Head loss

Solution:
⎛⎜ p V1
2 ⎞ ⎛⎜ p V2
2 ⎞ h
1 2 lT
Basic equation ⎜ ρ⋅ g + α⋅ + z 1 − ⎜ ρ⋅ g + α⋅ + z 2 = g = HlT
⎝ 2 ⋅ g ⎠ ⎝ 2 ⋅ g ⎠

Assumptions: 1) Steady flow 2) Incompressible flow 3) α at 1 and 2 is approximately 1

2 2
p1 − p2 V1 − V2
Then HlT = + + z1 − z2
ρ⋅ g 2⋅ g

( )
3 2 2 2
kg⋅ m
+ × 1.75 − 3.5 ⋅ ⎛⎜ ⎞ ×
3 N m s 1 2 2 m s
HlT = ( 70 − 45) × 10 ⋅ × × × + ( 2.25 − 3 ) ⋅ mHlT = 1.33 m
m
2 1000⋅ kg 2
s ⋅N
9.81⋅ m 2 ⎝ s ⎠ 9.81 ⋅m

2
m N⋅ s N⋅ m
In terms of energy/mass h lT = g ⋅ HlT h lT = 9.81⋅ × 1.33⋅ m × h lT = 13.0⋅
2 kg⋅ m kg
s
Problem 8.60 [Difficulty: 2]

Given: Data on flow in a pipe

Find: Head loss for horizontal pipe; inlet pressure for different alignments; slope for gravity feed

Solution:
⎛⎜ p V1
2 ⎞ ⎛⎜ p V2
2 ⎞
1 2
The basic equation between inlet (1) and exit (2) is ⎜ρ + α ⋅
1 2 + g ⋅ z 1 − ⎜ + α ⋅
2 2 + g ⋅ z 2 = h lT (8.29)
⎝ ⎠ ⎝ρ ⎠

m kg N⋅ s
Given or available data D = 75⋅ mm V = 5⋅ ρ = 999 ⋅ μ = 0.001 ⋅
s 3 2
m m

Horizontal pipe data p 1 = 275 ⋅ kPa p 2 = 0 ⋅ kPa (Gage pressures) z1 = z2 V1 = V2

p1 − p2 J
Equation 8.29 becomes h lT = h lT = 275 ⋅
ρ kg

For an inclined pipe with the same flow rate, the head loss will be the same as above; in addition we have the following new data

z1 = 0 ⋅ m z2 = 15⋅ m

Equation 8.29 becomes ( )


p 1 = p 2 + ρ⋅ g ⋅ z2 − z1 + ρ⋅ h lT p 1 = 422 ⋅ kPa

For a declining pipe with the same flow rate, the head loss will be the same as above; in addition we have the following new data

z1 = 0 ⋅ m z2 = −15⋅ m

Equation 8.29 becomes ( )


p 1 = p 2 + ρ⋅ g ⋅ z2 − z1 + ρ⋅ h lT p 1 = 128 ⋅ kPa

For a gravity feed with the same flow rate, the head loss will be the same as above; in addition we have the following new data

p 1 = 0 ⋅ kPa (Gage)

h lT
Equation 8.29 becomes z2 = z1 − z2 = −28.1 m
g
Problem 8.61

Given: Data on flow through above

To find: Inlet velocity

Solution:

Basic Equation:
 P1 V12   P2 V22  h
   Z1      Z 2   1T  H1T
  g 2g    g 2g  g

Assumptions:
(1) Steady flow
(2) Incompressible flow
(3)  at 1 and 2 is approximately 1.

Then,
V22  V12   2V1   V12  3V12
2

2  P1  P2 
  2 g  z1  z2   2 gH1T

2   P1  P2  
V1    g  z1  z2   gH1T 
3  

Substitute corresponding values in the expression of V1


2 3 N m3 kg  m 9.81 m 9.81 m 
V1   60  10 ×  2   3 m   2  2 m 
3 m 1000 kg s  m
2
s 2
s 
 2.70 m/s

Hence, the inlet velocity is 2.70 m/s .


Problem 8.62 [Difficulty: 2]

8.62 Consider the pipe flow from the water tower of Example
8.7. After another 5 years, the pipe roughness has increased
such that the flow is fully turbulent and f = 0.04. Find by how
much the flow rate is decreased.

Given: Increased friction factor for water tower flow.

Find: How much flow is decreased.

Solution:

2 ⋅ g ⋅ (z1 − z 2 )
Basic equation from Example 8.7 V2 =
L 
f ⋅  + 8 + 1
D 

where L = 207.3 m D = 101.6 mm z1 −z2 = 24.4 m

m m3
With f = 0.0308, we obtain V2 = 2.73 and Q = 0.022
s s

m 1
We need to recompute with f = 0.04 V2 = 2 × 9.81 × 24.4 m × V2 = 2.40 m / s
s 2
 
 207.3 
0.04 ×  + 8 +1
 101.6 
 1000 

π ⋅ D2
Hence Q = V2 ⋅ A = V2 ⋅
4

2
m π  101.6 
Q = 2.40 × × m Q = 0.019 m3 /s
s 4  1000 

m3 m3
Hence the flow is decreased by (0.022 − 0.019) = 3 × 10−3
s s
Problem 8.63 [Difficulty: 2]

8.63 Consider the pipe flow from the water tower of Problem 8.62.
To increase delivery, the pipe length is reduced from 183 m to 91 m
(the flow is still fully turbulent and f = 0.04). What is the flow rate?

Given: Increased friction factor for water tower flow, and reduced length.

Find: How much flow is decreased.

Solution:

2 ⋅ g ⋅ (z1 − z 2 )
Basic equation from Example 8.7 V2 =
L 
f ⋅  + 8 + 1
D 

where now we have L = 115.4 m D = 101.6 mm z1 −z2 = 24.4 m

m 1 m
We need to recompute with f = 0.04 V2 = 2 × 9.81 × 24.4 m × V2 = 3.20
s 2
  s
 115.4 
0.04  + 8 +1
 101.6 
 1000 

π ⋅ D2
Hence Q = V2 ⋅ A = V2 ⋅
4

2
m π  101.6 
Q = 3.20 × ×  m Q = 0.026 m3/s
s 4  1000 
Problem 8.64 [Difficulty: 2]
Problem 8.65 [Difficulty: 2]

Given: Data on flow through Alaskan pipeline

Find: Head loss

Solution:
⎛ p V1
2 ⎞ ⎛ p V2
2 ⎞ h
⎜ 1 ⎜ 2 lT
Basic equation + α⋅ + z1 − + α⋅ + z2 = = HlT
⎜ ρ ⋅g 2 ⋅ g ⎜ ρ ⋅g 2 ⋅ g g
⎝ oil ⎠ ⎝ oil ⎠
Assumptions: 1) Steady flow 2) Incompressible flow 3) α at 1 and 2 is approximately 1 4) SG = 0.9 (Table A.2)

p1 − p2
Then HlT = + z1 − z2
SGoil⋅ ρH2O⋅ g

3 2
3 N 1 m kg⋅ m s
HlT = ( 8250 − 350 ) × 10 ⋅ × × × × + ( 45 − 115 ) ⋅ m HlT = 825 m
2 0.9 1000⋅ kg 2 9.81⋅ m
m s ⋅N
2
m N⋅ s kN⋅ m
In terms of energy/mass h lT = g ⋅ HlT h lT = 9.81⋅ × 825 ⋅ m × h lT = 8.09⋅
2 kg⋅ m kg
s
Problem 8.66

Solution:

Computation Equation:
 1 V12   2 V22 
  1  gz 
1    2  gz2   hT (8.29)
P 2   P 2 

Assumptions:
(1) Incomplete flow , so, V1  V2
(2) Fully developed so, 1   2
(3) SG crude oil  0.90  table A.2
Then,
2  1   g  z1  z2    h
T

Substitute corresponding values in the above expressions


2   9.5 106 N/m 2    0.9  999 kg/m3  9.81 m/s 2   50 m  120 m  Ns 2 /kg  m
  0.9   999 kg/m3  7.2 103 N  M/kg 
 2.41 MPa

Hence, the outlet pressure is 2.41 MPa .


Problem 8.67 [Difficulty: 2]

8.67 Laufer [5] measured the following data for mean velocity near the wall in fully
developed turbulent pipe flow at ReU = 50,000 (U = 3 m/s and R = 123 mm) in air:
u /U 0.343 0.318 0.300 0.264 0.228 0.221 0.179 0.152 0.140
y/R 0.0082 0.0075 0.0071 0.0061 0.0055 0.0051 0.0041 0.0034 0.0030
Plot the data and obtain the best-fit slope, du /dy. Use this to estimate the wall shear stress
from τw = µ du /dy. Compare this value to that obtained using the friction factor f computed
using (a) the Colebrook formula (Eq. 8.37), and (b) and Blasius correlation (Eq. 8.38).


u
Solution: “Best-fit” slope is { from analysis in excel file}
U 0.40

d (u/U ) ∆ (u/U )
≈ = 39.8
d ( y/R) ∆ ( y/R)

du Ud (u /U ) m 1 1000 mm

u/U
= = 39.8 × 3 × × = 971 s −1 0.20
dy Rd ( y /R) s 123 mm m

For standard air, µ = 1.78 × 10 −5 N ⋅ s/m2. So

du N ⋅ s 971 N
τw = µ = 1.78 ×10−5 2 × = 1.73 ×10−2 2
dy m s m 0
0 0.005 0.010
y/R
Friction factor is f =f (Re, C/D). For ReU = 50,000, n = 6.8 from Eq. 8.23. Then from Eq. 8.24,

V 2n 2
= = 0.812 and Re V = 0.812 Re U = 0.812 × 50, 000 = 40, 600
U (n + 1) (2n + 1)

Assuming smooth pipe, f = 0.0219 from Eq. 8.37

Balancing forces on a fluid element:

πD 2 πD 2
Then ( p + ∆p ) − τ w πDL − P =0
4 4

R ∆P D L V 2 f m m
τw = = f ρ = ρV 2 ; V = 0.812 U = 0.812 × 3 = 2.44
2 L 4L D 2 8 s s

Substituting,

0.0219 kg m2 N τw
τw = × 1.23 3 × (2.44) 2 2 = 0.02 2 ← 
8 m s m

The result calculated from the friction factor is 15% higher than that evaluated graphically!
Problem 8.68

8.68 A pipe of length 103 m and diameter 18cm is laid at a slope of 1 in


150. Oil is pumped at the rate of 25 litres per second. The specific
gravity of the oil is 1.1 and viscosity is 0.18 Ns/m2. Determine the head
lost due to friction and also calculate the power required to pump the
oil.

Given: Dimensions of the pipe; specific gravity and viscosity of the oil; discharge rate of
the oil.

Find: Head loss due to friction and power required to pump the oil

Solution:

Diameter of the pipe, D  18 cm = 0.18 m

Length of the pipe, L  1000 m

1
Slope of the pipe, i  1in 150 =
150

Specific gravity if the oil, SG  1.1

kg
Therefore, density of the oil, ρ = 1.1  1000 = 1100
m3

Ns
Viscosity of the oil, μ = 0.18
m2

m3
Discharge, Q  25 litre/s = 0.025
s

Velocity of the flow is given by

Q 0.025 0.025 m
u    0.98
Area π  D 2 π  0.18 2
  s
4 4

Reynolds number, Re,

ρVD 1100  0.98  0.18


Re 
μ

0.18
 1078 V  u  0.98 m/s 
Since, the Reynolds number is less than 2000, the flow is viscous. For viscous flow, the co-
efficient of friction can be given by,

16 16
f    0.01484
Re 1078

Due to friction, head loss is given by,


2
4 f  L u
hf 
D  2g
4  0.01484  1000   0.98
2

 m
0.18  2  9.81
= 16.14 m

Due to slope of pipe 1 in 150, the height through which oil is to be raised by pump is given by,
 Slope  Length of pipe

1
iL  1000  6.66 m
150

Therefore, total head against which pump is to work,

H  h f  i  L  16.14  6.66  22.8 m

Thus, power required to pump the oil is given by,

ρ g Q  H
  6.15 kW
1000

Thus, the head loss due to friction is 16.14 m and power required to pump the oil is given by

6.15 kW
Problem 8.69 [Difficulty: 2]

Given: Data on flow in a pipe

Find: Friction factor; Reynolds number; if flow is laminar or turbulent

Solution:
∆p Pa kg
Given data D = 75⋅ mm = 0.075 ⋅ mrate = 0.075 ⋅
L m s

kg − 4 N⋅ s
From Appendix A ρ = 1000⋅ μ = 4 ⋅ 10 ⋅
3 2
m m

The governing equations between inlet (1) and exit (2) are

⎛⎜ p V1
2 ⎞ ⎛⎜ p V2
2 ⎞
1 2
⎜ ρ + α1⋅ 2 + g ⋅ z1 − ⎜ ρ + α2 ⋅ 2 + g⋅ z2 = h l (8.29)
⎝ ⎠ ⎝ ⎠
2
L V
hl = f ⋅ ⋅ (8.34)
D 2

For a constant area pipe V1 = V2 = V

Hence Eqs. 8.29 and 8.34 become


2⋅ D (p1 − p2) 2 ⋅ D ∆p
f = ⋅ = ⋅
2 ρ 2 L
L⋅ V ρ⋅ V

mrate m
For the velocity V = V = 0.017
π 2 s
ρ⋅ ⋅D
4

2 ⋅ D ∆p
Hence f = ⋅ f = 0.0390
2 L
ρ⋅ V

ρ⋅ V⋅ D
The Reynolds number is Re = Re = 3183
μ

This Reynolds number indicates the flow is turbulent.

(From Eq. 8.37, at this Reynolds number the friction factor for a smooth pipe is f = 0.043; the friction factor computed above thus
indicates that, within experimental error, the flow corresponds to turbulent flow in a smooth pipe)
Problem 8.70 [Difficulty: 2]
Problem 8.71 [Difficulty: 3]

Solution:

Using the add-in function Friction factor from the web site

e/D = 0 0.0001 0.0002 0.0005 0.001 0.002 0.005 0.01 0.02 0.04

Re f
500 0.1280 0.1280 0.1280 0.1280 0.1280 0.1280 0.1280 0.1280 0.1280 0.1280
1.00E+03 0.0640 0.0640 0.0640 0.0640 0.0640 0.0640 0.0640 0.0640 0.0640 0.0640
1.50E+03 0.0427 0.0427 0.0427 0.0427 0.0427 0.0427 0.0427 0.0427 0.0427 0.0427
2.30E+03 0.0473 0.0474 0.0474 0.0477 0.0481 0.0489 0.0512 0.0549 0.0619 0.0747
1.00E+04 0.0309 0.0310 0.0312 0.0316 0.0324 0.0338 0.0376 0.0431 0.0523 0.0672
1.50E+04 0.0278 0.0280 0.0282 0.0287 0.0296 0.0313 0.0356 0.0415 0.0511 0.0664
1.00E+05 0.0180 0.0185 0.0190 0.0203 0.0222 0.0251 0.0313 0.0385 0.0490 0.0649
1.50E+05 0.0166 0.0172 0.0178 0.0194 0.0214 0.0246 0.0310 0.0383 0.0489 0.0648
1.00E+06 0.0116 0.0134 0.0147 0.0172 0.0199 0.0236 0.0305 0.0380 0.0487 0.0647
1.50E+06 0.0109 0.0130 0.0144 0.0170 0.0198 0.0235 0.0304 0.0379 0.0487 0.0647
1.00E+07 0.0081 0.0122 0.0138 0.0168 0.0197 0.0234 0.0304 0.0379 0.0486 0.0647
1.50E+07 0.0076 0.0121 0.0138 0.0167 0.0197 0.0234 0.0304 0.0379 0.0486 0.0647
1.00E+08 0.0059 0.0120 0.0137 0.0167 0.0196 0.0234 0.0304 0.0379 0.0486 0.0647

Friction Factor vs Reynolds Number

1.000

0.100

f
e/D =

0 0.0001
0.010 0.0002 0.0005
0.001 0.002
0.005 0.01
0.02 0.04
0.001 Re
1.0E+02 1.0E+03 1.0E+04 1.0E+05 1.0E+06 1.0E+07 1.0E+08
Problem 8.72 [Difficulty: 2]
Problem 8.73 [Difficulty: 3]

Given: Flow through gradual contraction

Find: Pressure after contraction; compare to sudden contraction

Solution:
⎛⎜ p V1
2 ⎞ ⎛⎜ p V2
2 ⎞ V2
2
1 2
Basic equations ⎜ρ + α ⋅ + g ⋅ z 1 − ⎜ + α ⋅ + g ⋅ z 2 = h lm h lm = K⋅ Q = V⋅ A
⎝ 2 ⎠ ⎝ρ 2 ⎠ 2

Assumptions: 1) Steady flow 2) Incompressible flow 3) α at 1 and 2 is approximately 1 4) Horizontal

3
L m kg
Available data Q = 25⋅ Q = 0.025 D1 = 75⋅ mm D2 = 37.5⋅ mm p 1 = 500⋅ kPa ρ = 999⋅
s s 3
m

2
o ⎛ D2 ⎞ ⎛ 37.5 ⎞ 2
A2
For an included angle of 150 and an area ratio =⎜ =⎜ = 0.25 we find from Table 8.3 K = 0.35
A1
⎝ D1 ⎠ ⎝ 75 ⎠

⎛⎜ p V1 ⎞ ⎛⎜ p 2
2
V2 ⎞
2
V2
2
1 4⋅ Q 4⋅ Q
Hence the energy equation becomes ⎜ ρ + 2 − ⎜ ρ + 2 = K⋅ 2 with V1 = V2 =
⎝ ⎠ ⎝ ⎠ π⋅ D1
2
π⋅ D2
2

2
8 ⋅ ρ⋅ Q ⎡ ( 1 + K) 1 ⎤
p 2 = p 1 − ⋅ ⎡( 1 + K) ⋅ V2 − V1 ⎤ = p 2 −
ρ 2 2
⋅⎢ − ⎥
2 ⎣ ⎦ 2 ⎢ 4 4⎥
π
⎣ D2 D1

2
kg ⎛ m ⎞
3 2
3 N
p 2 = 500 × 10 ⋅ −
8
× 999 ⋅ × ⎜ 0.025 ⋅ × ⎡( 1 + 0.35) ×
1

1 ⎤ × N⋅ s p = 170 ⋅ kPa
2 2 3 ⎝ s ⎠ ⎢ 4 4⎥ kg⋅ m 2
m π m ⎣ ( 0.0375⋅ m) ( 0.075 ⋅ m) ⎦

Repeating the above analysis for an included angle of 180 o (sudden contraction) K = 0.41

2
⎛ m ⎞
3 2
⎤ × N⋅ s p = 155 ⋅ kPa
× ⎡( 1 + 0.41) ×
3 N 8 kg 1 1
p 2 = 500 × 10 ⋅ − × 999 ⋅ × ⎜ 0.025 ⋅ −
2 2 3 ⎝ s ⎠ ⎢ 4 4⎥ kg⋅ m 2
m π m ⎣ ( 0.0375⋅ m) ( 0.075 ⋅ m) ⎦
Problem 8.74 [Difficulty: 3]
Problem 8.75 [Difficulty: 3]

8.75 Water flows through a 50 mm diameter tube that suddenly


contracts to 25 mm diameter. The pressure drop across the
contraction is 3.45 kPa. Determine the volume flow rate.

Given: Flow through sudden contraction.

Find: Volume flow rate.

Solution:

 p1 V2  p V2  V22
Basic equations  + α ⋅ 1 + g ⋅ z1  −  2 + α ⋅ 2 + g ⋅ z 2  = h1m h1m = K ⋅ Q = V⋅A
ρ 2   ρ 2  2

Assumptions: 1) Steady flow 2) Incompressible flow 3) α at 1 and 2 is approximately 1 4) Horizontal

Hence the energy equation becomes

 p1 V12   p 2 V22  V22


ρ + − + = K ⋅
 2   ρ 2  2

A2
From continuity V1 = V2 ⋅ = V2 ⋅ AR
A1

 p1 V22 ⋅ AR 2   p 2 V22  V2
Hence ρ +  − +  = K⋅ 2
 2   ρ 2  2

2 2
2 ⋅ (p1 − p 2 ) D   1
Solving for V2 V2 = AR =  2  =   = 0.25 so from Fig. 8.14 K = 0.4
ρ⋅ (1 − AR 2 + K)  D1   2

 m3 
N 1 kg ⋅ m
Hence V2 = 2 × 3.45 × 103 ×
2 
 3  × × V2 = 2.27 m/s
m  10 kg  (1 − 0.25 + 0.4) N ⋅ s 2
2

π ⋅ D 22
Q = V2 ⋅ A 2 = ⋅ V2
4

π  25  m3 m3 L
Q= × m  × 2.27 m/s Q = 1.114 × 10 −3 Q = 0.067 Q = 67
4  1000  s min min
Problem 8.76

Given: Flow through sudden expansion

To find: Inlet speed; volume flow rate

Solution:

Basic Equation:
 1 V12   2 V22 
   1  gz 
1    2  gz2   h1 m
P 2   P 2 
2
V
h1 m  K  1
2
Q V  A
P   H 2o  g  h

Assumptions:
1) Steady flow
2) Incompressible flow
3)  at 1 and 2 is approximately 1
4) Horizontal

Hence, the energy equation becomes


 P1 V12   P2 V22  V12
  
     K 
 2   2  2

From Continuity,
A
V2  V1 1  V1. AR
A2

Hence,
 P1 V12   P2 V22  AR 2  V12
  
     K 
 2   2  2
Solving for V1
2  P2  P1 
V1 
 1  AR 2  K 
2
 D   80 
2

AR   1      0.32
 D2   250 

So, from figure 8.15


Loss coefficient K is 0.5
K  0.5

Also,
P2  P1   H 2O  g  h
10 N  s2
 1000 kg/m3  9.81 m/s 2  m
1000 kg  m
 98.1 Pa

Hence, substitute values in expression of 'V1 '


N m3 1 kg  m
V1  2  98.1 2   
m 1.23 kg 1  0.32  0.5  N  s
2

V1  20.3 m/s

Hence, the input speed is 20.3 m/s .

The volume flow rate is given by:


D2
Q  V1 A1   1 V1
4
  80
2

  m   20.03 m/s 
4  1000 
 0.10068 m3 /s
 6.04 m3 /min

Hence, the volume flow rate is 6.04 m3 /min .


Problem 8.77 [Difficulty: 4]

Given: Data on a pipe sudden contraction

Find: Theoretical calibration constant; plot

Solution:
Given data D1 = 45⋅ mm D2 = 22.5⋅ mm

The governing equations between inlet (1) and exit (2) are

⎛⎜ p V1
2 ⎞ ⎛⎜ p V2
2 ⎞
1 2
⎜ ρ + α1⋅ 2 + g ⋅ z1 − ⎜ ρ + α2 ⋅ 2 + g⋅ z2 = h l (8.29)
⎝ ⎠ ⎝ ⎠
2
V2
where h l = K⋅ (8.40a)
2

Hence the pressure drop is (assuming α = 1)


⎛⎜ V 2 V 2 V2 ⎞
2
2 1
∆p = p 1 − p 2 = ρ⋅ ⎜ − + K⋅
⎝ 2 2 2 ⎠

2
For the sudden contraction
π 2 π 2
V1 ⋅ ⋅ D1 = V2 ⋅ ⋅ D2 = Q ⎛ D1 ⎞
4 4 or V2 = V1 ⋅ ⎜
⎝ D2 ⎠
ρ⋅ V1 ⎡⎛ D ⎞ 4
2 ⎤
⎢ 1 ⎥
so ∆p = ⋅ ⎜ ( 1 + K) − 1
2 ⎢ D2 ⎥
⎣⎝ ⎠ ⎦
For the pressure drop we can use the manometer equation

∆p = ρ⋅ g ⋅ ∆h

ρ⋅ V1 ⎡⎛ D ⎞ 4
2 ⎤
⎢ 1 ⎥
Hence ρ⋅ g ⋅ ∆h = ⋅ ⎜ ( 1 + K) − 1
2 ⎢ D2 ⎥
⎣⎝ ⎠ ⎦

⎡⎛ D 4
2 ⎤
ρ ⎢ 1⎞
Q ⎥
In terms of flow rate Q ρ⋅ g ⋅ ∆h = ⋅ ⋅ ⎜ ( 1 + K) − 1
2 2 ⎢ D2 ⎥
⎛ ⋅D ⎞
π 2 ⎣⎝ ⎠ ⎦
⎜4 1
⎝ ⎠
⎡⎛ D ⎞ 4
2 ⎤
8⋅ Q ⎢ 1 ⎥
or g ⋅ ∆h = ⋅ ⎜ ( 1 + K) − 1
2 4 ⎢ D2 ⎥
π ⋅ D1 ⎣⎝ ⎠ ⎦

Hence for flow rate Q we find Q = k ⋅ ∆h

2 4
g ⋅ π ⋅ D1
where k=
⎡⎛ D ⎞ 4 ⎤
⎢ 1 ⎥
8⋅ ⎜
⎢ D ( 1 + K) − 1

⎣⎝ 2 ⎠ ⎦
2
⎛ D2 ⎞
For K, we need the aspect ratio AR AR = ⎜ AR = 0.25
⎝ D1 ⎠

From Fig. 8.15 K = 0.4 5


2 4 2
g ⋅ π ⋅ D1 −3 m
Using this in the expression for k, with the other given values k = k = 1.52 × 10 ⋅
⎡⎛ D ⎞ 4 ⎤ s
⎢ 1 ⎥
8⋅ ⎜
⎢ D ( 1 + K) − 1

⎣⎝ 2 ⎠ ⎦
L
min
For ∆h in mm and Q in L/min k = 2.89⋅
1
2
mm

The plot of theoretical Q versus flow rate ∆h can be done in Excel.

Calibration Curve for a


Sudden Contraction Flow Meter
60

50
Q (L/mm)

40

30

20

10

0
0 50 100 150 200 250 300 350
Dh (mm)

It is a practical device, but is a) Nonlinear and b) has a large energy loss


Problem 8.78 [Difficulty: 3]

Given: Flow through short pipe

Find: Volume flow rate; How to improve flow rate

Solution:
⎛⎜ p V1
2 ⎞ ⎛⎜ p V2
2 ⎞ 2 2
1 2 L V2 V2
Basic equations ⎜ ρ + α⋅ 2 + g⋅ z1 − ⎜ ρ + α⋅ 2 + g⋅ z2 = h lT h lT = h l + h lm = f ⋅ ⋅ + K⋅ Q = V⋅ A
⎝ ⎠ ⎝ ⎠ D 2 2

Assumptions: 1) Steady flow 2) Incompressible flow 3) α at 1 and 2 is approximately 1 4) L << so ignore hl 5) Reentrant

Hence between the free surface (Point 1) and the exit (2) the energy equation becomes
2 2 2
V1 V2 V2
+ g ⋅ z1 − = K⋅
2 2 2

A2
From continuity V1 = V2 ⋅
A1

2 2 2 2
V2 ⎛ A2 ⎞ V2 V2
Hence ⋅⎜ + g⋅ h − = K⋅
2
⎝ A1 ⎠ 2 2

2⋅ g⋅ h
Solving for V 2 V2 = and from Table 8.2 K = 0.78
⎡ ⎛ A2 ⎞ ⎤⎥
2

⎢1 + K − ⎜ A ⎥
⎣ ⎝ 1⎠ ⎦

m 1 m
Hence V2 = 2 × 9.81⋅ × 1⋅ m × V2 = 3.33
2 ⎡ 2⎤ s
s
⎢1 + 0.78 − ⎛⎜ 350 ⎞ ⎥
⎣ ⎝ 3500 ⎠ ⎦
2 3 3
m 2 ⎛ 1⋅ m ⎞ −3m m
Q = V2 ⋅ A2 Q = 3.33⋅ × 350 ⋅ mm × ⎜ 1000⋅ mm Q = 1.17 × 10 Q = 0.070 ⋅
s ⎝ ⎠ s min

The flow rate could be increased by (1) rounding the entrance and/or (2) adding a diffuser (both somewhat expensive)
Problem 8.79 [Difficulty: 3]
Problem 8.80

Solution:

Apply the energy equation for steady, incompressible pipe flow.

Basic Equation:
P1 V120 P V2
 1  gz1  2   2 2  gz2  hl Th ,
 2  2
V22
hl T  h20  hl1m ; hlm  K ent  ; P   H 2O g h
2

Assumptions:
1) V1  0
2) Neglect elevation changes
3) Uniforms flow at exit
4) Neglect frictional losses
Then,
P P1  P2 V22 V2
   K ent  2
  2 2
V22
 1  K ent 
2
 H2O g h


Or
2  P1  P2 
V2 
 1  K ent 
2  H 2O g h
 (1)
 1  K ent 

From table 8.2


For square edged; Kent  0.5 ,
For rounded entrance; Kent  0.4
Calculate V2 for Kent  0.5 , from equation (1)
2 kg m m3
V2   999 3  9.81 2  0.0035 m 
1.50 m s 1.23 kg
V2  6.09 m/s

Calculate V2 for Kent  0.04 , from equation (1)


2 kg m m3
V2  Modified    999 3  9.81 2  0.0035 m 
1.04 m s 1.23 kg
V2  Modified   7.32 m/s

Since,
Q V  A,
Then

Q  V2 m  V2  (2)

Substitute corresponding values in equation (2)



Q   7.32  6.09  m/s   0.180 
2
m2
4
 0.0312 m /s3

The percentage improvement is


Q
% 100
Q
V2 m  V2
 100 (3)
V2

Substitute corresponding values in equation (3)


 7.32  6.09 
%  100
 6.09 
 20.19%

Hence, the increase in volume flow rate that results from the change in entrance contour is
20.19% .
Problem 8.81 [Difficulty: 3]

Given: Flow out of water tank

Find: Volume flow rate using hole; Using short pipe section; Using rounded edge

Solution:
⎛⎜ p V1
2 ⎞ ⎛⎜ p V2
2 ⎞ 2 2
1 2 L V2 V2
Basic equations ⎜ ρ + α⋅ 2 + g⋅ z1 − ⎜ ρ + α⋅ 2 + g⋅ z2 = h lT h lT = hl + h lm = f ⋅ D ⋅ 2 + K⋅ 2 Q = V⋅ A
⎝ ⎠ ⎝ ⎠

Assumptions: 1) Steady flow 2) Incompressible flow 3) α at 1 and 2 is approximately 1 4) Vl << 5) L << so hl = 0

Available data D = 25⋅ mm r = 5 ⋅ mm h = 5⋅ m

Hence for all three cases, between the free surface (Point 1) and the exit (2) the energy equation becomes

2 2
V2 V2 2⋅ g⋅ h
g ⋅ z1 − = K⋅ and solving for V 2 V2 =
2 2 ( 1 + K)

From Table 8.2 Khole = 0.5 for a hole (assumed to be square-edged) Kpipe = 0.78 for a short pipe (rentrant)

r 5 ⋅ mm
Also, for a rounded edge = = 0.2 > 0.15 so from Table 8.2 Kround = 0.04
D 25⋅ mm

m 1 m
Hence for the hole V2 = 2 × 9.81⋅ × 5⋅ m × V2 = 8.09
2 ( 1 + 0.5) s
s
3
m π 2 −3m L
Q = V2 ⋅ A2 Q = 8.09⋅ × × ( 0.025 ⋅ m) Q = 3.97 × 10 Q = 3.97
s 4 s s

m 1 m
Hence for the pipe V2 = 2 × 9.81⋅ × 5⋅ m × V2 = 7.42
2 ( 1 + 0.78) s
s
3
m π 2 −3m L
Q = V2 ⋅ A2 Q = 7.42⋅ × × ( 0.025 ⋅ m) Q = 3.64 × 10 Q = 3.64
s 4 s s

L
Hence the change in flow rate is 7.42 − 8.09 = −0.67⋅ The pipe leads to a LOWER flow rate
s

m 1 m L
Hence for the rounded V2 = 2 × 9.81⋅ × 5⋅ m × V2 = 9.71 Q = V2 ⋅ A2 Q = 3.64
2 ( 1 + 0.04) s s
s

L
Hence the change in flow rate is 9.91 − 8.09 = 1.82⋅ The rounded edge leads to a HIGHER flow rate
s
Problem 8.82 [Difficulty: 3]

8.82 A conical diffuser of length 150 mm is used to expand a


pipe flow from a diameter of 50 mm to a diameter of 89 mm.
For a water flow rate of 47 Lit/s, estimate the static pressure
rise. What is the approximate value of the loss coefficient?

Given: Data on geometry of conical diffuser; flow rate.

Find: Static pressure rise; loss coefficient.

Solution:

p2 − p1 V12 V2 1
Basic equations Cp = (8.41) h1m = K ⋅ = (C pi − C p ) ⋅ 1 (8.44) C pi = 1 − (8.42)
1
⋅ρ⋅ V`12
2 2 AR 2
2

Given data D1 = 50 mm D 2 = 89 mm N = 150 mm (N= length) Q = 47Lit /s

1
From Eq. 8.41 ∆p = p2 − p1 = ⋅ρ ⋅ V12 ⋅ Cp (1)
2

1
Combining Eqs. 8.44 and 8.42 we obtain an expression for the loss coefficient K K = 1− − Cp (2)
AR 2

The pressure recovery coefficient Cp for use in Eqs. 1 and 2 above is obtained from Fig. 8.15 once compute AR and the
dimensionless length N/R1 (where R1 is the inlet radius).

2 2
D   89 
The aspect ratio AR is AR =  2  AR =   AR = 3.17
 D1   50 

D1 N
R1 = R1 = 25 mm Hence =6
2 R1

From Fig. 8.15, with AR = 3.17 and the dimensionless length N/R1 = 6, we find Cp = 0.6

2
 
Q 4 m  1 
3
To complete the calculations we need V1 V1 = V1 = × 47 Lit s × ×  V1 = 24 m/s
π π 1000 Lit  50 m 
⋅D 2  
4 1  1000 

1
We can now compute the pressure rise and loss coefficient from Eqs. 1 and 2 ∆p = ⋅ ρ⋅ V12 ⋅ C p
2

1 N
∆p = 103 kg m3 × (24 m s ) 2 × 0.6 × ∆p = 172.8 kPa
2 kg ⋅ m

1 1
K = 1− 2
− Cp K = 1− − 0.6 K = 0.3
AR 3.17 2
Problem 8.83 [Difficulty: 4]

Given: Sudden expansion

Find: Expression for minor head loss; compare with Fig. 8.15; plot

Solution:
The governing CV equations (mass, momentum, and energy) are

Assume: 1) Steady flow 2) In compressible flow 3) Uniform flow at each section 4) Ho rizontal: no body force 5) No
shaft work 6) Neglect viscous friction 7) Neglect gravity

The m ass eq uation becomes V1 ⋅ A1 = V2 ⋅ A2 (1)

The momentum equation becomes ( )


p 1 ⋅ A2 − p 2 ⋅ A2 = V1 ⋅ −ρ⋅ V1 ⋅ A1 + V2 ⋅ ρ⋅ V2 ⋅ A2 ( )
A1
or (using Eq. 1) p 1 − p 2 = ρ⋅ V1 ⋅
A2
(
⋅ V2 − V1 ) (2)

⎛ p1 2⎞ ⎛ p2 2⎞
The energy equation becomes Qrate = ⎜ u 1 +
⎝ ρ ⎠
(
+ V1 ⋅ −ρ⋅ V1 ⋅ A1 + ⎜ u 2 +
⎝ ρ
)
+ V2 ⋅ ρ⋅ V2 ⋅ A2

( )
2 2
Qrate V1 − V2 p1 − p2
or (using Eq. 1) h lm = u 2 − u 1 − = + (3)
mrate 2 ρ

2 2
V1 − V2 A1
Combining Eqs. 2 and 3 h lm =
2
+ V1 ⋅
A2
(
⋅ V2 − V1 )

⎡ 2 2⎤
V1
⎢ ⎛ V2 ⎞ ⎥ 2 A1 ⎡⎛ 2 ⎞
V ⎤
h lm = ⋅ 1− ⎜V + V1 ⋅ ⋅ ⎢⎜ − 1⎥
2 ⎢ ⎥ A2
⎣ ⎝ 1⎠ ⎦ ⎣⎝ V1 ⎠ ⎦
A1 V2
From Eq. 1 AR = =
A2 V1
2
Hence h lm =
V1
2
(
⋅ 1 − AR
2 ) + V12⋅AR⋅(AR − 1)
2
h lm =
V1
2
( 2 2
⋅ 1 − AR + 2 ⋅ AR − 2 ⋅ AR )
2 2
V1 2 V1
h lm = K⋅ = ( 1 − AR) ⋅
2 2

2
Finally K = ( 1 − AR)

This result, and the curve of Fig. 8.15, are shown below as computed in Excel. The agreement is excellent.

AR K CV K Fig. 8.15
0.0 1.00 1.00
0.1 0.81
0.2 0.64 0.60
0.3 0.49
0.4 0.36 0.38
0.5 0.25 0.25
0.6 0.16
0.7 0.09 0.10
0.8 0.04
0.9 0.01 0.01
1.0 0.00 0.00

(Data from F ig. 8.15


is "eyeballed")

Loss Coefficient for a


Sudden Expansion
1.0

0.8 Theoretical Curve

K Fig. 8.15
0.5

0.3

0.0
0.00 0.25 0.50 0.75 1.00
Area Ratio AR
Problem 8.84 [Difficulty: 3]
Problem 8.85 [Difficulty: 2]

Given: Sudden expansion

Find: Expression for upstream average velocity

Solution:
⎛⎜ p V1
2 ⎞ ⎛⎜ p V2
2 ⎞
1 2
The basic equation is ⎜ ρ + α1⋅ 2 + g⋅ z1 − ⎜ ρ + α2⋅ 2 + g⋅ z2 = h lT (8.29)
⎝ ⎠ ⎝ ⎠
2
V
h lT = h l + K ⋅
2

Assume: 1) Steady flow 2) Incompressible flow 3) h l = 0 4) α1 = α2 = 1 5) Neglect gravity

A1
The mass equation is V1⋅ A1 = V2⋅ A2 so V2 = V1⋅
A2

V2 = AR⋅ V1 (1)

2 2 2
p1 V1 p1 V1 V1
Equation 8.29 becomes + = + + K⋅
ρ 2 ρ 2 2
2
or (using Eq. 1)
∆p
ρ
=
p2 − p1
ρ
=
V1
2
(
⋅ 1 − AR − K
2 )
2 ⋅ ∆p
Solving for V1 V1 =
(
ρ⋅ 1 − AR − K
2 )
2 ⋅ ∆p
If the flow were frictionless, K = 0, so Vinviscid = < V1
(
ρ⋅ 1 − AR
2 )
Hence the flow rate indicated by a given ∆p would be lower

2
If the flow were frictionless, K = 0, so ∆pinvscid =
V1
2
(
⋅ 1 − AR )
2

2
compared to ∆p =
V1
2
(
⋅ 1 − AR − K
2 )
Hence a given flow rate would generate a larger ∆p for inviscid flow
Problem 8.86 [Difficulty: 2]
Problem 8.87 [Difficulty: 3]

Given: Data on water flow from a tank/tubing system

Find: Minimum tank level for turbulent flow

Solution:
⎛⎜ p V1
2 ⎞ ⎛⎜ p V2
2 ⎞
Basic equations: 1 2
⎜ρ

+ α ⋅
1 2 + g ⋅ z 1 − ⎜
⎠ ⎝ρ
+ α ⋅
2 2 + g ⋅ z 2 = h lT =
⎠ ∑ hl +
∑ h lm (8.29)
major minor

2 2 Le V2
ρ⋅ V⋅ D L V V
Re = hl = f ⋅ ⋅ (8.34) h lm = K⋅ (8.40a) h lm = f ⋅ ⋅ (8.40b)
μ D 2 2 D 2
⎛ e ⎞
64 1 ⎜ D 2.51
f = (8.36) (Laminar) = −2.0⋅ log ⎜ + (8.37) (Turbulent)
Re f ⎝ 3.7 Re⋅ f ⎠
2 2 2
V L V V
The energy equation (Eq. 8.29) becomes g ⋅ d − α⋅ = f⋅ ⋅ + K⋅
2 D 2 2

This can be solved expicitly for height d, o r solved using Solver


Problem 8.88 [Difficulty: 2]
Problem 8.89 [Difficulty: 3]

Given: Data on water flow from a tank/tubing system

Find: Minimum tank level for turbulent flow

Solution:
⎛⎜ p V1
2 ⎞ ⎛⎜ p V2
2 ⎞
Basic equations: 1 2
⎜ ρ + α1⋅ 2 + g⋅ z1 − ⎜ ρ + α2⋅ 2 + g⋅ z2 = h lT =
⎝ ⎠ ⎝ ⎠ ∑ hl +
∑ h lm (8.29)
major minor

2 2 Le V2
ρ⋅ V⋅ D L V V
Re = hl = f ⋅ ⋅ (8.34) h lm = K ⋅ (8.40a) h lm = f ⋅ ⋅ (8.40b)
μ D 2 2 D 2
⎛ e ⎞
1 ⎜ D 2.51
= −2.0⋅ log ⎜ + (8.37) (Turbulent)
f ⎝ 3.7 Re⋅ f ⎠

Assumptions: 1) Steady flow 2) Incompressible flow 3) α at 1 and 2 is approximately 1 4) Velocity at free surface is <<

kg − 3 N⋅ s
The available data is D = 7.5⋅ mm L = 500 ⋅ mm From Table A.8 at 10oC ρ = 1000 μ = 1.3⋅ 10 ⋅
3 2
m m
Re = 10000 Kent = 0.5 (Table 8.2) Kexit = 1

3
ρ⋅ V⋅ D ρ⋅ Q⋅ D π⋅ μ⋅ D⋅ Re −5m l
From Re = Re = or Q = Q = 7.66 × 10 Q = 0.0766⋅
μ π 2 4⋅ ρ s s
⋅D
4
Q m
Hence V = V = 1.73
⎛ π⋅ D 2⎞ s

⎝ 4 ⎠

= −2 ⋅ log⎛⎜ ⎞
1 2.51
Assuming a smooth tube so f = 0.0309
f ⎝ Re⋅ f ⎠

2 2 2
L V V V
The energy equation (Eq. 8.29) becomes g⋅ d = f ⋅ ⋅ + Kent⋅ + Kexit ⋅
D 2 2 2

2
⋅ ⎛⎜ f ⋅ + Kent + Kexit⎞
V L
Solving for d d = d = 545 ⋅ mm
2⋅ g ⎝ D ⎠

FOR r > 0.15D) Kent = 0.04 (Table 8.2) d = 475 ⋅ mm


Problem 8.90 [Difficulty: 3]

Given: Data on tube geometry

Find: Plot of reservoir depth as a function of flow rate

Solution:
⎛⎜ p V1
2 ⎞ ⎛⎜ p V2
2 ⎞
Basic equations: 1 2
⎜ ρ + α1⋅ 2 + g ⋅ z1 − ⎜ ρ + α2 ⋅ 2 + g⋅ z2 = h lT =
⎝ ⎠ ⎝ ⎠ ∑ hl +
∑ h lm (8.29)
major minor

2 2 Le V2
ρ⋅ V⋅ D L V V
Re = hl = f ⋅ ⋅ (8.34) h lm = K⋅ (8.40a) h lm = f ⋅ ⋅ (8.40b)
μ D 2 2 D 2
⎛ e ⎞
64 1 ⎜ D 2.51
f = (8.36) (Laminar) = −2.0⋅ log ⎜ + (8.37) (Turbulent)
Re f ⎝ 3.7 Re⋅ f ⎠

2 2 2
V L V V
The energy equation (Eq. 8.29) becomes g ⋅ d − α⋅ = f⋅ ⋅ + K⋅
2 D 2 2

This can be solved expicitly for height d, o r solved using Solver

2
V L
d= ⋅ ⎛⎜ α + f ⋅ + K⎞
2⋅ g ⎝ D ⎠
In Excel:
Required Reservoir Head versus Flow Rate

75

50

d (m)

25

0
0 2 4 6 8 10 12
Q (L/min)
Problem 8.91 [Difficulty: 3]

8.91 Oil with kinematic viscosity ν = 0.00005 m2/s flows at


3
0.003 m /s in a 25 m long horizontal drawn-tubing pipe of 4 cm
diameter. By what percentage ratio will the energy loss increase
if the same flow rate is maintained while the pipe diameter is
reduced to 1 cm?

Given: Flow of oil in a pipe.

Find: Percentage change in loss if diameter is reduced.

Solution:

 e 
L V2 64 1  2.51 
Basic equations h1 = f ⋅ ⋅ f= Laminar = −2.0 ⋅ log  D +  Turbulent
D 2 Re f  3.7 Re f 
 

2
Available data ν = 0.00005 m s L = 25 m D = 4 cm Q = 0.003 m3/s

2
Q 4⋅Q 4 3  1 
Here V= = V = × 0.003 m s ×   V = 2.39 m/s
A π ⋅ D2 π  0.04 m 

V⋅D  s 
Then Re = Re = 2.39 m s × 0.04 m ×   Re = 1912
ν  0.00005 m 2
 

2
L V2 64 L V 2 64 25 m (2.39 m s ) 2
The flow is LAMINAR h1 = f ⋅ ⋅ h1 = ⋅ ⋅ h1 = × × h1 = 59.75 m s2
D 2 Re D 2 1912 0.04 m 2

When the diameter is reduced to D = 1 cm


2
Q 4⋅Q 4 3  1 
V= = V = × 0.003 m s ×  V = 38.2 m/s
A π ⋅ D2 π  0.01 m 

V⋅D s
Re = Re = 38.2 m s × 0.01 m × Re = 7640
ν 0.00005 m 2

The flow is TURBULENT. For drawn tubing, from Table 8.1 e = 0.046 mm

 e 
1  2.51 
Given = −2.0 ⋅ log  D +  f = 0.0389
f  3.7 Re ⋅ f 
 
2
L V2 25 m (38.2 m s )
h1 = f ⋅ ⋅ h1 = 0.0389 × × h1 = 7.1 × 104 m2
s2
D 2 0.01 m 2

The increase in loss is 7.1× 104 m2 This is a HUGE increase! The main increase is because the diameter
s2
2
= 1188 reduction causes the velocity to increase; the loss goes as V2, and 1/D, so it
59.75 m s2 increases very rapidly.
Problem 8.92 [Difficulty: 4]

Given: Data on water system

Find: Minimum tank height; equivalent pressure

Solution:
⎛⎜ p V1
2 ⎞ ⎛⎜ p V2
2 ⎞
Basic equations: 1 2
⎜ρ

+ α ⋅
1 2 + g ⋅ z 1 − ⎜
⎠ ⎝ρ
+ α ⋅
2 2 + g ⋅ z 2 = h lT =
⎠ ∑ hl +
∑ (8.29)
h lm
major minor

2 2 Le V2
ρ⋅ V⋅ D L V V
Re = hl = f ⋅ ⋅ (8.34) h lm = K⋅ (8.40a) h lm = f ⋅ ⋅ (8.40b)
μ D 2 2 D 2

⎛ e ⎞
1 ⎜ D 2.51
= −2.0⋅ log ⎜ + (8.37) (Turbulent)
f ⎝ 3.7 Re⋅ f ⎠

Available data D = 7.5⋅ mm L = 1⋅ m Re = 100000 and so f = 0.0180 at th is Re

From Section 8.7 Kent = 0.5 Lelbow45 = 16⋅ D Lelbow90 = 30⋅ D LGV = 8 ⋅ D

Lelbow45 = 0.12 m Lelbow90 = 0.225 m LGV = 0.06 m

kg − 3 N⋅ s
From Table A.8 at 10oC ρ = 1000 μ = 1.3⋅ 10 ⋅
3 2
m m
3
π⋅ μ⋅ D⋅ Re −4m l Q m
Then Q = Q = 7.66 × 10 Q = 0.766 V = V = 17.3
4⋅ ρ s s ⎛ π⋅ D2 ⎞ s

⎝ 4 ⎠
2 2
The energy equation becomes V V ⎛ L Lelbow90 Lelbow45 LGV ⎞
d− = ⋅⎜f ⋅
+ 2⋅ f ⋅ + 2⋅ f ⋅ + f⋅
2⋅ g 2⋅ g ⎝ D D D D ⎠

2
V ⎛ L Lelbow90 Lelbow45 LGV ⎞
Hence d = ⋅⎜1 + f ⋅ + 2⋅ f ⋅ + 2⋅ f ⋅ + f⋅ d = 79.6⋅ m
2⋅ g ⎝ D D D D ⎠
Unrealistic!

IF INSTEAD the reservoir was pressurized ∆p = ρ⋅ g ⋅ d ∆p = 781 ⋅ kPa which is feasible


Problem 8.93 [Difficulty: 2]

8.93 Water from a pump flows through a 230 mm diameter


commercial steel pipe for a distance of 6400 m from the pump
discharge to a reservoir open to the atmosphere. The level of the
water in the reservoir is 15 m above the pump discharge, and
the average speed of the water in the pipe is 3 m/s. Calculate the
pressure at the pump discharge.

Given: Flow from pump to reservoir.

Find: Pressure at pump discharge.

Solution:

 p1 V2  p V2  L V12 V2
Basic equations  + α ⋅ 1 + g ⋅ z1  −  2 + α ⋅ 2 + g ⋅ z 2  = h1T h1T = h1 + h1m = f ⋅ ⋅ + K exit ⋅ 1
ρ 2   ρ 2  D 2 2

Assumptions: 1) Steady flow 2) Incompressible flow 3) α at 1 and 2 is approximately 1 4) V2 <<

Hence the energy equation between Point 1 and the free surface (Point 2) becomes

 p1 V 2  L V2 V2
 +  − (g ⋅ z 2 ) = f ⋅ . + K exit ⋅
ρ 2  D 2 2

 V2 L V2 V2 
Solving for p1 p1 = ρ ⋅  g ⋅ z 2 − + f. ⋅ + K exit ⋅ 
 2 D 2 2
 

From Table A.8 (20°C) ρ = 998 kg/m


3
ν = 1.01 × 10−6 m2/s

V⋅D m 230 m s
Re = Re = 3 × × Re = 6.83 × 105 Turbulent
ν s 1000 1.01 × 10−6 m 2

e
For commercial steel pipe e = 0.046 mm (Table 8.1) so = 0.0002
D

 e 
1  2.51 
Flow is turbulent: Given = −2.0 ⋅ log  D +  f = 0.0150
f  3.7 Re ⋅ f 
 

 L V2 
For the exit Kexit = 1.0 so we find p1 = ρ ⋅  g ⋅ z 2 + f ⋅ ⋅ 
 D 2
 

2
 m 1  N ⋅s
p1 = 998 kg m3 × 9.81 2 × 15 m + 0.0150 × 6400 m × × (3 m s ) 2  ×
6
p1 = 2.02 × 10 Pa
 s 2  kg ⋅ m
Problem 8.94 [Difficulty: 3]

Given: Data on reservoir/pipe system

Find: Plot elevation as a function of flow rate; fraction due to minor losses

Solution:
Required Head versus Flow Rate
L = 250 m
D =5 0 mm 200
e/D = 0.003
K ent = 0.5 150
K exit = 1.0
∆z (m)
ν = 1.01E-06 m2/s
100
3
Q (m /s) V (m/s) Re f ∆z (m) h lm /h lT
0.0000 0.000 0.00E+00 0.000 50
0.0005 0.255 1.26E+04 0.0337 0.562 0.882%
0.0010 0.509 2.52E+04 0.0306 2.04 0.972%
0
0.0015 0.764 3.78E+04 0.0293 4.40 1.01%
0.0000 0.0025 0.0050 0.0075 0.0100
0.0020 1.02 5.04E+04 0.0286 7.64 1.04%
Q (m3/s)
0.0025 1.27 6.30E+04 0.0282 11.8 1.05%
0.0030 1.53 7.56E+04 0.0279 16.7 1.07%
0.0035 1.78 8.82E+04 0.0276 22.6 1.07%
0.0040 2.04 1.01E+05 0.0275 29.4 1.08% Minor Loss Percentage versus Flow Rate
0.0045 2.29 1.13E+05 0.0273 37.0 1.09%
1.2%
0.0050 2.55 1.26E+05 0.0272 45.5 1.09%
0.0055 2.80 1.39E+05 0.0271 54.8 1.09%
0.0060 3.06 1.51E+05 0.0270 65.1 1.10% 1.1%
0.0065 3.31 1.64E+05 0.0270 76.2 1.10%
0.0070 3.57 1.76E+05 0.0269 88.2 1.10% h lm /h lT
0.0075 3.82 1.89E+05 0.0269 101 1.10% 1.0%
0.0080 4.07 2.02E+05 0.0268 115 1.11%
0.0085 4.33 2.14E+05 0.0268 129 1.11%
0.0090 4.58 2.27E+05 0.0268 145 1.11% 0.9%
0.0095 4.84 2.40E+05 0.0267 161 1.11%
0.0100 5.09 2.52E+05 0.0267 179 1.11%
0.8%
0.0000 0.0025 0.0050 0.0075 0.0100
Q (m3/s)
Problem 8.95 [Difficulty: 2]

8.95 3 In an air-conditioning installation, a flow rate of


35 m /min of air at 10°C is required. A smooth sheet metal duct
of rectangular section (0.23 m to 0.75 m) is to be used.
Determine the pressure drop (mm of water) for a 30 m
horizontal duct section.

Given: Flow through rectangular duct.

Find: Pressure drop.

Solution:

p V2  p V2 
Basic equations  1 + α ⋅ 1 + g ⋅ z1  −  2 + α ⋅ 2 + g ⋅ z 2  = h1T
ρ 2   ρ 2 
L V2  Le V 2 
h1T = h1 + h1m = f ⋅ ⋅ + ∑ f ⋅ ⋅
D 2 Minor  D 2 

4⋅a ⋅ b
Dh =
2 ⋅ (a + b)

Assumptions: 1) Steady flow 2) Incompressible flow 3) α is approximately 1

3
Available data Q = 35 m min L = 30 m b = 0.75 m a = 0.23 m

µ = 1.76 × 10− N⋅s/m


3 5 2 3
At 10°C, from Table A.10 ρ = 1.25 kg/m ρw = 1000 kg/m

Q 4⋅a ⋅ b
Hence V= V = 3.4 m/s and D h = D h = 0.35 m
a⋅b 2 ⋅ (a + b)

ρ⋅ V ⋅ D h
Re = Re = 8.45 × 104
µ

1  2.51 
For a smooth duct = −2 ⋅ log   so f = 0.017
f  Re ⋅ f
 

L V2
Hence ∆p = f ⋅ ⋅ρ⋅ ∆p = 10.528 Pa
Dh 2

∆p
or, in water h= h = 0.001 m
ρw ⋅ g
Problem 8.96 [Difficulty: 3]

Given: Data on circuit

Find: Plot pressure difference for a range of flow rates

Solution:
⎛⎜ p V1
2 ⎞ ⎛⎜ p V2
2 ⎞
Basic equations: 1 2
⎜ ρ + α1⋅ 2 + g ⋅ z1 − ⎜ ρ + α2 ⋅ 2 + g⋅ z2 = h lT =
⎝ ⎠ ⎝ ⎠ ∑ hl +
∑ h lm (8.29)
major minor

2 2 Le V2
ρ⋅ V⋅ D L V V
Re = hl = f ⋅ ⋅ (8.34) h lm = K⋅ (8.40a) h lm = f ⋅ ⋅ (8.40b)
μ D 2 2 D 2
⎛ e ⎞
64 1 ⎜ D 2.51
f = (8.36) (Laminar) = −2.0⋅ log ⎜ + (8.37) (Turbulent)
Re f ⎝ 3.7 Re⋅ f ⎠
The energy equation (Eq. 8.29) becomes for the circuit ( 1 = pump inlet, 2 = pump outlet)
2 2 2 2
p1 − p2 L V V V V ⎛L Lelbow Lvalve ⎞
= f⋅ ⋅ + 4 ⋅ f ⋅ Lelbow⋅ + f ⋅ Lvalve⋅ or ∆p = ρ⋅ f ⋅ ⋅⎜
+ 4⋅ +
ρ D 2 2 2 2 ⎝D D D ⎠

In Excel:
Required Pressure Head for a Circuit
1200

1000

800
Dp (kPa)

600

400

200

0
0.00 0.01 0.02 0.03 0.04 0.05 0.06 0.07
Q (m3/s)
Problem 8.97 [Difficulty: 3]

c
h

LA

d e

LB

Given: Pipe friction experiment

Find: Required average speed; Estimate feasibility of constant head tank; Pressure drop over 5 m

Solution:
⎛⎜ p V1
2 ⎞ ⎛⎜ p V2
2 ⎞ LA VA
2
LB VB
2
1 2
Basic equations ⎜ ρ + α⋅ 2 + g⋅ z1 − ⎜ ρ + α⋅ 2 + g⋅ z2 = h lT h lT = h A + h B = fA⋅ ⋅ + fB⋅ ⋅
⎝ ⎠ ⎝ ⎠ DA 2 DB 2

Assumptions: 1) Steady flow 2) Incompressible flow 3) α is approximately 1 4) Ignore minor losses

5
We wish to have ReB = 10

VB⋅ DB ReB⋅ ν 2
−6 m
Hence, from ReB = VB = and for water at 20oC ν = 1.01 × 10 ⋅
ν DB s
2
5 −6 m 1 m
VB = 10 × 1.01 × 10 ⋅ × VB = 4.04
s 0.025 ⋅ m s
2 2
⎛ DB ⎞ m ⎛ 2.5 ⎞ m
We will also need VA = VB⋅ ⎜ VA = 4.04⋅ × ⎜ 5 VA = 1.01
⎝ DA ⎠ s ⎝ ⎠ s

VA⋅ DA m s 4
ReA = ReA = 1.01⋅ × 0.05⋅ m × ReA = 5 × 10
ν s −6 2
1.01 × 10 ⋅m

Both tubes have turbulent flow

For PVC pipe (from Googling!) e = 0.0015⋅ mm

⎛ e ⎞

D
= −2.0⋅ log⎜
1 A 2.51 ⎟
For tube A Given + fA = 0.0210
fA ⎜ 3.7 ReA⋅ fA
⎝ ⎠
⎛ e ⎞

D
= −2.0⋅ log⎜
1 B 2.51 ⎟
For tube B Given + fB = 0.0183
fB ⎜ 3.7 ReB⋅ fB
⎝ ⎠
Applying the energy equation between Points 1 and 3

2 2 2
VB LA VA LB VB
(
g ⋅ LA + h − ) 2
= fA⋅ ⋅
DA 2
+ fB⋅ ⋅
DB 2

2
VB ⎛ LB ⎞
⋅ ⎜ 1 + fB⋅ − g⋅ h
2 DB
Solving for LA LA =
⎝ ⎠
⎛ fA VA
2⎞

⎜g − ⋅
DA 2
⎝ ⎠

2
× ⎛⎜ 4.04⋅
m⎞
× ⎛⎜ 1 + 0.0183 × ⎞ − 9.81⋅ m × 0.5⋅ m
1 20
2 ⎝ s ⎠ ⎝ 0.025 ⎠
s
2
LA = LA = 12.8 m
2
× ⎛⎜ 1.01⋅
m 0.0210 1 m⎞
9.81⋅ − ×
s
2 2 0.05⋅ m ⎝ s ⎠

Most ceilings are about 3.5 m or 4 m, so this height is IMPRACTICAL


Applying the energy equation between Points 2 and 3

⎛⎜ p VB ⎞ ⎛⎜ p 3
2
VB ⎞
2 2 2
2 L VB L VB
⎜ ρ + 2 − ⎜ ρ + 2 = fB⋅ D ⋅ 2 or ∆p = ρ⋅ fB⋅ ⋅
⎝ ⎠ ⎝ ⎠ B DB 2

2 2
5⋅ m N⋅ s
× ⎛⎜ 4.04⋅
kg 0.0183 m⎞
∆p = 1000⋅ × × × ∆p = 29.9⋅ kPa
m
3 2 0.025 ⋅ m ⎝ s ⎠ kg⋅ m
Problem 8.98 [Difficulty: 3]

8.98 Consider flow of standard air at 0.6 m3/s. Compare the


pressure drop per unit length of a round duct with that for
rectangular ducts of aspect ratio 1, 2, and 3. Assume that all
2
ducts are smooth, with cross-sectional areas of 0.09 m

Given: Same flow rate in various ducts.

Find: Pressure drops of each compared to round duct.

Solution:

p V2  p V2  4⋅ A
Basic equations  1 + α ⋅ 1 + g ⋅ z1  −  2 + α ⋅ 2 + g ⋅ z 2  = h1 Dh = e=0 (Smooth)
ρ 2   ρ 2  Pw

Assumptions: 1) Steady flow 2) Incompressible flow 3) α is approximately 1 4) Ignore minor losses

The energy equation simplifies to

L V2 ∆p f V2
∆p = p1 − p 2 = ρ⋅ f ⋅ ⋅ or = ρ⋅ ⋅
Dh 2 L Dh 2

Q m3 1 min 1
But we have V= V = 0.6 × × V = 6.67 m/s
A s 60 ⋅ s 0.09 m 2

From Table A.10 ν = 1.50 × 10−5 m2/s ρ = 1.21 kg/m3 at 20°C

V ⋅ Dh s
Hence Re = Re = 6.67 m s × × D h = 4.45 × 105 ⋅ D h (D h in m)
ν 1.50 × 10−5

4⋅ A 4
For a round duct Dh = D = Dh = × 0.09 m 2 Dh = 0.34 m
π π

4⋅ A 4⋅ b⋅ h 2 ⋅ h ⋅ ar b
For a rectangular duct Dh = = = where ar =
Pw 2 ⋅ (b + h) 1 + ar h

b b⋅h A A 2 ⋅ ar
But h= so h2 = = or h= and D h = A
ar ar ar ar 1 + ar

The results are:

1
Round D h = 0.34 m Re = 4.45 × 105 ⋅ × Dh Re = 1.5 × 105
m

 e 
1 D 2.51  ∆p f V2 ∆p
Given = −2.0 ⋅ log  h +  f = 0.0167 = ρ⋅ ⋅ = 1.32 N/m3
f  3.7 Re ⋅ f  L Dh 2 L
 
 
2 ⋅ ar 1
ar = 1 Dh = ⋅ A Dh = 0.3 m Re = 4.45 × 105 ⋅ ⋅ Dh Re = 1.34 × 105
1 + ar m

 e 
1 D 2.51  ∆p f V2 ∆p
Given = −2.0 ⋅ log  h +  f = 0.0171 = ρ⋅ ⋅ = 1.53 N/m3
f  3.7 Re ⋅ f  L Dh 2 L
 
 

1.53 − 1.32
Hence the square duct experiences a percentage increase in pressure drop of = 15.9%
1.32

2 ⋅ ar 1
ar = 2 Dh = ⋅ A Dh = 0.283 m Re = 4.45 × 105 ⋅ ⋅ Dh Re = 1.26 × 105
1 + ar m

 e 
1 D 2.51  ∆p f V2 ∆p
Given = −2.0 ⋅ log  h +  f = 0.0173 = ρ⋅ ⋅ = 1.65 N/m3
f  3.7 Re ⋅ f  L Dh 2 L
 
 

1.65 − 1.32
Hence the 2 × 1 duct experiences a percentage increase in pressure drop of = 25%
1.32

2 ⋅ ar 1
ar = 3 Dh = ⋅ A Dh = 0.26 m Re = 4.45 × 105 ⋅ ⋅ Dh Re = 1.16 × 105
1 + ar m

 e 
1 D 2.51  ∆p f V2 ∆p
Given = −2.0 ⋅ log  h +  f = 0.0176 = ρ⋅ ⋅ = 1.82 N/m3
f  3.7 Re ⋅ f  L Dh 2 L
 
 

1.82 − 1.32
Hence the 3 × 1 duct experiences a percentage increase in pressure drop of = 37.9%
1.32

Note that f varies only about 7%; the large change in ∆p/L is primarily due to the 1/Dh factor.
Problem 8.99 [Difficulty: 4]

Given: Flow down corroded iron pipe

Find: Pipe roughness; Power savings with new pipe

Solution:
⎛⎜ p V1
2 ⎞ ⎛⎜ p V2
2 ⎞ 2
1 2 L V
Basic equations ⎜ρ + α ⋅ + g ⋅ z 1 − ⎜ + α ⋅ + g ⋅ z 2 = hl hl = f ⋅ ⋅
⎝ 2 ⎠ ⎝ρ 2 ⎠ D 2

Assumptions: 1) Steady flow 2) Incompressible flow 3) α is approximately 1 4) No minor losses


3
m kg
Available data D = 50⋅ mm ∆z = 40⋅ m L = ∆z p 1 = 750⋅ kPa p 2 = 250⋅ kPa Q = 0.015⋅ ρ = 999⋅
s 3
m
Hence the energy equation becomes

2
⎛ p1 ⎞ ⎛ p2 ⎞ L V
⎜ + g ⋅ z1 − ⎜ + g ⋅ z2 = f ⋅ ⋅
⎝ρ ⎠ ⎝ρ ⎠ D 2

3
Q 4⋅ Q 4 m 1 m
Here V= = V= × 0.015⋅ × V = 7.64
A 2 π s 2 s
π⋅ D ( 0.05⋅ m)

In this problem we can compute directly f and Re, and hence obtain e/D

⎛ p1 − p2
2⋅ D ⎞
Solving for f f = ⋅⎜ + g ( z1 − z2 )
L⋅ V ⎝ ⎠
2 ρ

⎛ s ⎞ × ⎡⎢( 750 − 250 ) × 103⋅ N × m


2 3
kg⋅ m ⎤
+ 9.81⋅ × 40⋅ m⎥ f = 0.0382
0.05 m
f = 2× × ⎜ 7.64⋅ m ×
40 ⎝ ⎠ ⎢⎣ 2 1000⋅ kg 2 2 ⎥
m s ⋅N s ⎦

2
−6 m V⋅ D m s 5
From Table A.8 (20oF) ν = 1.01 × 10 ⋅ Re = Re = 7.64⋅ × 0.05⋅ m × Re = 3.78 × 10
s ν s −6 2
1.01 × 10 ⋅m

⎛ e ⎞
1 ⎜ D 2.51
Flow is turbulent: = −2.0⋅ log ⎜ +
f ⎝ 3.7 Re⋅ f ⎠
⎛ − 1 ⎞
⎜ 2⋅ f 2.51 e
Solving for e e = 3.7⋅ D⋅ ⎜ 10 − e = 0.507 mm = 0.0101
⎝ Re⋅ f ⎠ D

e
New pipe (Table 8.1) e = 0.15⋅ mm = 0.003
D

⎛ e ⎞
1 ⎜ D 2.51
Given = −2.0⋅ log ⎜ +
f ⎝ 3.7 Re⋅ f ⎠

f = 0.0326

⎡ L V
2⎤
In this problem

(
∆p = p 1 − p 2 = ρ⋅ ⎢g ⋅ z2 − z1 + f ⋅ ⋅
D 2
) ⎥

⎡ 2⎤ 2
Hence
∆pnew = 1000⋅
kg
× ⎢9.81⋅
m
× ( −40⋅ m) +
0.0326
×
40
× ⎛⎜ 7.64⋅
m⎞ ⎥ × N⋅ s ∆pnew = 369 ⋅ kPa
3 ⎢ 2 2 0.05 ⎝ s ⎠ ⎥ kg⋅ m
m ⎣ s ⎦

∆pold = p 1 − p 2 ∆pold = 500 kPa

Compared to ∆pold = 500 ⋅ kPa we find ∆pold − ∆pnew


= 26.3⋅ %
∆pold

As power is ∆pQ and Q is constant, the power reduction is the same as the above percentage!
Problem 8.100 [Difficulty: 3]

8.100 Water, at volume flow rate Q = 21 Lit/s, is delivered by a fire


hose and nozzle assembly. The hose (L = 76 m, D = 75 mm, and
e/D = 0.004) is made up of four 18 m sections joined by couplings.
The entrance is square-edged, the minor loss coefficient for each
coupling is Kc = 0.5, based on mean velocity through the hose. The
nozzle loss coefficient is Kn = 0.02, based on velocity in the exit jet,
of D2 = 25 mm diameter. Estimate the supply pressure required at
this flow rate.

Given: Flow through fire hose and nozzle.

Find: Supply pressure.

Solution:

p V2  p V2  L V2  V2 
Basic equations  1 + α ⋅ 1 + g ⋅ z1  −  2 + α ⋅ 2 + g ⋅ z 2  = h1T
ρ 2   ρ 2 
h1T = h1 + h1m = f ⋅ ⋅ + ∑ K⋅
D 2 Minor  2


Assumptions: 1) Steady flow 2) Incompressible flow 3) α is approximately 1 4) p2 = patm so p2 = 0 gage

Hence the energy equation between Point 1 at the supply and the nozzle exit (Point n); let the velocity in the hose be V

p1 Vn2 L V2 V2 V2
− =f⋅ ⋅ + (K e + 4 ⋅ K c ) ⋅ + Kn ⋅ n
ρ 2 D 2 2 2

2
 D  Q 4⋅Q 4 Lit 1 m3 1
From continuity Vn =   ⋅V and V= = V= × 21 × × V = 4.75 m/s
 D2  A π ⋅ D2 π s 1000 Lit (75 / 1000)2

ρ ⋅ V2  L 
4
 D 
Solving for p1 p1 = ⋅ f ⋅ + K e + 4 ⋅ K c +   ⋅ (1 + K n ) 
2  D  D2  
 

From Table A.8 (20°C) ρ = 998 kg/m3 ν = 1.01 × 10−6 m 2 /s

V⋅D 75 m s 5
Re = Re = 4.75 m/s × × Re = 3.53 × 10 Turbulent
ν 1000 1.01 × 10−6 m 2

e
For the hose = 0.004
D
 e 
1  2.51 
Flow is turbulent: Given = −2.0 ⋅ log  D +  f = 0.0287
f  3.7 Re ⋅ f 
 

1  76  75 
4  N ⋅ s2
p1 = × 998 kg/m3 × (4.75 m/s)2 ×  0.0287 × + 0.5 + 4 × 0.5 +   × (1 + 0.02) ×
2  75 /1000  25   kg ⋅ m

p1 = 1.29 × 106 N/m 2 p1 = 1290 kPa


Problem 8.101 [Difficulty: 4]

Given: Proposal for bench top experiment

Find: Design it; Plot tank depth versus Re

Solution:
⎛⎜ p V1
2 ⎞ ⎛⎜ p V2
2 ⎞
Basic equations: 1 2
⎜ρ

+ α ⋅
1 2 + g ⋅ z 1 − ⎜
⎠ ⎝ρ
+ α ⋅
2 2 + g ⋅ z 2 = h lT =
⎠ ∑ hl +
∑ h lm (8.29)
major minor

2 2 Le V2
ρ⋅ V⋅ D L V V
Re = hl = f ⋅ ⋅ (8.34) h lm = K ⋅ (8.40a) h lm = f ⋅ ⋅ (8.40b)
μ D 2 2 D 2
⎛ e ⎞
64 1 ⎜ D 2.51
f = (8.36) (Laminar) = −2.0⋅ log ⎜ + (8.37) (Turbulent)
Re f ⎝ 3.7 Re⋅ f ⎠
The energy equation (Eq. 8.29) becomes

2 2 2
V L V V
g ⋅ H − α⋅ = f⋅ ⋅ + K⋅
2 D 2 2

This can be solved explicity for reservoir height H

2
V L
H= ⋅ ⎛⎜ α + f ⋅ + K⎞
2⋅ g ⎝ D ⎠
In Excel:
Computed results:

Q (L/min) V (m/s) Re Regime f H (m)


0.200 0.472 1413 Laminar 0.0453 0.199
0.225 0.531 1590 Laminar 0.0403 0.228
0.250 0.589 1767 Laminar 0.0362 0.258
0.275 0.648 1943 Laminar 0.0329 0.289
0.300 0.707 2120 Laminar 0.0302 0.320
0.325 0.766 2297 Laminar 0.0279 0.353
0.350 0.825 2473 Turbulent 0.0462 0.587
0.375 0.884 2650 Turbulent 0.0452 0.660
0.400 0.943 2827 Turbulent 0.0443 0.738
0.425 1.002 3003 Turbulent 0.0435 0.819
0.450 1.061 3180 Turbulent 0.0428 0.904

The flow rates are realistic, and could easily be measured using a tank/timer system
The head required is also realistic for a small-scale laboratory experiment
Around Re = 2300 the flow may oscillate between laminar and turbulent:
Once turbulence is triggered (when H > 0.353 m), the resistanc e to flow increases
requiring H >0.587 m to maintain; hence the flow reverts to la minar, only to trip over
again to turbulent! This behavior will be visible: the exit flow will switch back and
forth between smooth (laminar) and chaotic (turbulent)

Required Reservoir Head


versus Reynolds Number
1.00

0.75

H (m)
0.50

Laminar
0.25 Turbulent

0.00
1000 1500 2000 2500 3000 3500
Re
Problem 8.102 [Difficulty: 3]
Problem 8.103 [Difficulty: 4]

Given: Draining of swimming pool with larger hose

Find: Flow rate and average velocity

Solution:
⎛⎜ p V1
2 ⎞ ⎛⎜ p V2
2 ⎞ 2 2
1 2 L V V
Basic equations ⎜ ρ + α⋅ 2 + g⋅ z1 − ⎜ ρ + α⋅ 2 + g⋅ z2 = h l hl = f ⋅ ⋅ h lm = Kent⋅
⎝ ⎠ ⎝ ⎠ D 2 2

Assumptions: 1) Steady flow 2) Incompressible flow 3) α is approximately 1 4) No minor losses


2
−6 m
Available data D = 25⋅ mm L = 30⋅ m e = 0.2⋅ mm h = 3⋅ m ∆z = 1.5⋅ m Kent = 0.5 ν = 1 ⋅ 10 ⋅
s
2 2 2
L V V V
Hence the energy equation becomes g ⋅ ( ∆z + h ) = f ⋅ ⋅ + Kent⋅ +
D 2 2 2

2 ⋅ g ⋅ ( ∆z + h )
Solving for V V= (1)
L
f⋅ + Kent + 1
D
⎛ e ⎞
V⋅ D 1 ⎜ D 2.51
We also have Re = (2) In addition = −2.0⋅ log ⎜ + (3)
ν f ⎝ 3.7 Re⋅ f ⎠

Equations 1, 2 and 3 form a set of simultaneous equations for V, Re and f, which we can solve iteratively

2 ⋅ g ⋅ ( ∆z + h ) m V⋅ D 4
Make a guess for f f = 0.1 then V = V = 0.852 Re = Re = 2.13 × 10
L s ν
f⋅ + Kent + 1
D
using Eq 3, at this Re f = 0.0382

2 ⋅ g ⋅ ( ∆z + h ) m V⋅ D 4
Then, repeating V = V = 1.37 Re = Re = 3.41 × 10
L s ν
f⋅ + Kent + 1
D
using Eq 3, at this Re f = 0.0371

2 ⋅ g ⋅ ( ∆z + h ) m V⋅ D 4
Then, repeating V = V = 1.38 Re = Re = 3.46 × 10
L s ν
f⋅ + Kent + 1
D

Using Eq 3, at this Re f = 0.0371 which is the same as before, so we have convergence

2 3
π⋅ D −4m l
The flow rate is then Q = V⋅ Q = 6.79 × 10 Q = 0.679
4 s s

Note that we could use Excel's Solver for this problem


Problem 8.104 [Difficulty: 2]
Problem 8.105 [Difficulty: 3]

8.105 The students of Phi Gamma Delta are putting a kiddy pool
on a porch attached to the second story of their house and plan
to fill it with water form a garden hose. The kiddy pool has a
diameter of 1.5 m and is 0.76 m deep. The porch is 5.5 m above
the faucet. The garden hose is very smooth on the inside, has a
length of 15 m and a diameter of 1.6 cm. If the water pressure at
the faucet is 414 kPa, how long will it take to fill the pool?
Neglect minor losses.

Given: Kiddy pool on a porch.

Find: Time to fill pool with a hose.

Solution:

 p1 V2  p V2 
Basic equations:  + α1 1 + gz1  −  2 + α 2 2 + gz2  = hlT
 ρ 2   ρ 2 

L V2 V2 1 e/D 2.51 
hlT = hl + hlm = f +K = −2.0 log  +  Q = VA
D 2 2 f  3.7 Re f 
 

V12 V2
Assumptions: 1) Steady flow 2) Incompressible 3) Neglect minor losses 4) α1 = α2 2
2 2

Given data p1 = 414 kPa gage z1 = 0 m D = 1.6 cm p2 = 0 kPa gage

e
z2 = 6.26 m =0 L = 15 m
D

p1 L V2
The energy equation becomes: − gz2 = f
ρ D 2

p 
2 ⋅ D ⋅  1 − g ⋅ z2 
 ρ  k
Solving for V: V = V = (1)
f ⋅L f

p   414 ×103 Pa m 
2 ⋅ D ⋅  1 − g ⋅ z2  2 × 0.016 m  − 9.81 2 × 6.26 m 
 1000 kg/m 3 
 ρ   s 
k= = k = 0.87 m/s
L 15 m

ρ ⋅V ⋅ D ρ⋅D
We also have Re = or Re = c ⋅ V (2) where c=
µ µ

kg
1000 × 0.016 m
 N ⋅s  m3 s
Assuming water at 20°C  ρ = 1000 kg/m3 , µ = 1.01× 10−3 2  c= = 15841.6
 m  N ⋅ s m
1.01× 10−3 2
m
1  2.51 
In addition: = −2.0 log   (3)
f  Re f 
 

Equations 1, 2 and 3 form a set of simultaneous equations for V , Re and f

k
Make a guess for f f = 0.015 then V = = 7.10 m/s Re = c ⋅ V = 1.1 × 105
f

1  2.51  k
Given = −2.0 log   f = 0.0177 V = = 6.54 m/s Re = c ⋅ V = 1.04 × 105
f  Re f  f
 

1  2.51  k
Given = −2.0 log   f = 0.0179 V = = 6.50 m/s Re = c ⋅ V = 1.03 × 105
f  Re f  f

The flowrate is then:

π m
Q= × (0.016 m) 2 × 6.5 = 0.0013 m3 /s
4 s

π
Volume pool = 0.76 m × × (1.5)2 m 2 = 1.3 m3
4

Volume pool 1.3 m3


time = = = 1000 s = 16.7 min.
Q 0.0013 m3 /s

This problem can also be solved explicitly in the following manner:

p1 L V2
The energy equation becomes: − gz2 = f
ρ D 2

p 
2 D  1 − gz2 
1  ρ 
or: f =
V L

Plugging this into the Colebrook equation:

  
  
1 L 2.51µ  L
=V = −2.0 log  V 
f  p1   ρVD  p  
2D  − gz2    2 D  1 − gz2   
 ρ  
   ρ  

Noting that the V s on the right hand side cancel provides:

 
   p1  
2.51µ L   2 D  − gz2  
V = −2.0 log   ρ 
 ρD  p1   
 2D  − gz2    L

  ρ 
N ⋅s
Assuming water at 20°C (ρ = 1000 kg/m3, µ = 1.01× 10−3 ) and g = 9.81 m/s2 gives the remaining information needed to
m2
perform the calculation finding:

 
 2.51×1.01× 10−3 N ⋅ s 
 m2 15 m 
V = −2.0 log  
3
 1000 kg/m × 0.016 m  414 × 103 Pa m  
2 × 0.016 m  − 9.81 2 × 6.26 m 
 3 
  1000 kg/m s  
  3  
 2 × 0.016 m  414 ×10 Pa − 9.81 m × 6.26 m  
×  3 2  
 1000 kg/m s 
 
 15 m 
V = 6.5 m/s

and:

π m m3
Q= × (0.016)2 m 2 × 6.5 = 0.0013
4 s s

π
Volume pool = 0.76 m × (1.5) 2 m 2 = 1.3 m3
4

Volume pool 1.3m3


time = = = 1000 s = 16.7 min.
Q 0.0013 m3 /s
Problem 8.106 [Difficulty: 3]
Problem 8.107 [Difficulty: 3]
Problem 8.108 [Difficulty: 3]
Problem 8.109 [Difficulty: 3]

Given: Galvanized drainpipe

Find: Maximum downpour it can handle

Solution:
⎛⎜ p V1
2 ⎞ ⎛⎜ p V2
2 ⎞ 2
1 2 L V
Basic equations ⎜ρ + α⋅ + g ⋅ z 1 − ⎜ + α⋅ + g ⋅ z 2 = hl hl = f ⋅ ⋅
⎝ 2 ⎠ ⎝ρ 2 ⎠ D 2

Assumptions: 1) Steady flow 2) Incompressible flow 3) α is approximately 1 4) No minor losses


2
−6 m
Available data D = 50⋅ mm e = 0.15⋅ mm (Table 8.1) h=L From Table A.7 (20oC) ν = 1.01 × 10 ⋅
s
2
L V
The energy equation becomes (
g ⋅ z1 − g ⋅ z2 = g ⋅ z1 − z2 = g ⋅ h = f ⋅ ⋅
D 2
)

2 ⋅ D⋅ g ⋅ h 2 ⋅ D⋅ g k
Solving for V V= = V= (1)
L⋅ f f f

m m
k= 2 ⋅ D⋅ g k = 2 × 0.05⋅ m × 9.81⋅ k = 0.99
2 s
s

V⋅ D D
We also have Re = or Re = c⋅ V (2) where c=
ν ν

s 4 s
c = 0.05⋅ m × c = 4.95 × 10 ⋅
−6 2 m
1.01 × 10 ⋅m

⎛ e ⎞
1 ⎜ D 2.51 (3)
In addition = −2.0⋅ log ⎜ +
f ⎝ 3.7 Re⋅ f ⎠

Equations 1, 2 and 3 form a set of simultaneous equations for V, Re and f

k m 5
Make a guess for f f = 0.01 then V = V = 9.90 Re = c⋅ V Re = 4.9 × 10
f s

⎛ e ⎞
1 ⎜ D 2.51
Given = −2.0⋅ log ⎜ +
f ⎝ 3.7 Re⋅ f ⎠
k m 5
f = 0.0264 V = V = 6.09 Re = c⋅ V Re = 3.01 × 10
f s
⎛ e ⎞
1 ⎜ D 2.51
Given = −2.0⋅ log ⎜ +
f ⎝ 3.7 Re⋅ f ⎠

k m 5
f = 0.0266 V = V = 6.07 Re = c⋅ V Re = 3.00 × 10
f s

⎛ e ⎞
1 ⎜ D 2.51
Given = −2.0⋅ log ⎜ +
f ⎝ 3.7 Re⋅ f ⎠

k m 5
f = 0.0266 V = V = 6.07 Re = c⋅ V Re = 3.00 × 10
f s

2 3
π⋅ D m
The flow rate is then Q = V⋅ Q = 0.0119⋅
4 s
3
m
0.0119⋅
Q s 100 ⋅ cm 60⋅ s cm
The downpour rate is then = × × = 0.143 ⋅ The drain can handle 0.143 cm/min
Aroof 2 1⋅ m 1 ⋅ min min
500 ⋅ m

Note that we could use Excel's Solver for this problem


Problem 8.110 [Difficulty: 3] Part 1/2
[Difficulty: 3] Part 2/2
Problem 8.111 [Difficulty: 3]

Given: Flow in a tube

Find: Effect of tube length on flow rate; Plot

Solution:
⎛⎜ p V1
2 ⎞ ⎛⎜ p V2
2 ⎞
Governing equations: 1 2
⎜ρ

+ α ⋅
1 2 + g ⋅ z 1 − ⎜
⎠ ⎝ρ
+ α ⋅
2 2 + g ⋅ z 2 = h lT =
⎠ ∑ hl +
∑ h lm (8.29)
major minor

2 2 Le V2
ρ⋅ V⋅ D L V V
Re = hl = f ⋅ ⋅ (8.34) h lm = K⋅ (8.40a) h lm = f ⋅ ⋅ (8.40b)
μ D 2 2 D 2
⎛ e ⎞
64 1 ⎜ D 2.51
f = (8.36) (Laminar) = −2.0⋅ log ⎜ + (8.37) (Turbulent)
Re f ⎝ 3.7 Re⋅ f ⎠
The energy equation (Eq. 8.29) becomes for flow in a tube
2
L V
p 1 − p 2 = ∆p = ρ⋅ f ⋅ ⋅
D 2

This cannot be solved explicitly for velocity V, (and hence flow rate Q) because f depends on V; solution for a given L requires
iteration (or use of Solver)
Flow Rate vs Tube Length for Fixed Dp
10.0

Laminar
Q (m3/s) Turbulent
4
x 10 1.0

0.1
0 5 10 15 20 25 30 35
L (km)

The "critical" length of tube is between 15 and 20 km. For this range, the fluid is making a transition between laminar and
turbulent flow, and is quite unstable. In this range the flow oscillates between laminar and turbulent; no consistent solution is
found (i.e., an Re corresponding to turbulent flow needs an f assuming laminar to produce the ∆p required, and vice versa!) More
realistic numbers (e.g., tube length) are obtained for a fluid such as SAE 10W oil (The graph will remain the same except for scale)
Problem 8.112 [Difficulty: 4]

Given: System for fire protection

Find: Height of water tower; Maximum flow rate; Pressure gage reading

Solution:
⎛⎜ p V1
2 ⎞ ⎛⎜ p V2
2 ⎞
Governing equations: 1 2
⎜ ρ + α1⋅ 2 + g ⋅ z1 − ⎜ ρ + α2 ⋅ 2 + g⋅ z2 = h lT =
⎝ ⎠ ⎝ ⎠ ∑ hl +
∑ h lm(8.29)
major minor

2 Le V2
ρ⋅ V⋅ D L V
Re = hl = f ⋅ ⋅ (8.34) h lm = 0.1⋅ h l h lm = f ⋅ ⋅ (8.40b)
μ D 2 D 2

⎛ e ⎞
64 1 ⎜ D 2.51
f = (8.36) (Laminar) = −2.0⋅ log ⎜ + (8.37) (Turbulent)
Re f ⎝ 3.7 Re ⋅ f ⎠
For no flow the energy equation (Eq. 8.29) applied between the water tower free surface (state 1; height H) and pressure gage is

p2 p2
g⋅ H = or H= (1)
ρ ρ⋅ g

The energy equation (Eq. 8.29) becomes, for maximum flow (and α = 1)
2 2
⋅ ⎛⎜ 1 + 1.1⋅ f ⋅
V V L⎞
g⋅ H − = h lT = ( 1 + 0.1) ⋅ h l or g⋅ H = (2)
2 2 ⎝ D⎠

This can be solved for V (and hence Q) by iterating, or by using Solver

The energy equation (Eq. 8.29) becomes, for restricted flow

p2 2 2
⋅ ⎛⎜ 1 + 1.1⋅ ρ⋅ f ⋅
V V L⎞
g⋅ H − + = h lT = ( 1 + 0.1) ⋅ h l p 2 = ρ⋅ g ⋅ H − ρ⋅ (3)
ρ 2 2 ⎝ D⎠

The results in Excel are shown below:


Problem 8.113 [Difficulty: 4]

8.113 A large open water tank has a horizontal cast iron


drainpipe of diameter D = 2.5 cm and length L = 1.5 m attached
at its base. If the depth of water is h = 3.5 m, find the flow rate
3
(m /hr) if the pipe entrance is (a) reentrant, (b) square-edged,
and (c) rounded (r = 3.75 mm).

Given: Tank with drainpipe.

Find: Flow rate for reentrant, square-edged, and rounded entrances.

Solution:

 p1 V2  p V2  L V2 V2
Basic equations  + α ⋅ 1 + g ⋅ z1  −  2 + α ⋅ 2 + g ⋅ z 2  = h1T h1T = f ⋅ ⋅ + K ent ⋅
ρ 2   ρ 2  D 2 2

Assumptions: 1) Steady flow 2) Incompressible flow 3) α is approximately 1

Available data D = 2.5 cm L = 1.5 m e = 0.26 mm (Table 8.1) h = 3.5 m r = 3.75 mm

Hence the energy equation applied between the tank free surface (Point 1) and the pipe exit (Point 2, z = 0) becomes

V22 V2 L V2 V2
g ⋅ z1 − = g ⋅ z1 − =f⋅ ⋅ + K ent ⋅
2 2 D 2 2

2⋅g⋅ h
Solving for V V= (1)
 L
 1 + K ent + f ⋅ 
D

V⋅D D
We also have Re = or Re = c ⋅ V (2) where c=
ν ν

m2 D s
From Table A.8 (20°C) ν = 1.01 ×10−6 ⋅ c= c = 2.48 × 104
s ν m

 e 
1
In addition = −2.0 ⋅ log  D 2.51  (3)
f  + 
 3.7 Re⋅ f 

Equations 1, 2 and 3 form a set of simultaneous equations for V, Re and f

For a reentrant entrance, from Table 8.2 Kent = 0.78

2⋅g⋅ h m
Make a guess for f f = 0.01 then V= V = 5.37
 L s
 1 + K ent + f ⋅ 
D

Re = c ⋅ V Re = 1.33 × 105
 e 
1
Given = −2.0 ⋅ log  D + 2.51  f = 0.0389
f  
 3.7 Re ⋅ f 

2⋅g⋅ h m
V= V = 4.09 Re = c ⋅ V Re = 1.01 × 105
 L s
 1 + K ent + f ⋅ 
D

 e 
1
Given = −2.0 ⋅ log  D + 2.51  f = 0.0391
f  
 3.7 Re ⋅ f 

2⋅g⋅ h m
V= V = 4.08 Re = c ⋅ V Re = 1.01 × 105
 L s
 1 + K ent + f ⋅ 
D

 e 
1
Given = −2.0 ⋅ log  D 2.51  f = 0.0391
f  3.7 + 
Re⋅ f 

2⋅g⋅ h m
V= V = 4.08 Re = c ⋅ V Re = 1.01 × 105
 L s
 1 + K ent +f⋅ 
D

Note that we could use Excel’s Solver for this problem

π ⋅ D2 m3
The flow rate is then Q = V⋅ Q = 2 × 10−3
4 s

For a square-edged entrance, from Table 8.2 Kent = 0.5

2⋅g⋅ h m
Make a guess from f f = 0.01 then V= V = 5.72
 L s
 1 + K ent + f ⋅ 
D

Re = c ⋅ V Re = 1.42 × 105

 e 
1
Given = −2.0 ⋅ log  D 2.51  f = 0.0389
f  + 
 3.7 Re⋅ f 

2⋅g⋅ h m
V= V = 4.23 Re = c ⋅ V Re = 1.05 × 105
 L s
 1 + K ent +f⋅ 
D

 e 
1
Given = −2.0 ⋅ log  D 2.51  f = 0.0390
f  3.7 + 
Re⋅ f 

2⋅g⋅ h m
V= V = 4.23 Re = c ⋅ V Re = 1.05 × 105
 L s
 1 + K ent + f ⋅ 
D
π ⋅ D2 m3
The flow rate is then Q = V⋅ Q = 2.08 × 10 −3
4 s

r
For a rounded entrance, from Table 8.2 = 0.15 Kent = 0.04
D

2⋅g⋅ h
Make a guess for f f = 0.01 then V=
 L
 1 + K ent + f ⋅ 
D

V = 6.47 m/s Re = c ⋅ V Re = 1.6 × 105

 e 
1
Given 
= −2.0 ⋅ log D 2.51  f = 0.0388
f  3.7 + 
Re⋅ f 

2⋅g⋅ h m
V= V = 4.52 Re = c ⋅ V Re = 1.12 × 105
 L s
 1 + K ent + f ⋅ 
D

 e 
1
Given = −2.0 ⋅ log  D 2.51  f = 0.0389
f  3.7 + 
Re⋅ f 

2⋅g⋅ h m
V= V = 4.51 Re = c ⋅ V Re = 1.12 × 105
 L s
 1 + K ent + f ⋅ 
D

 e 
1
Given = −2.0 ⋅ log  D 2.51  f = 0.0389
f  3.7 + 
Re⋅ f 

2⋅g⋅ h m
V= V = 4.51 Re = c ⋅ V Re = 1.12 × 105
 L s
 1 + K ent + f ⋅ 
D

Note that we could use Excel’s Solver for this problem

π ⋅ D2 m3
The flow rate is then Q = V⋅ Q = 2.21 × 10 −3
4 s

m3 m3 m3
In summary: Renentrant: Q = 2 × 10 −3 Square-edged: Q = 2.08 × 10 −3 Rounded: Q = 2.21 × 10 −3
s s s
Problem 8.114 [Difficulty: 4]

8.114 Repeat Problem 8.113, expect now the pipe is vertical, as


shown.

Given: Tank with drainpipe.

Find: Flow rate for reentrant, square-edged, and rounded entrances.

Solution:

 p1 V2  p V2  L V2 V2
Basic equations  + α ⋅ 1 + g ⋅ z1  −  2 + α ⋅ 2 + g ⋅ z 2  = h1T h1T = f ⋅ ⋅ + K ent ⋅
ρ 2   ρ 2  D 2 2

Assumptions: 1) Steady flow 2) Incompressible flow 3) α is approximately 1

Available data D = 2.5 cm L = 1.5 m e = 0.26 mm (Table 8.1) h = 3.5 m r = 3.75 mm

Hence the energy equation applied between the tank free surface (Point 1) and the pipe exit (Point 2, z = 0) becomes

V22 V2 L V2 V2
g ⋅ z1 = = g ⋅ z1 − =f⋅ ⋅ + K ent ⋅
2 2 D 2 2

2⋅g⋅ h
Solving for V V= (1)
 L
 1 + K ent + f ⋅ 
D

where now we have H=h+L H=5m

V⋅D D
We also have Re = or Re = c ⋅ V (2) where c=
ν ν

m2 D s
From Table A.8 (20°C) ν = 1.01 × 10−6 ⋅ c= 2.48 × 104
s ν m

 e 
1
In addition = −2.0 ⋅ log  D 2.51  (3)
f  + 
 3.7 Re⋅ f 

Equations 1, 2 and 3 form a set of simultaneous equations for V, Re and f

For a reentrant entrance, from Table 8.2 Kent = 0.78

2⋅g ⋅H
Make a guess for f f = 0.01 then V= V = 6.42 m/s
 L
1 + K ent + f ⋅ D 
 

Re = c ⋅ V Re = 1.59 × 105
 e 
1
Given = −2.0 ⋅ log  D 2.51  f = 0.0388
f  + 
 3.7 Re⋅ f 

2⋅g ⋅H m
V= V = 4.89 Re = c ⋅ V Re = 1.21 × 105
 L s
 1 + K ent + f ⋅ 
D

 e 
1
Given = −2.0 ⋅ log  D 2.51  f = 0.0389
f  + 
 3.7 Re⋅ f 

2⋅g ⋅H m
V= V = 4.88 Re = c ⋅ V Re = 1.21 × 105
 L s
1 + K ent + f ⋅ D 
 

Note that we could use Excel’s Solver for this problem

π ⋅ D2 m3 m3
The flow rate is then Q = V⋅ Q = 2.4 × 10−3 Q = 8.62
4 s hr

For a square-edged entrance, from Table 8.2 Kent = 0.5

2⋅g ⋅H m
Make a guess from f f = 0.01 then V= V = 6.83
 L s
1 + K ent + f ⋅ D 
 

Re = c ⋅ V Re = 1.69 × 105

 e 
1
Given = −2.0 ⋅ log  D 2.51  f = 0.0388
f  3.7 + 
Re⋅ f 

2⋅g ⋅H m
V= V = 5.06 Re = c ⋅ V Re = 1.25× 105
 L s
1 + K ent +f ⋅ 
 D

 e 
1
Given = −2.0 ⋅ log  D 2.51  f = 0.0388
f  3.7 + 
Re⋅ f 

2⋅g ⋅H m
V= V = 5.06 Re = c ⋅ V Re = 1.25 × 105
 L s
1 + K ent + f ⋅ D 
 

π ⋅ D2 m3 m3
The flow rate is then Q = V⋅ Q = 2.48 × 10 −3 Q = 8.94
4 s hr

r
For a rounded entrance, from Table 8.2 = 0.15 Kent = 0.04
D
2⋅g ⋅H
Make a guess for f f = 0.01 then V=
 L
1 + K ent + f ⋅ D 
 

m
V = 7.73 Re = c ⋅ V Re = 1.91 × 105
s

 e 
1
Given = −2.0 ⋅ log  D 2.51  f = 0.0387
f  3.7 + 
Re⋅ f 

2⋅g ⋅H m
V= V = 5.4 Re = c ⋅ V Re = 1.34 × 105
 L s
1 + K ent + f ⋅ D 
 

 e 
1
Given = −2.0 ⋅ log  D 2.51  f = 0.0389
f  3.7 + 
Re⋅ f 

2⋅g ⋅H m
V= V = 5.39 Re = c ⋅ V Re = 1.34 × 105
 L s
1 + K ent + f ⋅ D 
 

 e 
1
Given = −2.0 ⋅ log  D 2.51  f = 0.0389
f  3.7 + 
Re⋅ f 

2⋅g ⋅H m
V= V = 5.39 Re = c ⋅ V Re = 1.34 × 105
 L s
1 + K ent + f ⋅ D 
 

Note that we could use Excel’s Solver for this problem

π ⋅ D2 m3 m3
The flow rate is then Q = V⋅ Q = 2.65 × 10 −3 Q = 9.52
4 s hr

m3 m3 m3
In summary: Renentrant: Q = 8.62 Square-edged: Q = 8.94 Rounded: Q = 9.52
hr hr s
Problem 8.115 [Difficulty: 4]

8.115 Consider again the Roman water supply discussed in 0

Example 8.10. Assume that the 15 m length of horizontal zo 5 1.5 m D 5 25 mm


constant-diameter pipe required by law has been installed. The
relative roughness of the pipe is 0.01. Estimate the flow rate of
water delivered by the pipe under the inlet conditions of the L 5 15 m
example. What would be the effect of adding the same diffuser 1
to the end of the 15 m pipe?

Solution: Apply the energy equation for steady, incompressible pipe flow.

Computing equation:

p0 V02 p V2  L V
2
+ α0 + gzo = 1 + α1 1 + g z1 + hlT ; hlT =  f + K ent  1
ρ 2 ρ 2  D  2

Assumptions: (1) p0 = p1 = patm (3) α1 ≈ 1

(2) V1 ≈ 0 (4) Kent = 0.04

Then

V12  L V
2
2 gz0
gz0 = + f + K ent  1 or V1 =
2  D  2 L
1+ f +K
D

For e/D = 0.01, f = 0.038 from Eq. 8.37*, so

1/2
 
 
m 1
V1 =  2 × 9.81 2 × 1.5 m × + 0.04  = 1.10 m/s
 s  1  
 1 +  0.038 × 0.15 m ×  
  25 mm  

Checking, assuming T = 20°C,

VD m sec
Re = = 1.10 × 0.025 m × −6 2
= 2.75 × 104 ; from Eq. 8.37*, f ≈ 0.040, so
V sec 1.0 × 10 m

1/2
 m  1  
V1 =  2 × 9.81 2 × 1.5 m × 1 +  0.040 × 15 m ×  + 0.04 = 1.08 m/s
 s  25 mm  

m π
Q = V, A = 1.08 × (0.025) 2 m 2 = 5.30 × 10−4 m3 /s (no diffuser) Q
s 4

1
The diffuser would increase head loss by Kdiffuser = 0.3 (See Example 8.10), but would reduce V2 to V1 . The energy equation
2
would be
V22  L V
2
1 L V
2
gz 0 = +  f + K ent + K diff  1 =  + f + K ent + K d  1 N
2  D  2 4 D  2 R1 5 3.0

1 AR 5 2.0
2
Thus

2gz 0
V1 =
L
0.25 + f + K ent + K diff
D

1/2
 m 1 
V1 =  2 × 9.81 2 × 1.5 m ×  = 1.09 m/s
 s  1  
0.25 +  0.040 × 15 m ×  + 0.04 + 0.3
  25 mm  

and

m π
Q = V1A = 1.09 × (0.025)2 m 2 = 5.35 × 10−4 m3 /s (with diffuser)
s 4

{The diffuser increases flow rate only slightly (~1 percent), because loss is dominated by fL/D.} Q

*Values of f obtained using Excel’s Solver (or Goal Seek).


Problem 8.116 [Difficulty: 4] Part 1/2

8.116 You are watering your lawn with an old hose. Because 1
main
lime deposits have built up over the years, the 19 mm-i.d. hose 2 3
now has an average roughness height of 0.56 mm. One 15 m
length of the hose, attached to your spigot, delivers 57 Lit/min
of water (15°C). Compute the pressure at the spigot, in kPa.
Estimate the delivery if two 15 m lengths of the hose are
connected. Assume that the pressure at the spigot varies with
flow rate and the water main pressure remains constant at
345 kPa.

Solution: Apply the energy equation for steady, incompressible flow between the spigot  and the hose discharge .

p V2  p V2 
Basic equations:  2 × α 2 2 + gz 2  −  3 × α 3 3 + gz3  = h l T (8.29)
 ρ 2   ρ 2 
L V2
h lT = h l + h lm , h l = f
D 2

Assumptions: (1) p3 = patm


(2) V2 = V3 , α2 = α3 = 1.0

(3) z2 = z3
(4) Turbulent flow so ∆p1 × Q2

Then
L V2
p 2 = pf
D 2

3
Q 4Q 4 −5 m 1 m
V= = 2
= × 95 × 10 × 2
= 3.4
A πD π s (0.19 m) s

DV 19 m
Re = = m × 3.4 × 1.14 ×10 −6 m 2 = 5.67 × 104 {V from Table A.7}
V 100 s

0.56
e/D = = 0.0295
19

From Eq. 8.37 f = 0.056*. From Eq. 1,

kg 15 m 1000 mm 1 m2 N ⋅ s2
p 2 = 999 3
× 0.056 × × × × (3.4) 2 2 ×
m 19 mm m 2 s kg ⋅ m

p2 = 255.3 kPa p2

The pressure drop from the main  to the spigot  is proportional to the square of the flow rate. Obtain the loss coefficient using
the energy equation between  and .

 p1 V2  p V2  V2
 + α1 1 + gz1  −  2 + α 2 2 + gz 2  = K 2
ρ 2   ρ 2  2
Assumptions: (4) V1 = 0

(5) z1 = z2

 V2 V2  V2
p1 − p 2 = p  K 2 + 2  = p 2 [K + 1]
 2 2  2

∆⋅p m3 s 2 kg ⋅ m
K= − 1 = 2(345 − 255.3) × 103 kPa × ⋅ (3.4)2 2 × −1
1
2
pV22 999 kg m N ⋅ s2

K = 14.5

* Value of f obtained using Excel’s Solver (or Goal Seek).


[Difficulty: 4] Part 2/2

To find the delivery with two hoses, again apply the energy equation from the main  to the end  of the second hose

 p1 V2  p V2  L V2 V2
 + α1 1 + gz1  −  4 + α 4 4 + gz 4  = f 24 4 + K 4
ρ 2   ρ 2  D 2 2

p 4 = patm , zl = z4, V1 = 0 , α4 =1

p1 − p atm p lg ⋅ s V42  L 24 
= =  f D + K + 1 and
ρ ρ 2  

1/2
 
 2p1g 
V4 =  
  L 
 p  2f D + K + 1 
  

Delivery will be reduced somewhat with two length of hose but f will not change much. Assume f ≈ 0.056 are clear.

1
 2
 3 
 3 N m 1 kg ⋅ m 
V4 = 2 × 34.5 × 10 2 × × ×
 m 999 kg  30 m 1000 mm  N s2 
  0.056 × 19 mm × + 14.5 + 1 
 m  

V4 = 2.58 m/s

Clearing,

DV 19 m s
Re = = m × 2.58 × 1.14 × 10−6 2 = 4.30 × 104 , so f ≈ 0.56
V 1000 s m

Thus with two hoses,

2
m π  19  1000 Lit 60 s
Q = V A = 2.58 × ×  × m2 × × = 44 Lit/ min Q
s 4  1000  m3 min

{Similar calculations could be performed using any desired number of hose lengths.}
Problem 8.117 [Difficulty: 4] Part 1/2
[Difficulty: 4] Part 2/2
Problem 8.118 [Difficulty: 5]

8.118 Your boss, from the “old school,” claims that for pipe
flow the flow rate, Q ∞ ∆p , where ∆p is the pressure
difference driving the flow. You dispute this, so perform some
calculations. You take a 25 mm diameter commercial steel pipe
and assume an initial flow rate of 4.7 Lit/min of water. You
then increase the applied pressure in equal increments and
compute the new flow rate so you can plot Q versus ∆p, as
computed by you and your boss. Plot the two curves on the
same graph. Was your boss right?

Applying the energy equation between inlet and exit:

∆p L V2 ∆p ρ f V 2
= f or =
ρ D 2 L D 2

2
∆p  ∆p   Q0 
“Old school”: = 
L  L  0  Q 

D = 25 mm

e = 0.046 mm

v = 1.01 × 10−6 m2/s

ρ = 998 kg/m3

Q (lit/min) Q (m3/s) V (m/s) Re f (Old school) (kPa) (kPa/m)


4.7 0.000078 0.159 3930 0.0401 0.0059 0.0059
5.7 0.000095 0.194 4790 0.038 0.0084 0.0079
6.7 0.000112 0.228 5630 0.0364 0.0114 0.01
7.7 0.000128 0.261 6450 0.035 0.0149 0.013

Flow Rate versus Pressure Drop


0.0006

0.0005
Flow (lit/min)

0.0004

0.0003

0.0002

0.0001

0
0.05 0 0.05 0.1 0.15 0.2 0.25 0.3 0.35
Pressure Drop (kPa/m)

Your boss 32.7 0.000545 1.11 27420 0.024 0.286 0.17


was wrong! 33.7 0.000562 1.14 28161 0.0238 0.302 0.18
Problem 8.119 [Difficulty: 3]

8.119 A hydraulic press is powered by a remote high-pressure


pump. The gage pressure at the pump outlet 20.7 MPa, whereas
the pressure required for the press is 18.9 MPa (gage), at a flow
3
rate of 0.00057 m /s. The press and pump are connected by
50.3 m of smooth, drawn steel tubing. The fluid is SAE 10W oil
at 38°C. Determine the minimum tubing diameter that may be
used.

Given: Hydraulic press system.

Find: Minimum required diameter of tubing.

Solution:
p V2  p V2  L V22
Basic equations:  1 × α ⋅ 1 + g ⋅ z1  −  2 + α ⋅ 2 + g ⋅ z 2  = h l hl = f ⋅ ⋅
ρ 2   ρ 2  D 2
   

Assumptions: 1) Steady flow 2) Incompressible flow 3) α at 1 and 2 is approximately 1 4) Ignore minor losses

The flow rate is low and it's oil, so try assuming laminar flow. Then, from Eq. 8.13c

1
128 ⋅ µ ⋅ Q ⋅ L  128 ⋅ µ ⋅ Q ⋅ L  4
∆p = or D=
π ⋅ D4  π ⋅ ∆p 

N ⋅s
For SAE 10W oil at 38°C (Fig. A.2) µ = 3.5 × 10−2
m2

1
 128 N m3 m2 4
Hence D= × 3.5 × 10−2 2 × 0.00057 × 50.3 m ×  D = 0.012 m D = 1.2 cm
 π m s (20.7 − 18.9)106 N 

2
Q 4⋅Q 4 m3  1 
Check Re to assure flow is laminar V= = V= × 0.00057 ×  V = 5.04 m/s
A π ⋅ D2 π s  0.012 

SG oil ⋅ρH 2O ⋅ V ⋅ D
From Table A.2 SGoil = 0.92 so Re =
µ

kg m m2 N ⋅ s2
Re = 0.92 × 998 × 5.04 × 0.012 m × × Re = 1587
m 3 s 3.5 × 10−2 N ⋅ s kg ⋅ m

Hence the flow is laminar, Re < 2300. The minimum diameter is 1.2 cm, so 1.5 cm ID tube should be chosen.
Problem 8.120 [Difficulty: 4]

Given: Flow out of reservoir by pump

Find: Smallest pipe needed

Solution:
⎛⎜ p V1
2 ⎞ ⎛⎜ p V2
2 ⎞ 2 2 2
1 2 L V2 V2 Le V2
Basic equations ⎜ ρ + α ⋅ 2 + g ⋅ z1 − ⎜ ρ + α ⋅ 2 + g⋅ z2 = h lT h lT = h l + hlm = f ⋅ D ⋅ 2 + Kent ⋅ 2 + f ⋅ D ⋅ 2
⎝ ⎠ ⎝ ⎠

Assumptions: 1) Steady flow 2) Incompressible flow 3) α at 1 and 2 is approximately 1 4) Vl <<

Hence for flow between the free surface (Point 1) and the pump inlet (2) the energy equation becomes

2 2 2 2
p2 V2 p2 V L V V Le V2
− − g ⋅ z2 − =− − g ⋅ z2 − = f⋅ ⋅ + Kent ⋅ + f ⋅ ⋅ and p = ρ⋅ g ⋅ h
ρ 2 ρ 2 D 2 2 D 2

2
V ⎡ ⎛L Le ⎞ ⎤
Solving for h 2 = p 2/ρg h 2 = −z2 − ⋅ ⎢f ⋅ ⎜ + + Kent⎥ (1)
2⋅ g ⎣ ⎝ D D⎠ ⎦
Le
From Table 8.2 Kent = 0.78 for rentrant, and from Table 8.4 two standard elbows lead to = 2 × 30 = 60
D
2
−6 m
We also have e = 0.046 ⋅ mm (Table 8.1) ν = 1.51 × 10 ⋅ (Table A.8)
s
3
L −3m
and we are given Q = 6⋅ Q = 6 × 10 z2 = 3.5⋅ m L = ( 3.5 + 4.5) ⋅ m L = 8 m h 2 = −6 ⋅ m
s s

Equation 1 is tricky because D is unknown, so V is unknown (even though Q is known), L/D and Le/D are unknown, and Re and
hence f are unknown! We COULD set up Excel to solve Eq 1, the Reynolds number, and f, simultaneously by varying D, but here
we try guesses:

4⋅ Q m V⋅ D 5
D = 2.5⋅ cm V = V = 12.2 Re = Re = 2.02 × 10
2 s ν
π⋅ D

⎛ e ⎞
1 ⎜ D 2.51
Given = −2.0⋅ log ⎜ + f = 0.0238
f ⎝ 3.7 Re⋅ f ⎠

2
V ⎡ ⎛L Le ⎞ ⎤
h 2 = −z2 − ⋅ ⎢f ⋅ ⎜ + + Kent⎥ h 2 = −78.45 m but we need -6 m!
2⋅ g ⎣ ⎝ D D⎠ ⎦
4⋅ Q m V⋅ D 5
D = 5 ⋅ cm V = V = 3.06 Re = Re = 1.01 × 10
2 s ν
π⋅ D
⎛ e ⎞
1 ⎜ D 2.51
Given = −2.0⋅ log ⎜ + f = 0.0219
f ⎝ 3.7 Re⋅ f ⎠
2
V ⎡ ⎛ L Le ⎞ ⎤
h 2 = −z2 − ⋅ ⎢f ⋅ ⎜ + + Kent⎥ h 2 = −6.16 m but we need -6 m!
2⋅ g ⎣ ⎝ D D⎠ ⎦
4⋅ Q m V⋅ D 4
D = 5.1⋅ cm V = V = 2.94 Re = Re = 9.92 × 10
2 s ν
π⋅ D
⎛ e ⎞
1 ⎜ D 2.51
Given = −2.0⋅ log ⎜ + f = 0.0219
f ⎝ 3.7 Re⋅ f ⎠

2
V ⎡ ⎛L Le ⎞ ⎤
h 2 = −z2 − ⋅ ⎢f ⋅ ⎜ + + Kent⎥ h 2 = −5.93 m
2⋅ g ⎣ ⎝ D D⎠ ⎦

To within 1%, we can use 5-5.1 cm tubing; this corresponds to standard 2 in pipe.
Problem 8.121 [Difficulty: 4]
Problem 8.122 [Difficulty: 4]

8.122 Air at 20°C flows in a horizontal square cross-section


duct made from commercial steel. The duct is 25 m long. What
3
size (length of a side) duct is required to convey 2 m /s of air
with a pressure drop of 1.5 cm H2O?

Given: Flow of air in square duct.

Find: Minimum required size.

Solution:
p V2  p V2  L V2 4⋅ A
Basic equations:  1 + α ⋅ 1 + g ⋅ z1  −  2 + α ⋅ 2 + g ⋅ z 2  = h l hl = f ⋅ ⋅ Dh =
ρ   
 2   ρ 2  Dh 2 Pw

Assumptions: 1) Steady flow 2) Incompressible flow 3) α at 1 and 2 is approximately 1 4) Ignore minor losses

m3
Available data Q=2 L = 25 m e = 0.046 mm (Table 8.1) ∆h = 1.5 cm
s

ρH 2O = 1000 kg / m3 ν = 1.50 × 10−5 m 2 /s ρ = 1.21 kg / m3 (Table A.10)

Hence for flow between the inlet (Point 1) and the exit (2) the energy equation becomes

p1 p 2 ∆p L V2
− = =f⋅ ⋅ and ∆p = ρH 2O ⋅ g ⋅ ∆h ∆p = 147 Pa
ρ ρ ρ Dh 2

4⋅ h ⋅h
For a square duct Dh = =h and also A = h ⋅ h = h2
2 ⋅ (h + h)

V2 Q2 ρ⋅ f ⋅ L ⋅ Q 2
Hence ∆p = ρ⋅ f ⋅ L = ρ⋅ f ⋅ L ⋅ =
2⋅ h 2 ⋅ h ⋅ A2 2 ⋅ h5

1
 ρ⋅ f ⋅ L ⋅ Q 2  5
Solving for h h=  (1)
 2 ⋅ ∆p 

Equation 1 is tricky because h is unknown, so Dh is unknown, hence V is unknown (even though Q is known), and Re and hence f
are unknown! We COULD set up Excel to solve Eq 1, the Reynolds number, and f, simmultaneously by varying h, but here we try
guesses:
1
 ρ⋅ f ⋅ L ⋅ Q 2  5 Q
f = 0.01 h=  h = 0.333 m V= V = 18.0 m/s
 2 ⋅ ∆p  h2

V ⋅ Dh
Dh = h Dh = 0.333 m Re = Re = 4.00 × 105
ν

 e 
= −2.0 ⋅ log  D h 
1
Given 2.51 f = 0.0152
 
f  3.7 + 
 Re⋅ f 
1
 ρ⋅ f ⋅ L ⋅ Q 2  5 Q ft
h=  h = 0.362 m V= 2
V = 15.2 ⋅
 2 ⋅ ∆p  h s

V ⋅ Dh
Dh = h Dh = 0.362 m Re = Re = 3.68 × 105
ν

 e 
1 D 2.51 
Given = −2.0 ⋅ log  h +  f = 0.0153
f  3.7 Re⋅ f 
 

1
 ρ⋅ f ⋅ L ⋅ Q 2  5 Q m
h=  h = 0.363 m V= 2
V = 15.2
 2 ⋅ ∆p  h s

V ⋅ Dh
Dh = h Dh = 0.363 m Re = Re = 3.68 × 105
ν

In this process h and f have converged to a solution. The minimum dimensions are 0.363 m by 0.363 m, or 36.3 cm by 36.3 cm.
Problem 8.123 [Difficulty: 3]

Given: Flow in a tube

Find: Effect of diameter; Plot flow rate versus diameter

Solution:
Basic equations: ⎛⎜ p V1
2 ⎞ ⎛⎜ p V2
2 ⎞
1 2 (8.29)
⎜ρ + α ⋅
1 2 + g ⋅ z 1 − ⎜ + α ⋅
2 2 + g ⋅ z 2 = hl
⎝ ⎠ ⎝ρ ⎠
2
ρ⋅ V⋅ D L V
Re = hl = f ⋅ ⋅ (8.34)
μ D 2
⎛ e ⎞
64 1 ⎜ D 2.51
f = (8.36) (Laminar) = −2.0⋅ log ⎜ + (8.37) (Turbulent)
Re f ⎝ 3.7 Re⋅ f ⎠
The energy equation (Eq. 8 .29) b ecomes for flow in a tube
2
L V
p 1 − p 2 = ∆p = ρ⋅ f ⋅ ⋅
D 2

This cannot be solved explicitly for velocity V (and hence flow rate Q), because f depends on V; solution for a given diameter D
requires iteration (or use of Solver)
Flow Rate versus Tube Diameter for Fixed Dp
0.8

0.6
3
Q (m /s) Laminar
4
x 10 0.4 Turbulent

0.2

0.0
0.0 2.5 5.0 7.5 10.0
D (mm)
Problem 8.124 [Difficulty: 3] Part 1/2
[Difficulty: 3] Part 2/2
Problem 8.125 [Difficulty: 4]
Problem 8.126 [Difficulty: 2]

8.126 A pump draws water at a steady flow rate of 11.3 kg/s


through a piping system. The pressure on the suction side of the
pump is −17.2 kPa (gage). The pump outlet pressure is 345 kPa
(gage). The inlet pipe diameter is 75 mm; the outlet pipe
diameter is 50 mm. The pump efficiency is 70 percent.
Calculate the power required to drive the pump.

Given: Flow through water pump.

Find: Power required.

Solution:
p V2  p V2  Q 4⋅Q
Basic equations: h pump =  d × d + g ⋅ z d  −  s + s + g ⋅ zs  V= =
 ρ    A π ⋅ D2
 2   ρ 2 

Assumptions: 1) Steady flow 2) Incompressible flow 3) Uniform flow

2
4 kg m3  1000 × 1  m
Hence for the inlet Vs = × 11.3 × ×  Vs = 2.56 ps = −17.2 kPa
π s 998 kg  75 m  s

2
4 kg m3  1000 × 1 m
For the outlet Vd = × 11.3 × × Vd = 5.77 pd = 345 kPa
π s 998 kg  50 m  s

pd − ps Vd2 − Vs2
Then h pump = + and Wpump = m pump ⋅ h pump
ρ 2

 p − ps Vd2 − Vs2 
w pump = m pump ⋅  d + 
 ρ 2
 

Note that the software cannot render a dot, so the power is Wpump and mass flow rate is mpump!

kg  m Ns 
2
N m3 1 2
Wpump = 11.3 × (345 − (−17 ⋅ 2)) × 103 2 × + × (5.77 2 − 2.562 )   × 
s 
 m 998 kg 2  s  kgm 

Wpump
Wpump = 4.252 kW For an efficiency of η = 70% Wrequired = Wrequired = 6.074 kW
η
Problem 8.127 [Difficulty: 1]

8.127 The pressure rise across a water pump is 75 kPa when the
volume flow rate is 25 L/s. If the pump efficiency is 80 percent,
determine the power input to the pump.

Given: Flow through water pump.

Find: Power required.

Solution:
p V2  p V2 
Basic equations: h pump =  d + d + g ⋅ z d  −  s + s + g ⋅ zs 
 ρ 2   ρ 2 
   

Assumptions: 1) Steady flow 2) Incompressible flow 3) Uniform flow

In this case we assume Ds = Dd so Vs = V d

The available data is ∆p = 75 kPa Q = 25 L/s η = 80%

p d − p s ∆p
Then h pump = = and Wpump = m pump ⋅ h pump
ρ ρ

∆p ∆p
w pump = m pump ⋅ = ρ⋅ Q ⋅ = Q ⋅ ∆p
ρ ρ

L 0.001 m3 N
w pump = 25 × × 75 × 103 2 Wpump = 1.88 kW
s 1⋅ L m

Note that the software cannot render a dot, so the power is Wpump and mass flow rate is mpump!

Wpump
For an efficiency of η = 80% Wrequired = Wrequired = 2.34 kW
η
Problem 8.128 [Difficulty: 3]

Given: Flow in pipeline with pump

Find: Pump pressure ∆p

Solution:
⎛⎜ p V1
2 ⎞ ⎛⎜ p V2
2 ⎞ ∆p
1 2 pump
Basic equations ⎜ ρ + α⋅ 2 + g⋅ z1 − ⎜ ρ + α⋅ 2 + g⋅ z2 + = h lT
⎝ ⎠ ⎝ ⎠ ρ

2 Le V2 2
L V V
hl = f ⋅ ⋅ h lm = f ⋅ ⋅ h lm = K⋅
D 2 D 2 2

Assumptions: 1) Steady flow 2) Incompressible flow 3) α is approximately 1


l
Available data L = 50⋅ m D = 125 ⋅ mm Q = 50⋅ e = 0.15⋅ mm
s
p 1 = 150 ⋅ kPa p 2 = 0 ⋅ kPa z1 = 15⋅ m z2 = 30⋅ m

From Section 8.8 Kent = 0.5 Lelbow90 = 30⋅ D Lelbow90 = 3.75 m LGV = 8 ⋅ D LGV = 1 m

kg − 3 N⋅ s
LAV = 150 ⋅ D LAV = 18.75 m ρ = 1000 μ = 1.3⋅ 10 ⋅ (Table A.8)
3 2
m m

Q m ρ⋅ V⋅ D 5
Hence V = V = 4.07 Re = Re = 3.918 × 10
⎛ π⋅ D 2⎞ s μ

⎝ 4 ⎠
⎛ e ⎞
1 ⎜ D 2.51
and Given = −2 ⋅ log⎜ + f = 0.0212
f ⎝ 3.7 Re⋅ f ⎠
The loss is then
2 2
V ⎛ L Lelbow90 LGV LAV ⎞ m
h lT = ⋅⎜f ⋅
+ 7⋅ f ⋅ + 5⋅ f ⋅ + f⋅ + Kent h lT = 145
2 ⎝ D D D D ⎠ 2
s

p1 − p2 2 ∆ppump
V
The energy equation becomes
ρ
(
+ g ⋅ z1 − z2 −
2
)+
ρ
= h lT

2
V
(
∆ppump = ρ⋅ h lT + ρ⋅ g ⋅ z2 − z1 + ρ⋅
2
)
+ p2 − p1 ( ) ∆ppump = 150 ⋅ kPa
Problem 8.129 [Difficulty: 3]

8.129 Cooling water is pumped from a reservoir to rock drills Vj 5 37 m/s


Pipe, D 5 100 mm
on a construction job using the pipe system shown. The flow (aluminum)
rate must be 38 Lit/s and water must leave the spray nozzle at Total length: L 5 213 m
37 m/s. Calculate the minimum pressure needed at the pump Joints: 15, each with
outlet. Estimate the required power input if the pump efficiency Kjoint 5 1 122 m
is 70 percent. Pump

Gate valve, open

Solution:

 2   2 
 p1 + α V1 + gz  −  p 2 + α V 2 + gz  + ∆h
Computing equations: pump = h lT (8.48)
 ρ   ρ
1 1 2 2
2 2 
  

L V2 V2  Le 
h lT = h l + h l ,
m
hl = f
D 2
, h lm =
2 
 ∑ K +∑ f  D   .
Assumptions: (1) V1 = 0 (2) α2 = α3 = 1 (3) p1 = p2 = patm

Then
V22 L V2 V2   Le   Le  
∆h pump = gz 2 + +f +  K ent + f   + 2f   + 15K j  (1)
2 D 2 2   D 90°el  D  45°el 

Q 4Q 4 38 Lit m3 1
V= = 2
= × × × = 4.84 m/s
A πD π s 1000 Lit (0.1 m) 2

DV 100 m
Re = = × 4.84 × 1.14 × 10 −6 m 2 = 4.25 × 105 {Vat T = 15°C, Table A.8}
V 1000 s

Table 8.1, e = 0.0015 mm (drawn tubing) ∴ e/D = 1.5 ×10−5

From Fig. 8.13, f = 0.0135

From Table 8.1, Kent = 0.78

From Table 8.4, Le(D)g⋅4 = 8, Le(D)90°el = 30, Le(D)45°el = 16

Then from Eq. (1)

m 1 m2 213 1 m2 1 m2
∆h pump = 9.81 2
× 122 m + (37) 2 2 + 0.0135 × × × (4.84)2 2 + (4.84)2 2 [0.78 + 0.0135(30) + 2 × 0.0135(16) + 15(1)]
s 2 s 0.1 2 s 2 s

m2
∆h pump = 2.4 × 103
s2


The theoretical power input to the pump is given by W pump = m
 ∆h pump
From the definition of efficiency, η = w
 theor /w
 act , then

∆h pump Q∆h pump


 =m
W  =ρ
act
η η

3 2
 = 999 kg × 3.8 Lit × m 3 m N s2
Wact × 2.4 × 10 × = 130.2 kW
0.7 m3 s 1000 Lit s 2 kg ⋅ m

The discharge pressure from the pump is obtained by applying Eq. 8.4B between sections ① and ③ neglecting losses in the inlet
section, elevation change, and kinetic energy at ③

kg m2 N ⋅ s2
p3 − pA = p∆h pump = 999 × 2.4 × 103 × = 2398 kPa p3
m3 s2 kg ⋅ m
Problem 8.130 [Difficulty: 3]

Given: Flow in air conditioning system

Find: Pressure drop; cost

Solution:
⎛⎜ p V1
2 ⎞ ⎛⎜ p V2
2 ⎞ 2
1 2 L V
Basic equations ⎜ρ + α⋅ + g ⋅ z 1 − ⎜ + α⋅ + g ⋅ z 2 = hl hl = f ⋅ ⋅
⎝ 2 ⎠ ⎝ρ 2 ⎠ D 2

Assumptions: 1) Steady flow 2) Incompressible flow 3) α is approximately 1


3
m
Available data L = 5 ⋅ km D = 0.75⋅ m e = 0.046 ⋅ mm Q = 0.65⋅ ηp = 85⋅ % ηm = 85⋅ %
s

kg − 3 N⋅ s 0.14
ρ = 1000 μ = 1.3⋅ 10 ⋅ (Table A.8) cost = (dollars)
3 2 kW⋅ hr
m m

Q m ρ⋅ V⋅ D 5
Then V = V = 1.47 Re = Re = 8.49 × 10
⎛ π⋅ D 2⎞ s μ

⎝ 4 ⎠
⎛ e ⎞
1 ⎜ D 2.51
so Given = −2 ⋅ log⎜ + f = 0.0131
f ⎝ 3.7 Re⋅ f ⎠
2
L V
The energy equation becomes ∆p = f ⋅ ⋅ ρ⋅ ∆p = 94.4⋅ kPa
D 2

and Wpump = Q⋅ ∆p Wpump = 61.3⋅ kW

Wpump
The required power is Power = Power = 84.9⋅ kW
ηp ⋅ ηm

The daily cost is then C = cost⋅ Power⋅ day C = 285 dollars


Problem 8.131 [Difficulty: 4]

d e f
c

Given: Fire nozzle/pump system

Find: Design flow rate; nozzle exit velocity; pump power needed

Solution:
⎛⎜ p V2
2 ⎞ ⎛⎜ p V3
2 ⎞ 2
2 3 L V2
Basic equations ⎜ ρ + α⋅ 2 + g⋅ z2 − ⎜ ρ + α⋅ 2 + g⋅ z3 = h l hl = f ⋅ ⋅ for the hose
⎝ ⎠ ⎝ ⎠ D 2

Assumptions: 1) Steady flow 2) Incompressible flow 3) α at 2 and 3 is approximately 1 4) No minor loss

2 2
p3 V3 p4 V4
+ + g ⋅ z3 = + + g ⋅ z4 for the nozzle (assuming Bernoulli applies)
ρ 2 ρ 2

⎛⎜ p V2
2 ⎞ ⎛⎜ p V1
2 ⎞
2 1
⎜ ρ + α⋅ 2 + g⋅ z2 − ⎜ ρ + α⋅ 2 + g⋅ z1 = h pump for the pump
⎝ ⎠ ⎝ ⎠

Assumptions: 1) Steady flow 2) Incompressible flow 3) α at 1 and 2 is approximately 1 4) No minor loss

p2 − p3 2
∆p L V 2 ⋅ ∆p⋅ D
Hence for the hose = = f⋅ ⋅ or V=
ρ ρ D 2 ρ⋅ f ⋅ L

We need to iterate to solve this for V because f is unknown until Re is known. This can be done using Excel's Solver, but here:
2
kg −6 m
∆p = 750 ⋅ kPa L = 100 ⋅ m e = 0 D = 3.5⋅ cm ρ = 1000⋅ ν = 1.01 × 10 ⋅
3 s
m

2 ⋅ ∆p⋅ D m V⋅ D 5
Make a guess for f f = 0.01 V = V = 7.25 Re = Re = 2.51 × 10
ρ⋅ f ⋅ L s ν

⎛ e ⎞
1 ⎜ D 2.51
Given = −2.0⋅ log ⎜ + f = 0.0150
f ⎝ 3.7 Re⋅ f ⎠

2 ⋅ ∆p⋅ D m V⋅ D 5
V = V = 5.92 Re = Re = 2.05 × 10
ρ⋅ f ⋅ L s ν

⎛ e ⎞
1 ⎜ D 2.51
Given = −2.0⋅ log ⎜ + f = 0.0156
f ⎝ 3.7 Re⋅ f ⎠
2 ⋅ ∆p⋅ D m V⋅ D 5
V = V = 5.81 Re = Re = 2.01 × 10
ρ⋅ f ⋅ L s ν

⎛ e ⎞
1 ⎜ D 2.51
Given = −2.0⋅ log ⎜ + f = 0.0156
f ⎝ 3.7 Re⋅ f ⎠

2 ⋅ ∆p⋅ D m V⋅ D 5
V = V = 5.80 Re = Re = 2.01 × 10
ρ⋅ f ⋅ L s ν

2 3 3
π⋅ D π 2 m −3m m
Q = V⋅ A = ⋅V Q = × ( 0.035 ⋅ m) × 5.80⋅ Q = 5.58 × 10 Q = 0.335 ⋅
4 4 s s min

p3 V3
2
p4 V4
2
(
2⋅ p3 − p4 ) 2
For the nozzle + + g ⋅ z3 = + + g ⋅ z4 so V4 = + V3
ρ 2 ρ 2 ρ

3 2
kg⋅ m
+ ⎛⎜ 5.80⋅
3 N m m⎞ m
V4 = 2 × 700 × 10 ⋅ × × V4 = 37.9
m
2 1000⋅ kg 2
s ⋅N ⎝ s ⎠ s

⎛⎜ p V2
2 ⎞ ⎛⎜ p V1
2 ⎞ p2 − p1
For the pump 2 1
⎜ρ + α⋅ + g ⋅ z 2 − ⎜ + α⋅ + g ⋅ z 1 = h pump so h pump =
⎝ 2 ⎠ ⎝ρ 2 ⎠ ρ

p 1 = 350 ⋅ kPa p 2 = 700 ⋅ kPa + 750 ⋅ kPa p 2 = 1450⋅ kPa

The pump power is Ppump = mpump⋅ h pump P pump and mpump are pump power and mass flow rate (software can't do a dot!)

(p2 − p1) −3 m
3
3 N
Ppump = ρ⋅ Q⋅
ρ
(
= Q⋅ p 2 − p 1 ) Ppump = 5.58 × 10 ⋅
s
× ( 1450 − 350 ) × 10 ⋅
2
Ppump = 6.14⋅ kW
m

Ppump 6.14⋅ kW
Prequired = Prequired = Prequired = 8.77⋅ kW
η 70⋅ %
Problem 8.132 [Difficulty: 2]

Given: Flow in water fountain

Find: Daily cost

Solution:

Basic equations Wpump = Q⋅ ∆p ∆p = ρ⋅ g ⋅ H

Assumptions: 1) Steady flow 2) Incompressible flow 3) α is approximately 1

3
m
Available data Q = 0.075 ⋅ H = 10⋅ m ηp = 85⋅ % ηm = 85⋅ %
s

kg 0.14
ρ = 999 ⋅ Cost = (dollars)
3 kW⋅ hr
m

Hence ∆p = ρ⋅ g ⋅ H ∆p = 98⋅ kPa

Wpump = Q⋅ ∆p Wpump = 7.35⋅ kW

Wpump
Power = Power = 10.2⋅ kW
ηp ⋅ ηm

C = Cost⋅ Power⋅ day C = 34.17 (dollars)


Problem 8.133 Equations [Difficulty: 4]

8.133 A water pump can generate2 a pressure difference


∆p (kPa) given by ∆p = 999 − 859 Q , where the flow rate is
3
Q m /s. It supplies a pipe of diameter 0.5 m, roughness 13 mm,
and length 760 m. Find the flow rate, pressure difference, and
the power supplied to the pump if it is 70 percent efficient. If
the pipe were replaced with one of roughness 6 mm, how much
would the flow increase, and what would the required power
be?

Given: Pump/pipe system.

Find: Flow rate, pressure drop, and power supplied; Effect of roughness.

Solution:
ρ⋅V ⋅D  p1 V2  p V2  L V2
Re =  + α1 ⋅ 1 + g ⋅ z1  −  2 + α2 ⋅ 2 + g ⋅ z 2  = h1T − ∆h pump h1T = f ⋅ ⋅
µ ρ 2   ρ 2  D 2

 e 
64 1
f= (Laminar) = −2.0 ⋅ log  D 2.51  (Turbulent)
Re f  + 
 3.7 Re⋅ f 

The energy equation becomes for the system (1 = pipe inlet, 2 = pipe outlet)

L V2 L V2
∆h pump = f ⋅ ⋅ or ∆ppump = ρ ⋅ f ⋅ ⋅ (1)
D 2 D 2

This must be matched to the pump characteristic equation; at steady state, the pressure generated by the pump just equals that lost
to friction in the circuit

∆p pump = 999 − 859 ⋅ Q2 (2)

Finally, the power supplied to the pump, efficiency η, is

Q ⋅ ∆p
Power = (3)
η

In Excel:

Tabulated or graphical data: Given data:


µ = 1.02E-03 N ⋅ s/m2 L = 760 m
ρ = 103 kg/m3 D = 0.5 m
(Appendix A) ηpump = 70%
Computed Results: e = 13 mm
3
Q (m /s) V(m/s) Re f p(MPa)(Eq 1) p(MPa)(Eq 2)
0.28 1.43 7.01 × 105 0.0539 0.08 0.93
0.34 1.73 8.48 × 105 0.0539 0.12 0.9
0.4 2.04 10.0 × 105 0.0539 0.17 0.86
0.45 2.29 1.12 × 106 0.0539 0.21 0.83
0.51 2.6 1.27 × 106 0.0539 0.28 0.78
0.57 2.9 1.42 × 106 0.0539 0.34 0.72
0.62 3.16 1.55 × 106 0.0539 0.41 0.69
0.68 3.46 1.70 × 106 0.0539 0.49 0.6
0.75 3.82 1.87 × 106 0.0539 0.59 0.52
0.79 4.02 1.97 × 106 0.0539 0.66 0.46
0.84 4.28 2.10 × 106 0.0539 0.75 0.39
Error (approx)
0.74 3.77 1.85 × 106 0.0539 0.58 0.53 0 Using Solver!

Power = 0.59 MW (Eq. 3)


Repeating, with smoother pipe

Computed results: e = 6 mm
3
Q (m /s) V(m/s) Re f p(MPa)(Eq 1) p(MPa)(Eq 2)
0.28 1.43 7.01 × 105 0.0404 0.06 0.93
0.34 1.73 8.48 × 105 0.0404 0.09 0.9
0.4 2.04 10.0 × 105 0.0404 0.13 0.86
0.45 2.29 1.12 × 106 0.0404 0.16 0.83
0.51 2.6 1.27 × 106 0.0404 0.21 0.78
0.57 2.9 1.42 × 106 0.0404 0.26 0.72
0.62 3.16 1.55 × 106 0.0404 0.31 0.69
0.68 3.46 1.70 × 106 0.0404 0.37 0.6
0.74 3.82 1.87 × 106 0.0404 0.44 0.52
0.8 4.07 1.99 × 106 0.0404 0.5 0.46
0.84 4.28 2.10 × 106 0.0404 0.56 0.39
Error (approx)
0.79 4.02 1.97 × 106 0.0404 0.49 0.46 0 Using Solver!

Power = 0.53 MW (Eq. 3)

Pump and Pipe Pressure Heads


1
Pipe (l 5 13 mm)
0.8 Pipe (l 5 6 mm)
Pump
p (MPa)

0.6

0.4

0.2

0
0 0.1 0.2 0.3 0.4 0.5 0.6 0.7 0.8 0.9
Q (m3/s)
Problem 8.134 [Difficulty: 3]

Given: Fan/duct system

Find: Flow rate

Solution:

⎛⎜ p V1
2 ⎞ ⎛⎜ p V2
2 ⎞ 2 2
1 2 L V V
⎜ ρ + α1⋅ 2 + g ⋅ z1 − ⎜ ρ + α2 ⋅ 2 + g⋅ z2 = h lT − ∆hfan h lT = f ⋅ ⋅ + K⋅
⎝ ⎠ ⎝ ⎠ Dh 2 2

The energy equation becomes for the system (1 = duct inlet, 2 = duct outlet)
2 2
L V V
∆hfan = f ⋅ ⋅ + K⋅
Dh 2 2

2 2
ρ⋅ V L 4⋅ A 4⋅ h
or ∆ppump = ⋅ ⎛⎜ f ⋅ + K⎞ (1) where Dh = = =h
2
⎝ Dh ⎠ Pw 4⋅ h

This must be matched to the fan characteristic equation; at steady state, the pressure generated by the fan just equals that lost to
friction in the circuit

2
∆pfan = 1020 − 25⋅ Q − 30⋅ Q (2)

In Excel:
Fan and Duct Pressure Heads
2500

2000

1500
Dp (Pa)

1000 Duct
Fan
500

0
0.0 0.5 1.0 1.5 2.0 2.5 3.0
3
Q (m /s)
Problem 8.135 [Difficulty: 4]

8.135 The head versus capacity curve for a certain fan may be
2
approximated by the equation H = 762 − 11.4 Q , where H is
the output static head in mm of water and Q is the air flow rate
3
in m /s. The fan outlet dimensions are 200 × 400 mm.
Determine the air flow rate delivered by the fan into a 61 m
straight length of 200 × 400 mm rectangular duct.

Solution:
(1) (2) (1) (2)
 p 2   p
2 
Basic equation:  1 + α1 V 1 + z1  −  2 + α 2 V 2 + z2  = H p (8.30)
 pg zg   pg zg  T
   

h V2 4A
H pT = f + h pm ; Dh =
D h zg Pw

Assumptions: (1) V1 = V2 , α1 = α2 ≈ 1

Duct a 5 200 mm
(2) z1 = z2
b 5 400 mm
(3) h pm = 0

200 400
A = ab = m× m = 0.08 m2
1000 1000

4A 4A 2 × 0.08 m2
Dh = = = = 0.267 m
Pw 2(a + b) (0.2 + 0.4)m

L V2 L P Q2
From Eq. 8.30 ∆p = f ρair =f = γ H2O H duct
D 2 D 2 A2

where Hduct is the pressure drop in head of water.

f L ρair Q2 f 61m m3 1 Qm 3 1000 mm


H duct = = × × 1.23 kg / m3 × × × ×
2γ H2O D H A 2 2 0.267 m 9800 N 0.08 m2 s m

Hduct = 2240 fQ2 (where H is in mm, H2O) (1)

For a smooth duct, f = f(Re)

VD h Dh Q m2
Re = = . For T = 20°C, from Table A.10, V = 1.5 ×10 −5
V VA s

0.267 m 1 s m3
Re = × × 105 2 × Q = 222500 Q
0.08 m2 1.5 m s

To determine the air flow rate delivered we need to determine the operating point of the fan.
The operating point is at the intersection of the far head capacity curve, and the system curve (the head loss in the duct).

This is shown on the plot below.

Note that the friction factor f is determined from the Colebrook equation (8.37a) using Eq. 8.37b for the initial estimate of f.

Q Re H(fan) f0 f H(duct)
(m3/s) (-) (mm.H2O) (-) (-) (mm.H2O)
0 762 0
2.4 5.34E+05 696.3 0.0130 0.0131 169.0
3.5 7.79E+05 622.4 0.0121 0.0122 334.0
4.5 1.07E+05 499.3 0.0115 0.0116 598.7
5.9 1.31E+05 365.2 0.0111 0.0112 873.0
7.2 1.61E+05 171.0 0.0108 0.0108 1254.1

Operating point

Q = 4.5 m3/s

H = 499.3 (mm, H2O)

1125

1000

875
Duct
750
H (mm, H2O)

625

500

375
Fan
250

125

0
0 2.25 4.50 6.75
Q (m3/s)
Problem 8.136 [Difficulty: 5]

Given: Pipe system

Find: Flow in each branch

Solution:
⎛⎜ p V1
2 ⎞ ⎛⎜ p V2
2 ⎞ 2
1 2 L V
Governing equations: ⎜ ρ + α1⋅ 2 + g ⋅ z1 − ⎜ ρ + α2 ⋅ 2 + g⋅ z2 = h l (8.29) h lT = f ⋅ ⋅ (8.34)
⎝ ⎠ ⎝ ⎠ D 2

⎛⎜ e ⎞
64 1 D 2.51
f = (Laminar) (8.36) = −2.0⋅ log ⎜ + ⎟ (Turbulent) (8.37)
Re 0.5 ⎜ 3.7 0.5
f ⎝ Re⋅ f ⎠
2
L V
The energy equation (Eq. 8.29) can be simplified to ∆p = ρ⋅ f ⋅ ⋅
D 2
This can be written for each pipe section
In addition we have the following contraints
Q0 = Q1 + Q4 (1) Q4 = Q2 + Q3 (2)

∆p = ∆p0 + ∆p1 (3) ∆p = ∆p0 + ∆p4 + ∆p2 (4)

∆p2 = ∆p3 (5)

We have 5 unknown flow rates (or, equivalently, velocities) and five equations

In Excel:
Problem 8.137 [Difficulty: 5]

Given: Pipe system

Find: Flow in each br anch if pi pe 3 is blocked

Solution:
⎛⎜ p V1
2 ⎞ ⎛⎜ p V2
2 ⎞ 2
1 2 L V
Governing equations: ⎜ ρ + α1⋅ 2 + g ⋅ z1 − ⎜ ρ + α2 ⋅ 2 + g⋅ z2 = h l (8.29) h lT = f ⋅ ⋅ (8.34)
⎝ ⎠ ⎝ ⎠ D 2

⎛⎜ e ⎞
64 1 D 2.51
f = (Laminar) (8.36) = −2.0⋅ log ⎜ + ⎟ (Turbulent) (8.37)
Re 0.5 ⎜ 3.7 0.5
f ⎝ Re⋅ f ⎠
2
L V
The energy equation (Eq. 8.29) can be simplified to ∆p = ρ⋅ f ⋅ ⋅
D 2
This can be written for each pipe section
In addition we have the following contraints
Q0 = Q1 + Q4 (1) Q4 = Q2 (2)

∆p = ∆p0 + ∆p1 (3) ∆p = ∆p0 + ∆p4 + ∆p2 (4)

We have 4 unknown flow rates (or, equivalently, velocities) and four equations

In Excel:
Problem 8.138 [Difficulty: 5]

8.138 A cast-iron pipe system consists of a 46 m section of


water pipe, after which the flow branches into two 46 m
sections. The two branches then meet in a final 46 m section.
Minor losses may be neglected. All sections are 38 mm
diameter, except one of the two branches, which is 25 mm
diameter. If the applied pressure across the system is 345 kPa,
find the overall flow rate and the flow rates in each of the two
branches.

Given: Water pipe system.

Find: Flow rates.

Solution:

p V2  p V2  L V2
Basic equations  1 + α1 ⋅ 1 + g ⋅ z1  −  2 + α 2 ⋅ 2 + g ⋅ z 2  = h1 h1T = f ⋅ ⋅
ρ 2   ρ 2  D 2

 e 
64 1
f= (Laminar) = −2.0 ⋅ log  D + 2.51  (Turbulent)
Re f  
 3.7 Re ⋅ f 

L V2
The energy equation can be simplified to ∆p = ρ ⋅ f ⋅ ⋅
D 2

This can be written for each pipe section

LA VA2
Pipe A (first section) ∆p A = ρ ⋅ f A ⋅ ⋅ (1)
DA 2

L B VB2
Pipe B (1.5 in branch) ∆p B = ρ ⋅ f B ⋅ ⋅ (2)
DB 2

LC VC2
Pipe C (1 in branch) ∆p C = ρ ⋅ f C ⋅ ⋅ (3)
DC 2

L D VD2
Pipe D (last section) ∆p D = ρ ⋅ f D ⋅ ⋅ (4)
DD 2

In addition we have the following contraints

QA = QD (5)

Q A = Q B + QC (6)

∆p = ∆pA + ∆pB + ∆pD (7)

∆p B = ∆p C (8)

We have 4 unknown flow rates (or velocities) and four equations (5 - 8); Eqs 1–4 relate pressure drops to flow rates (velocities).
In Excel:

The workbook for Example problem 8.11 is modified for use in this problem

Pipe Data:
Pipe L (m) D (m) e (m)
A 46 38 0.00026
B 46 38 0.00026
C 46 25 0.00026
D 46 38 0.00026

Fluid Properties:
p = 998 kg/m3
µ = 1.00E-03 N.s/m2

Available Head:
∆p = 345 kPa

Flows: QA(m3/s) QB(m3/s) QC(m3/s) QD(m3/s)


0.0029 0.0022 0.007 0.0029

VA(m/s) VB(m/s) VC(m/s) VD(m/s)


2.56 1.9 1.44 2.56

ReA ReB ReC ReD


9.61E+04 7.13E+04 3.56E+04 9.61E+04

fA fB fC fD
0.0342 0.0345 0.0397 0.0342

Heads: ∆pA (kPa) ∆pB (kPa) ∆pC (kPa) ∆pD (kPa)


135.4 75.4 75.4 135.4

Constraints: (5) QA = QD (6) QA = QB + QC


0.00% 0.05%

(7) ∆p = ∆pA + ∆pB + ∆pC (8) ∆pB = ∆pC


0.00% 0.00%

Error: 0.00% Vary QA, QB, QC, and QD


Using Solver to minimize total error
Problem 8.139 [Difficulty: 4]

8.139 A swimming pool has a partial-flow filtration system. 3m


Total length:
12.2 m
Water at 24°C is pumped from the pool through the system From
shown. The pump delivers 1.9 Lit/s. The pipe is nominal 20 mm pool
Total length:
PVC (i.d. = 20.93 mm). The pressure loss through the filter is 6.1 m
2
approximately ∆p = 1039 Q , where ∆p is in kPa and Q is in patm
Filter
Lit/s. Determine the pump pressure and the flow rate through
each branch of the system.

Solution: Apply the energy equation for steady, incompressible pipe flow.

p1 V2 p V2   L Le   V2
Computing equations + α1 1 + gz1 = 2 + α 2 2 + gz 2 + h lT ; h lT = f  +  + K
ρ 2 ρ 2  D D   2

Assumptions: (1) α1V12 = α 2 V22 ; (2) z1 = z2, (3) h lm = 0 for 1 → 2, (4) Ignore “tee” at 

Q
The flow rate is Q12 = 1.9 Lit/s (0.0014 m3/sec), so V = = 5.5 m/s. Then
A
VD m 20.93 m s
Re = = 5.5 × × −7 2
= 1.24 × 105 , so f = 0.017
ν s 1000 9.29 × 10 m

L V2 3m 1 kg m 2 N ⋅ s 2 1000 mm
∆p12 = f ρ = 0.017 × × × 998 3 × 5.5 × × = 36.8 kPa
D 2 20.93 mm 2 m s kg ⋅ m 1m

Branch flow rates are unknown, but flow split must produce the same drop in each branch. Solve by iteration to obtain

Q 23 = 0.328 Lit /s; V23 = 0.953 m/s; Re = 2.15 × 10 4 , and f = 0.025*

L Le  V 2
∆p 23 = f  + 2  ρ + 0.6Q 2
D D 2

 6100 1 kg m2 N
∆p 23 = 0.025  + 2(30) × 998 3 × 0.953 2 × + (1039 × 0.328) 2 kPa = 115.76 kPa
 20.93  2 m s kg ⋅ m

Q 24 = 1.572 Lit/s; V24 = 4.6 m/s; Re = 1.03 × 105 , and f = 0.018

2
 L Le  V  12200 1 kg m2 N
∆p 24 = f  +  ρ = 0.018  + 30  × 998 3 × 4.6 2 × = 116.5 kPa
 D D  2  20.93  2 m s kg ⋅m

The pump outlet pressure is

∆p pump = ∆p12 + ∆p 23 = (36.8 + 115.76)kPa = 152.56 kPa ∆p

The branch flow rates are


Q 23 ≈ 0.329 Lit/s Q23

Q 24 ≈ 1.572 Lit/s Q24

*Value of f obtained from Eq 8.37 using Excel’s Solver (or Goal Seek).
Problem 8.140 [Difficulty: 5] Part 1/2
[Difficulty: 5] Part 2/2
Problem 8.141 [Difficulty: 2]
Problem 8.142

Solution:

Basic Equation:
mactual  K  At  2   P1  P2 
 K  At  2 P
(NOTE) that mactual is mass flow rate (the software cannot render a dot!)

From table A-8, At 70


  978 kg/m3 ,
v  4.10 107 m 2 /s
Now,
Q
V
A
4 1 L 0.001 m3
   30 
  0.16 m  1 L
2
s
 1.49 m/s

V D
R eD1 
v

Substitute corresponding values in the above expression


 1 
R eD1  1.49 m/s  0.16 m   7 2 
 4.10 10 m 
 5.81105

Now calculate,
D
 t
D1

Substitute corresponding values in the above equation


80
  0.5
160

From figure 8.20


K  0.598

Then,
2
m   1   Q   1 
P   actual     
 K  At   2    K  At   2  
2
  Q 
  
2  K  At 

Substitute corresponding values in the above expression


1 kg  L 0.001 m3 1 4 1 
P   980 3 30    
2 m  s 1L 0.598   0.080 m 2 
 48.80 kPa

Hence, the pressure drop is 48.80 kPa .


Problem 8.143 [Difficulty: 2]

8.143 A venturi meter with a 76.2 mm diameter throat is placed


in a 152 mm diameter line carrying water at 24°C. The pressure
drop between the upstream tap and the venturi throat is 305 mm
of mercury. Compute the rate of flow.

Given Flow through a venturi meter.

Find: Flow rate.

Solution:
Basic equation: C ⋅ At C ⋅ At Note that mactual is mass flow rate (the
m actual = ⋅ 2 ⋅ρ⋅ (p1 − p 2 ) = ⋅ 2 ⋅ρ⋅ ∆p software cannot render a dot!)
1 − β4 1 − β4

For Re D1 > 2 × 105 , 0.980 < C < 0.995. Assume C = 0.99, then check Re

Dt 3
β= β= β = 0.5
D1 6

Also ∆p = ρHg ⋅ g ⋅ ∆h = SG Hg ⋅ρ⋅ g ⋅ ∆h

mactual C ⋅ At π ⋅ C ⋅ D 2t π ⋅ C ⋅ D 2t
Then Q= = ⋅ 2 ⋅ρ ⋅ ∆p = ⋅ 2 ⋅ρ ⋅ SG Hg ⋅ ρ ⋅ g ⋅ ∆h = ⋅ 2 ⋅ SG Hg ⋅ g ⋅ ∆h
ρ ρ ⋅ 1 − β4 4 ⋅ρ ⋅ 1 − β4 4 ⋅ 1 − β4

2
π  76.2  m
Q= × 0.99 ×  m  × 2 ×13.6 × 9.81 2 × 0.305 m Q = 0.042 m3 /s
4 × 1 − 0.5 4  1000  s

Q 4⋅Q 4 1 m3 m
Hence V= = V= × × 0.042 V = 2.31
A π ⋅ D12 π  152  2 s s
 m
1000 

m2
At 24°C, (Table A.8) ν = 9.25 × 10−7
s

V ⋅ D1 m 152 s
Re D1 = Re D1 = 2.31 × m× Re D1 = 3.80 × 105
ν s 1000 9.25 × 10 −7 m 2

m3
Thus ReD1 > 2 × 105. The volume flow rate is Q = 0.042
s
Problem 8.144 [Difficulty: 2]

8.144 Consider a horizontal 50 mm × 25 mm venturi with water


flow. For a differential pressure of 150 kPa calculate the
volume flow rate.

Given Flow through an venturi meter.

Find: Flow rate.

Solution:
Basic equation C ⋅ At C ⋅ At Note that mactual is mass flow rate
m actual = ⋅ 2 ⋅ρ⋅ (p1 − p 2 ) = ⋅ 2 ⋅ρ⋅ ∆p (the software cannot render a dot!)
1 − β4 1 − β4

For Re D1 > 2 × 105 , 0.980 < C < 0.995. Assume C = 0.99, then check Re

kg
Available data D1 = 50 mm Dt = 25 mm ∆p = 150 kPa ρ = 1000
m3
Dt
β= β = 0.5 and assume C = 0.99
D1

mactual C ⋅ At
Then Q= = ⋅ 2 ⋅ ρ.∆p
ρ ρ⋅ 1 − β4

π ⋅ C ⋅ D 2t 2 ⋅ ∆p
Q= .
4⋅ 1− β 4 ρ

π N m3 kg ⋅ m m3
Q= × 0.99 × (0.025 mm) 2 × 2 × 150 × 103 × × 2 = 8.69 × 10−3
4 1 − 0.54 m2 1000 kg s N s

Q 4⋅Q
Hence V= V=
A π ⋅ D12

3
4 1 −3 m m
V= × × 8.69 × 10 = 4.43
π (0.05 m) 2 s s

m2 V ⋅ D1
At 20°C, (Table A.8) ν = 1.01× 10−6 Re D1 = Re D1 = 2.19 × 105
s ν

m3
Thus ReD1 > 2 × 105. The volume flow rate is Q = 8.69 × 10−3
s
Problem 8.145 [Difficulty: 2]

8.145 Gasoline flows through a 50 mm × 25 mm venturi meter.


The differential pressure is 380 mm of mercury. Find the
volume flow rate.

Solution: Apply the analysis of Section 8-10.3.

CA t
Computing equations:  actual =
m 2ρ(p1 − p 2 ) (8.52)
1 − p4

C = 0.99 for Re D1 > 2 × 105

For the manometer, ∆p = ρHg g ∆h = SG Hg ρH 2O g∆h


m CA t 2 ∆p CA t 2SG Hg ρH 2O g ∆h CA t 2SG Hg g ∆h
Then Q= = = =
ρ 1 − β4 ρ 1 − β4 SG gas ρH 2O 1 − β4 SG gas

0.99 π 13.6 m
Q= (0.025)2 m 2 2 × × 9.81 2 × 0.38 m = 0.00592 m3 /s
1 − (0.5)44 0.73 s

Now check Reynolds number:

Q 4
V1 = = 0.00592 m3 /s × = 3.02 m/s
A1 π(0.050) 2 m 2

Assume viscosity midway between octane and heptane at 20°C. From Fig. A.1,

µ ≈ 5.0 × 10−4 N ⋅ s / m 2

V1D1 kg m m2 N ⋅ s2
Re D1 = ρ = (0.73) 1000 3 × 3.02 × 0.050 m × − 4
× = 2.20 × 105
µ m s 5.0 × 10 N ⋅ s kg ⋅ m

Thus assumption that C = 0.99 is okay

Q = 0.00592 m3/s Q
Problem 8.146 [Difficulty: 3]

8.146 Air flows through the venturi meter described in


Problem 8.195. Assume that the upstream pressure is 413 kPa,
and that the temperature is everywhere constant at 20°C.
Determine the maximum possible mass flow rate of air for
which the assumption of incompressible flow is a valid
engineering approximation. Compute the corresponding
differential pressure reading on a mercury manometer.

Given: Flow through a venturi meter.

Find: Maximum flow rate for incompressible flow; Pressure reading.

Solution:
Basic equation: C ⋅ At C ⋅ At Note that mactual is mass flow rate
m actual = ⋅ 2 ⋅ρ⋅ (p1 − p 2 ) = ⋅ 2 ⋅ρ⋅ ∆p (the software cannot render a dot!)
4 4
1− β 1− β

Assumptions: 1) Neglect density change 2) Use ideal gas equation for density

p N kg ⋅ K 1
Then ρ= ρ = 413 × 103 2
× × ρ = 4.91 kg/m3
R air ⋅ T m 287 J (20 + 273)K

For incompressible flow V must be less than about 100 m/s at the throat. Hence
2
kg m π  76.2 
m actual = ρ⋅ V2 ⋅ A 2 m actual = 4.91 × 100 × × m m actual = 2.24 kg/s
m3 s 4  1000 

Dt 3
β= β= β = 0.5
D1 6

∆p
Also ∆p = ρHg ⋅ g ⋅ ∆h ∆h =
ρHg ⋅ g

2 2
1  mactual  4 (1 − β 4 )  mactual 
and in addition ∆p = ⋅  ⋅ (1 − β ) so ∆h = ⋅
2 ⋅ρ  C ⋅ A t  2 ⋅ ρ⋅ρHg ⋅ g  C ⋅ A t 

For Re D1 > 2 × 105 , 0.980 < C < 0.995. Assume C = 0.99, then check Re

 1 − 0.5 4  m3 m3 s2  kg 1 4  1000  
2
∆h =   × × × ×  2.24 × × ×  ∆h = 0.177 m
 2
  4.91 kg 13.6 × 998 kg 9.81 m  s 0.99 π  76.2  

Q 4 ⋅ mactual 4 m3 1 m
Hence V= = V= × × × 2.24 kg / s V = 25.1
A π ⋅ρ⋅ D12 π 4.91  152  2 s
 1000 m 
 
m2
At 20°C, (Table A.8) ν = 1⋅ 01 × 10 −6
s
V ⋅ D1 m 152 s
Re D1 = Re D1 = 25.1 × m× Re D1 = 3.78 × 106
ν s 1000 1.01 × 10−6 m 2
Thus ReD1 > 2 × 105. The mass flow rate is m actual = 2.24 kg/s and pressure ∆h = 177 mm Hg
Problem 8.147 [Difficulty: 3]
Problem 8.148

Given: Flow through venture

To find: Maximum flow rate before cavitation

Solution:

Basic equation:

C  At C  At
mactual   2   p1  p2   2 p
1  4 1  4

Note that mactual is mass flow rate

For ReD1  2 105 , 0.980  c  0.995. Assume c  0.99 , the check Re

Available data

D1  110mm Dt  55mm p1g  250kPa


c  0.99  assumption - verify later 
patm  101kPa
pv  2.34kPa  steam tables - saturation pressure at 20° 
kg m2
  998 v  1.01106  table A-8 
m3 s

Then,

 Dt 2 
At     55  mm 2  2375.83mm 2
2

4 4
 D1 
2
A1    110  mm 2  9503.32mm 2
2

4 4
D 55mm
 t   0.5
D1 110mm
p1  patm  p1g
101kPa+250kPa
p1  351kPa
The smallest allowable throat pressure is the saturation pressure

pt  pv pt  2.34kPa

Hence the largest p is

p  p1  pt  349kPa
Then,
C  At
mrate  2 p
1  4
kg
mrate  64.16
s
mrate kg 1 m3 m3
Q  64.16   0.0642
 s 998 kg s
Q m3 106 m
V1   0.0642  2
 6.75
A1 s 9503.32m s

Check the Re

V1  D1 m 110m 106 s
Re D1   6.75    2
v s 1000 1.01 m
Re D1  7.3510 5

Thus Re D1  2 105 . The volume flow rate is

m3 L
Q  0.0642 Q  64.2
s s
Problem 8.149 [Difficulty: 1]

V 1, A 1 V 2, A 2

Given: Flow through a diffuser

Find: Derivation of Eq. 8.42

Solution:
2 2
p2 − p1 p1 V1 p2 V2
Basic equations Cp = + + g ⋅ z1 = + + g ⋅ z2 Q = V⋅ A
1 2 ρ 2 ρ 2
⋅ ρ⋅ V1
2

Assumptions: 1) All the assumptions of the Bernoulli equation 2) Horizontal flow 3) No flow separation

2 2 2 2 2
p2 − p1 V1 V2⎛ A1 ⎞ V1 V1
From Bernoulli = − = − ⋅⎜ using continuity
2 2 2 2
ρ
⎝ A2 ⎠
⎡ V 2 V 2 ⎛ A ⎞ 2⎤ 2
p2 − p1 ⎢ 1
1 1 1 ⎥ ⎛ A1 ⎞
Hence Cp = ⋅⎜ ⎜A
1
=
1

⎢ 2

2 ⎥=1−
⋅ ρ⋅ V1
2 2
⋅ V1 ⎣ ⎝ A2 ⎠ ⎦ ⎝ 2⎠
2 2

1
Finally Cp = 1 − which is Eq. 8.42.
2
AR

This result is not re alistic as a real diffuser is very likely to have flow separation
Problem 8.150 [Difficulty: 4] Part 1/2
[Difficulty: 4] Part 2/2
Problem 8.151 [Difficulty: 5] Part 1/2

8.151 In some western states, water for mining and irrigation


was sold by the “minner’s inch,” the rate at which water flows
2
through an opening in a vertical plank of 645 mm area, up to
102 mm tall, under a head of 152 to 229 mm. Develop an
equation to predict the flow rate through such an orifice.
Specify clearly the aspect ratio of the opening, thickness of the
plank, and datum level for measurement of head (top, bottom,
or middle of the opening). Show that the unit of measure varies
from 38.4 (in Colorado) to 50 (in Arizona, Idaho, Nevada, and
3
Utah) miner’s inches equal to 0.0283 m /s.

Analysis: The geometry of the opening in a vertical plank is shown. The analysis includes the effect on flow speed of the
variation in water depth vertically across the opening.

H0
V= 2gy Aspect ratio, ar ≡ ; area, A = wH0 = 1 in.2
y a W w
H
b α = 0, ½, 1
αHo
Ho
dy

b  2 3/2 
b
2  b  3 2 

Qgeom = VdA = ∫a
2gy w dy = w 2g  y  = wa 2ga    − 1
3 a 3   a  

For ar = 1, α = 0, a = H = 229 mm, b = 254 mm, w = 25 mm.

 2  254  2  m 2
1 3
2  m m
Qgeom = × 25 mm × 229 mm  2 × 9.8 2 × 229 mm ×  ×   229  − 1 6 2
= 1.4 × 10−3 m3 /s
  
3  s 1000 mm  10 mm

m3 1 m3
Qactual = 0.6 Qgeom = 8.4 × 10−4 ; thus = 190.5 MI = 0.0283
s 8.4 × 10−4 s

Numerical results are presented in the spread sheet on the next page.

Discussion: All results assume a vena contracta in the liquid jet leaving the opening, reducing the effective flow area to 60 percent
of the geometric area of the opening.

The calculated unit of measure varies from 1105 to 1850 miner’s inch per cubic meter of water flow per second. This range
encompasses the 38.4 and 50 values given in the problem statement.

Trends may be summarized as follows. The largest flow rate occurs when datum H is measured to the top of the opening in the
vertical plank. This gives the deepest submergence and thus the highest flow speeds through the opening.

When ar = 1, the opening is square; when ar = 16, the opening is 102 mm tall and 1/102 mm wide. Increasing ar from 1 to 16
increases the flow rate through the opening when H is measured to the top of the opening, because it increases the submergence of
the lower portion of the opening, thus increasing the flow speeds. When H is measured to the center of the opening ar has almost
no effect on flow rate. When H is measured to the bottom of the opening, increasing ar reduces the flow rate. For this case, the
depth of the opening decreases as ar becomes larger.
[Difficulty: 5] Part 2/2
Plank thickness does not affect calculated flow rates since a vena contracta is assumed. In this flow model, water separates from
the interior edges of the opening in the vertical plank. Only if the plank were several inches thick might the stream reattach and
affect the flow rate.

The actual relationship between Qflow and Qgeom might be a weak function of aspect ratio. The flow separates from all four edges of
the opening in the vertical plank. At large ar, contraction on the narrow ends of the stream has a relatively small effect on flow
area. As ar approaches 1 the effect becomes more pronounced, but would need to be measured experimentally. Assuming a
constant 60 percent area fraction certainly gives reasonable trends.

Computation of “Miner’s Inch” in Engineering Units:


a = depth to top of opening (mm)
a r = aspect ratio of opening (---)
A = area of opening 645 mm2
b = depth to bottom of opening (mm)
H = nominal head (mm)
H 0 = height of opening (mm)
MI = “miner’s inch” (mixed)
Q = volume flow rate (m3/s)
w = width of opening (mm)

Assume Qflow = 0.6 × Qgeometric to account for contraction of the stream leaving the opening.

(a) Measure H to top of opening


H ar Ho a b w Q Q, (x 10−4) MI/m3
229 1 25 229 254 25 0.0014 8.4 1190.5
229 2 36 229 264 18 0.00139 8.34 1199
229 4 51 229 279 12.8 0.00143 8.58 1165.5
229 8 72 229 300 9 0.00145 8.7 1149.4
229 16 102 229 330 6.4 0.00151 9.06 1103.8

152 1 25 152 178 25 0.00117 7.02 1424.5


152 2 36 152 188 18 0.00118 7.08 1412.4
152 4 51 152 203 12.8 0.00122 7.32 1366
152 8 72 152 224 9 0.00124 7.44 1344
152 16 102 152 254 6.4 0.0013 7.8 1282

(b) Measure H to middle of opening:


H ar Ho a b w Q Q, (x 10−4) MI/m3
229 1 25 216 241.3 25 0.00134 8.04 1244
229 2 36 210.6 247 18 0.00139 8.34 1199
229 4 51 203 254 12.8 0.00138 8.28 1208
229 8 72 193 264 9 0.00135 8.1 1235
229 16 102 178 279 6.4 0.00136 8.16 1225

152 1 25 140 165 25 0.00108 6.48 1543


152 2 36 134.3 170.4 18 0.00112 6.72 1488
152 4 51 127 179 12.8 0.00115 6.9 1449
152 8 72 116.6 188 9 0.00111 6.66 1502
152 16 102 102 203 6.4 0.00111 6.66 1502

(c) Measure H to bottom of opening:


H ar Ho a b w Q Q, (x 10−4) MI/m3
229 1 25 203 229 25 0.00134 8.04 1244
229 2 36 193 229 18 0.00132 7.92 1263
229 4 51 178 229 12.8 0.0013 7.8 1282
229 8 72 156.7 229 9 0.00126 7.56 1323
229 16 102 127 229 6.4 0.00122 7.32 1366
Problem 8.152 [Difficulty: 5] Part 1/2
[Difficulty: 5] Part 2/2

You might also like